[obm-l] Re: [obm-l] Re: [obm-l] problemas fáceis pro ensino fundamental

2018-08-02 Por tôpico Claudio Buffara
Oi, Artur:

Fiz alguns comentários (abaixo) sobre os 3 problemas que você mencionou.

[]s,
Claudio.

2018-08-01 15:48 GMT-03:00 Artur Steiner :

> Pelo que já vi, a esmagadora maioria dos alunos do ensino médio teria
> muita dificuldade e nenhum interesse nesses problemas mais elaborados. O
> fato é que pouquíssimas pessoas apreciam matemática. A maioria odeia.
>
> É difícil apreciar algo que você nunca aprendeu direito e o máximo que fez
foi memorizar algumas fórmulas e alguns procedimentos que teve que
regurgitar numa prova.
A meu ver, um dos grandes problemas da matemática escolar é que os
professores raramente explicam o porquê das coisas, não mencionam que
problema prático as fórmulas e procedimentos foram criados pra resolver (o
que seria, a meu ver, a forma correta de contextualizar a matemática) e nem
como o criador da fórmula chegou até ela (muito provavelmente
experimentando, analisando casos particulares, raciocinando por analogia,
procurando invariantes, analisando situações extremas, etc - justamente as
técnicas que são ensinadas em cursos preparatórios pra olimpíadas).

Vou dar 3 problemas bem mais  simples do que os que vc deu e que quase todo
> mundo erra. Já vi bons engenheiros errando. E muitos teimam em suas
> respostas erradas.
> 1. Suponha que a Terra seja uma esfera perfeita e que ao redor dela seja
> passado um fio de espessura desprezível formando um círculo concêntrico com
> a Terra. Se o comprimento do fio exceder de 1 m a circunferência da Terra,
> dê exemplo de um animal que passaria sob o fio sem tocá- lo. Muitos dizem,
> ora talvez um micróbio.  Um pardal certamente não passaria. O fio está
> praticamente colado no solo.
>
> No entanto, acho que boa parte dos alunos de EM e certamente dos
engenheiros conhece a fórmula do comprimento da circunferência em função do
raio (C = 2*Pi*r).
O que falta é conseguir raciocinar com base nesta fórmula a fim de resolver
este problema.


> 2., Um ônibus percorreu a1a metade de seu trajeto com velocidade média de
> 80 km/h. Na segunda metade, o trânsito estava ruim e a velocidade média foi
> de apenas 20 km/h. Pode-se então afirmar que a velocidade média ao longo de
> todo o trajeto foi de ..
>
> A maioria diz 50 km/h.
>
> Mais uma vez estamos diante de um caso de falta de raciocínio. Quem
responde 50 km/h simplesmente olha pros números 80 e 20, pra palavra
"média" e daí, automaticamente calcula a média aritmética de 80 e 20. Pode
ser um caso de "resposta automática indevida" (um fenômeno bastante
estudado pelo Daniel Kahneman). No entanto, se a pessoa pensar um pouco
sobre o significado de "velocidade" (ok, a pessoa tem que memorizar algumas
definições) vai conseguir entender porque 50 km/h não é a resposta correta.
Também é interessante que muita gente sabe que velocidade =
distância/tempo, mas a maioria acha estranho (e, de fato, nem pensa em)
escrever tempo = distância/velocidade, que é a mesma coisa.


> 3. 99% da massa de uma melancia de 1 kg é composta por água. A melancia é
> exposta ao sol e, devido à evaporação, a água passa a representar 98% da
> massa total. Qual a nova massa da melancia? Muitos dão um valor muito
> próximo de 1kg e teimam.
>
> Outro problema que as pessoas erram por preguiça de raciocinar.
Se 99% da massa é água, então a massa seca é de 1% de 1kg ou 10 g. Este é o
INVARIANTE do problema.
Depois da evaporação, a massa seca (que permanece em 10g) passa a ser 2% da
massa total. Logo, a nova massa total é 10/0,02 = 500 g.

Os 3 problemas que você mencionou envolvem, de uma forma ou outra, a
importante noção de proporcionalidade, que é muito enfatizada no currículo
do Ensino Fundamental e também na prova do Enem. Obviamente não está sendo
bem ensinada.

E tem aquilo que se fazia quando eu era garoto e impressionava muitos, como
> mágica: Pense um número e não o diga. Multiplique por  2. Some 10. Divida a
> soma por 2. Desta soma, deduza o número que vc pensou. Pronto? Sim! Deu 5.
> É mesmo! Vc adivinhou meu número?
>
>

> Artur Costa Steiner
>
> Em qua, 1 de ago de 2018 12:38, Claudio Buffara 
> escreveu:
>
>> Um dos atrativos da matemática (pelo menos pra mim) é a existência de
>> problemas que estão em aberto há décadas (ou séculos) mas cujos enunciados
>> podem ser facilmente compreendidos por alunos de ensino fundamental. Três
>> exemplos são a conjectura de Goldbach, a conjectura de que existe uma
>> infinidade de primos gêmeos, e a conjectura de Collatz.
>>
>> É interessante que existem 3 problemas elementares cujos enunciados são
>> parecidos com os das conjecturas acima:
>> 1) Prove que todo número natural maior do que 11 pode ser escrito como a
>> soma de dois números compostos;
>> 2) Encontre todos os “primos trigêmeos” (trios de números primos que
>> diferem por 2, tais como 3, 5 e 7);
>> 3) O primeiro termo de uma sequência é 10. Cada termo seguinte é igual à
>> metade do termo anterior, se este for par, ou 7 unidades maior do que o
>> termo anterior, se este for ímpar. Qual o 2018º termo da sequência?

[obm-l] Re: [obm-l] Re: [obm-l] Re: [obm-l] Re: [obm-l] Re: [obm-l] problemas fáceis pro ensino fundamental

2018-08-01 Por tôpico Rodrigo Ângelo
É que você só analisou os primeiros termos da sequência. No seu argumento
não tem nada que garante que a partir do vigésimo termo ela não passe a ter
ciclo diferente de 5 (ou mesmo que ela não deixe de ser cíclica). Teria que
ter algo tipo: Dados 6 termos consecutivos quaisquer dessa sequência a1,
a2, a3, a4, a5, a6; provar que a1 = a6.

On Wed, Aug 1, 2018 at 2:13 PM Olson  wrote:

> Não basta afirmar que a sequência se repete?
>
> Em qua, 1 de ago de 2018 15:25, Claudio Buffara 
> escreveu:
>
>> A solução do 3 está correta mas também incompleta. Como você observou, a
>> sequência é periódica de período 5. Mas esta afirmação precisa ser
>> justificada. Repare que você concluiu algo sobre todos os termos da
>> sequência (ou pelo menos sobre os primeiros 2018 termos) mediante a
>> observação de somente 11 termos.
>>
>> Enviado do meu iPhone
>>
>> Em 1 de ago de 2018, à(s) 14:57, Arthur Vieira 
>> escreveu:
>>
>> Problema 3:
>> Ao analisar os primeiros termos da sequência temos
>> 10-5-12-6-3-10-5-12-6-3-10-...
>> A sequência se repete a cada 5 números.
>> Assim podemos dividir a sequência em "bloquinhos" de 5 números cada
>> (10,5,12,6,3, nessa ordem)
>> Como queremos o 2018o termo da sequência basta dividir 2018 por 5 e
>> observar o resto, que é 3. isso significa que o número é o 3o dentro do
>> "bloquinho"
>> Resposta: 12
>>
>> Em 1 de agosto de 2018 14:33, Arthur Vieira 
>> escreveu:
>>
>>> Acho que consegui uma solução para o ultimo problema:
>>> Somar esses dois primos consecutivos e dividir por dois é o mesmo que
>>> fazer a média aritmética entre eles.
>>> Essa média aritmética é maior que o primeiro primo e menor que o
>>> segundo primo.
>>> Por definição, só existem compostos entre eles, ou seja, é
>>> impossível que o resultado seja um número primo.
>>>
>>
>>
>> --
>> Esta mensagem foi verificada pelo sistema de antivírus e
>> acredita-se estar livre de perigo.
>>
>>
>> --
>> Esta mensagem foi verificada pelo sistema de antivírus e
>> acredita-se estar livre de perigo.
>>
>
> --
> Esta mensagem foi verificada pelo sistema de antivírus e
> acredita-se estar livre de perigo.

-- 
Esta mensagem foi verificada pelo sistema de antiv�rus e
 acredita-se estar livre de perigo.



Re: [obm-l] Re: [obm-l] Re: [obm-l] Re: [obm-l] Re: [obm-l] problemas fáceis pro ensino fundamental

2018-08-01 Por tôpico Claudio Buffara
Não. Esta é uma constatação (correta, é claro) mas baseada apenas na observação 
de uns poucos termos da sequência. Pode ser que falhe mais adiante. 

Por exemplo, f(n) = n^2 - n + 41 é primo para todo natural n de 0 a 40. Mas 
f(41) é composto.

Pra justificar a periodicidade da sequência do problema, vc poderia dizer, por 
exemplo, que como cada termo é expresso como função apenas do termo anterior, 
assim que um determinado termo se repetir (no caso, o 10), todos os seguintes 
se repetirão, na mesma ordem e com a mesma periodicidade.

Enviado do meu iPhone

Em 1 de ago de 2018, à(s) 16:02, Olson  escreveu:

> Não basta afirmar que a sequência se repete?
> 
> Em qua, 1 de ago de 2018 15:25, Claudio Buffara  
> escreveu:
>> A solução do 3 está correta mas também incompleta. Como você observou, 
>> a sequência é periódica de período 5. Mas esta afirmação precisa ser 
>> justificada. Repare que você concluiu algo sobre todos os termos da 
>> sequência (ou pelo menos sobre os primeiros 2018 termos) mediante a 
>> observação de somente 11 termos. 
>> 
>> Enviado do meu iPhone
>> 
>> Em 1 de ago de 2018, Ã (s) 14:57, Arthur Vieira  
>> escreveu:
>> 
>>> Problema 3:
>>> Ao analisar os primeiros termos da sequência temos
>>> 10-5-12-6-3-10-5-12-6-3-10-...
>>> A sequência se repete a cada 5 números.
>>> Assim podemos dividir a sequência em "bloquinhos" de 5 números cada 
>>> (10,5,12,6,3, nessa ordem)
>>> Como queremos o 2018o termo da sequência basta dividir 2018 por 5 e 
>>> observar o resto, que é 3. isso significa que o número é o 3o 
>>> dentro do "bloquinho"
>>> Resposta: 12
>>> 
>>> Em 1 de agosto de 2018 14:33, Arthur Vieira  escreveu:
 Acho que consegui uma solução para o ultimo problema:
 Somar esses dois primos consecutivos e dividir por dois é o mesmo que 
 fazer a média aritmética entre eles.
 Essa média aritmética é maior que o primeiro primo e menor que o 
 segundo primo.
 Por definição, só existem compostos entre eles, ou seja, é 
 impossível que o resultado seja um número primo.
>>> 
>>> 
>>> -- 
>>> Esta mensagem foi verificada pelo sistema de antivírus e 
>>> acredita-se estar livre de perigo.
>> 
>> -- 
>> Esta mensagem foi verificada pelo sistema de antivírus e 
>> acredita-se estar livre de perigo.
> 
> -- 
> Esta mensagem foi verificada pelo sistema de antivírus e 
> acredita-se estar livre de perigo.

-- 
Esta mensagem foi verificada pelo sistema de antiv�rus e
 acredita-se estar livre de perigo.



Re: [obm-l] Re: [obm-l] problemas fáceis pro ensino fundamental

2018-08-01 Por tôpico Claudio Buffara
Acredito.

Por isso acho que a matemática está sendo ensinada de forma errada - conteúdo 
errado e metodologia errada. E acho que o problema começa no Ensino 
Fundamental, com alunos de 6, 7 ou 8 anos, cujos professores não têm preparo 
adequado pra ensinar matemática (basta ver o currículo dos cursos de pedagogia, 
que é de onde saem os professores e professoras do 1o ao 5o ano).

O resultado: milhares de alunos que não dominam a matemática básica (na qual 
enquadro os 3 problemas que você mencionou, que envolvem conceitos que fazem 
parte do currículo atual do Ensino Fundamental).

O que falta ‘a maioria dos alunos de EM e até aos bons engenheiros que não 
conseguem resolvê-los não é conhecimento ou inteligência, mas sim a habilidade 
de raciocinar matematicamente, habilidade esta que deveria ser adquirida e 
exercitada na escola, mas claramente não é.


Enviado do meu iPhone

Em 1 de ago de 2018, à(s) 15:48, Artur Steiner  
escreveu:

> Pelo que já vi, a esmagadora maioria dos alunos do ensino médio teria muita 
> dificuldade e nenhum interesse nesses problemas mais elaborados. O fato é 
> que pouquíssimas pessoas apreciam matemática. A maioria odeia. 
> 
> Vou dar 3 problemas bem mais  simples do que os que vc deu e que quase todo 
> mundo erra. Já vi bons engenheiros errando. E muitos teimam em suas 
> respostas erradas.  
> 
> 1. Suponha que a Terra seja uma esfera perfeita e que ao redor dela seja 
> passado um fio de espessura desprezível formando um círculo concêntrico 
> com a Terra. Se o comprimento do fio exceder de 1 m a circunferência da 
> Terra, dê exemplo de um animal que passaria sob o fio sem tocá- lo. Muitos 
> dizem, ora talvez um micróbio.  Um pardal certamente não passaria. O fio 
> está praticamente colado no solo.
> 
> 2., Um ônibus percorreu a1a metade de seu trajeto com velocidade média de 
> 80 km/h. Na segunda metade, o trânsito estava ruim e a velocidade média foi 
> de apenas 20 km/h. Pode-se então afirmar que a velocidade média ao longo de 
> todo o trajeto foi de ..
> 
> A maioria diz 50 km/h.
> 
> 3. 99% da massa de uma melancia de 1 kg é composta por água. A melancia é 
> exposta ao sol e, devido à evaporação, a água passa a representar 98% da 
> massa total. Qual a nova massa da melancia? Muitos dão um valor muito 
> próximo de 1kg e teimam.
> 
> E tem aquilo que se fazia quando eu era garoto e impressionava muitos, como 
> mágica: Pense um número e não o diga. Multiplique por  2. Some 10. Divida 
> a soma por 2. Desta soma, deduza o número que vc pensou. Pronto? Sim! Deu 5. 
> É mesmo! Vc adivinhou meu número?
> 
> Artur Costa Steiner
> 
> Em qua, 1 de ago de 2018 12:38, Claudio Buffara  
> escreveu:
>> Um dos atrativos da matemática (pelo menos pra mim) é a existência de 
>> problemas que estão em aberto há décadas (ou séculos) mas cujos 
>> enunciados podem ser facilmente compreendidos por alunos de ensino 
>> fundamental. Três exemplos são a conjectura de Goldbach, a conjectura de 
>> que existe uma infinidade de primos gêmeos, e a conjectura de Collatz.
>> 
>> É interessante que existem 3 problemas elementares cujos enunciados são 
>> parecidos com os das conjecturas acima:
>> 1) Prove que todo número natural maior do que 11 pode ser escrito como a 
>> soma de dois números compostos;
>> 2) Encontre todos os “primos trigêmeos” (trios de números primos que 
>> diferem por 2, tais como 3, 5 e 7);
>> 3) O primeiro termo de uma sequência é 10. Cada termo seguinte é igual à 
>>  metade do termo anterior, se este for par, ou 7 unidades maior do que o 
>> termo anterior, se este for ímpar. Qual o 2018º termo da sequência?
>> 
>> Como vocês podem verificar, os três problemas são fáceis, ainda que, 
>> pra resolvê-los, sejam necessários um mínimo de raciocínio e alguma 
>> experimentação.
>> 
>> Mas o que eu quero saber é se um aluno normal de 7o ou 8o ano (de 12 a 14 
>> anos de idade, em média) seria capaz de resolver tais problemas.
>> O que vocês acham?
>> 
>> E será que um aluno de 6o ano (11-12 anos) seria capaz de explicar porque a 
>> soma de dois números primos consecutivos não pode ser igual ao dobro de um 
>> número primo?
>> 
>> OBS: Todos estes problemas envolvem apenas conceitos que são vistos antes 
>> do 6o ano: operações com números naturais e números pares, ímpares, 
>> primos e compostos.
>> 
>> []s,
>> Claudio.
>> 
>> 
>> -- 
>> Esta mensagem foi verificada pelo sistema de antivírus e 
>> acredita-se estar livre de perigo.
> 
> -- 
> Esta mensagem foi verificada pelo sistema de antivírus e 
> acredita-se estar livre de perigo.

-- 
Esta mensagem foi verificada pelo sistema de antiv�rus e
 acredita-se estar livre de perigo.



[obm-l] Re: [obm-l] Re: [obm-l] Re: [obm-l] problemas fáceis pro ensino fundamental

2018-08-01 Por tôpico Arthur Vieira
Acho que consegui uma solução para o ultimo problema:
Somar esses dois primos consecutivos e dividir por dois é o mesmo que fazer
a média aritmética entre eles.
Essa média aritmética é maior que o primeiro primo e menor que o segundo
primo.
Por definição, só existem compostos entre eles, ou seja, é impossível que o
resultado seja um número primo.

-- 
Esta mensagem foi verificada pelo sistema de antiv�rus e
 acredita-se estar livre de perigo.



[obm-l] Re: [obm-l] Re: [obm-l] Re: [obm-l] Re: [obm-l] problemas fáceis pro ensino fundamental

2018-08-01 Por tôpico Olson
Não basta afirmar que a sequência se repete?

Em qua, 1 de ago de 2018 15:25, Claudio Buffara 
escreveu:

> A solução do 3 está correta mas também incompleta. Como você observou, a
> sequência é periódica de período 5. Mas esta afirmação precisa ser
> justificada. Repare que você concluiu algo sobre todos os termos da
> sequência (ou pelo menos sobre os primeiros 2018 termos) mediante a
> observação de somente 11 termos.
>
> Enviado do meu iPhone
>
> Em 1 de ago de 2018, à(s) 14:57, Arthur Vieira 
> escreveu:
>
> Problema 3:
> Ao analisar os primeiros termos da sequência temos
> 10-5-12-6-3-10-5-12-6-3-10-...
> A sequência se repete a cada 5 números.
> Assim podemos dividir a sequência em "bloquinhos" de 5 números cada
> (10,5,12,6,3, nessa ordem)
> Como queremos o 2018o termo da sequência basta dividir 2018 por 5 e
> observar o resto, que é 3. isso significa que o número é o 3o dentro do
> "bloquinho"
> Resposta: 12
>
> Em 1 de agosto de 2018 14:33, Arthur Vieira 
> escreveu:
>
>> Acho que consegui uma solução para o ultimo problema:
>> Somar esses dois primos consecutivos e dividir por dois é o mesmo que
>> fazer a média aritmética entre eles.
>> Essa média aritmética é maior que o primeiro primo e menor que o
>> segundo primo.
>> Por definição, só existem compostos entre eles, ou seja, é
>> impossível que o resultado seja um número primo.
>>
>
>
> --
> Esta mensagem foi verificada pelo sistema de antivírus e
> acredita-se estar livre de perigo.
>
>
> --
> Esta mensagem foi verificada pelo sistema de antivírus e
> acredita-se estar livre de perigo.
>

-- 
Esta mensagem foi verificada pelo sistema de antiv�rus e
 acredita-se estar livre de perigo.



[obm-l] Re: [obm-l] problemas fáceis pro ensino fundamental

2018-08-01 Por tôpico Artur Steiner
Pelo que já vi, a esmagadora maioria dos alunos do ensino médio teria muita
dificuldade e nenhum interesse nesses problemas mais elaborados. O fato é
que pouquíssimas pessoas apreciam matemática. A maioria odeia.

Vou dar 3 problemas bem mais  simples do que os que vc deu e que quase todo
mundo erra. Já vi bons engenheiros errando. E muitos teimam em suas
respostas erradas.

1. Suponha que a Terra seja uma esfera perfeita e que ao redor dela seja
passado um fio de espessura desprezível formando um círculo concêntrico com
a Terra. Se o comprimento do fio exceder de 1 m a circunferência da Terra,
dê exemplo de um animal que passaria sob o fio sem tocá- lo. Muitos dizem,
ora talvez um micróbio.  Um pardal certamente não passaria. O fio está
praticamente colado no solo.

2., Um ônibus percorreu a1a metade de seu trajeto com velocidade média de
80 km/h. Na segunda metade, o trânsito estava ruim e a velocidade média foi
de apenas 20 km/h. Pode-se então afirmar que a velocidade média ao longo de
todo o trajeto foi de ..

A maioria diz 50 km/h.

3. 99% da massa de uma melancia de 1 kg é composta por água. A melancia é
exposta ao sol e, devido à evaporação, a água passa a representar 98% da
massa total. Qual a nova massa da melancia? Muitos dão um valor muito
próximo de 1kg e teimam.

E tem aquilo que se fazia quando eu era garoto e impressionava muitos, como
mágica: Pense um número e não o diga. Multiplique por  2. Some 10. Divida a
soma por 2. Desta soma, deduza o número que vc pensou. Pronto? Sim! Deu 5.
É mesmo! Vc adivinhou meu número?

Artur Costa Steiner

Em qua, 1 de ago de 2018 12:38, Claudio Buffara 
escreveu:

> Um dos atrativos da matemática (pelo menos pra mim) é a existência de
> problemas que estão em aberto há décadas (ou séculos) mas cujos enunciados
> podem ser facilmente compreendidos por alunos de ensino fundamental. Três
> exemplos são a conjectura de Goldbach, a conjectura de que existe uma
> infinidade de primos gêmeos, e a conjectura de Collatz.
>
> É interessante que existem 3 problemas elementares cujos enunciados são
> parecidos com os das conjecturas acima:
> 1) Prove que todo número natural maior do que 11 pode ser escrito como a
> soma de dois números compostos;
> 2) Encontre todos os “primos trigêmeos” (trios de números primos que
> diferem por 2, tais como 3, 5 e 7);
> 3) O primeiro termo de uma sequência é 10. Cada termo seguinte é igual à
> metade do termo anterior, se este for par, ou 7 unidades maior do que o
> termo anterior, se este for ímpar. Qual o 2018º termo da sequência?
>
> Como vocês podem verificar, os três problemas são fáceis, ainda que, pra
> resolvê-los, sejam necessários um mínimo de raciocínio e alguma
> experimentação.
>
> Mas o que eu quero saber é se um aluno normal de 7o ou 8o ano (de 12 a 14
> anos de idade, em média) seria capaz de resolver tais problemas.
> O que vocês acham?
>
> E será que um aluno de 6o ano (11-12 anos) seria capaz de explicar porque
> a soma de dois números primos consecutivos não pode ser igual ao dobro de
> um número primo?
>
> OBS: Todos estes problemas envolvem apenas conceitos que são vistos antes
> do 6o ano: operações com números naturais e números pares, ímpares, primos
> e compostos.
>
> []s,
> Claudio.
>
>
> --
> Esta mensagem foi verificada pelo sistema de antivírus e
> acredita-se estar livre de perigo.

-- 
Esta mensagem foi verificada pelo sistema de antiv�rus e
 acredita-se estar livre de perigo.



Re: [obm-l] Re: [obm-l] Re: [obm-l] Re: [obm-l] problemas fáceis pro ensino fundamental

2018-08-01 Por tôpico Claudio Buffara
A solução do 3 está correta mas também incompleta. Como você observou, a 
sequência é periódica de período 5. Mas esta afirmação precisa ser justificada. 
Repare que você concluiu algo sobre todos os termos da sequência (ou pelo menos 
sobre os primeiros 2018 termos) mediante a observação de somente 11 termos. 

Enviado do meu iPhone

Em 1 de ago de 2018, à(s) 14:57, Arthur Vieira  escreveu:

> Problema 3:
> Ao analisar os primeiros termos da sequência temos
> 10-5-12-6-3-10-5-12-6-3-10-...
> A sequência se repete a cada 5 números.
> Assim podemos dividir a sequência em "bloquinhos" de 5 números cada 
> (10,5,12,6,3, nessa ordem)
> Como queremos o 2018o termo da sequência basta dividir 2018 por 5 e observar 
> o resto, que é 3. isso significa que o número é o 3o dentro do "bloquinho"
> Resposta: 12
> 
> Em 1 de agosto de 2018 14:33, Arthur Vieira  escreveu:
>> Acho que consegui uma solução para o ultimo problema:
>> Somar esses dois primos consecutivos e dividir por dois é o mesmo que fazer 
>> a média aritmética entre eles.
>> Essa média aritmética é maior que o primeiro primo e menor que o segundo 
>> primo.
>> Por definição, só existem compostos entre eles, ou seja, é impossível 
>> que o resultado seja um número primo.
> 
> 
> -- 
> Esta mensagem foi verificada pelo sistema de antivírus e 
> acredita-se estar livre de perigo.

-- 
Esta mensagem foi verificada pelo sistema de antiv�rus e
 acredita-se estar livre de perigo.



[obm-l] Re: [obm-l] Re: [obm-l] Re: [obm-l] problemas fáceis pro ensino fundamental

2018-08-01 Por tôpico Arthur Vieira
Problema 3:
Ao analisar os primeiros termos da sequência temos
10-5-12-6-3-10-5-12-6-3-10-...
A sequência se repete a cada 5 números.
Assim podemos dividir a sequência em "bloquinhos" de 5 números cada
(10,5,12,6,3, nessa ordem)
Como queremos o 2018o termo da sequência basta dividir 2018 por 5 e
observar o resto, que é 3. isso significa que o número é o 3o dentro do
"bloquinho"
Resposta: 12

Em 1 de agosto de 2018 14:33, Arthur Vieira  escreveu:

> Acho que consegui uma solução para o ultimo problema:
> Somar esses dois primos consecutivos e dividir por dois é o mesmo que
> fazer a média aritmética entre eles.
> Essa média aritmética é maior que o primeiro primo e menor que o segundo
> primo.
> Por definição, só existem compostos entre eles, ou seja, é impossível que
> o resultado seja um número primo.
>

-- 
Esta mensagem foi verificada pelo sistema de antiv�rus e
 acredita-se estar livre de perigo.



[obm-l] Re: [obm-l] Re: [obm-l] problemas fáceis pro ensino fundamental

2018-08-01 Por tôpico Claudio Buffara
E, é claro, dois primos gêmeos, tais como 3 e 5, são também primos
consecutivos. Mas a recíproca nem sempre é verdadeira.

2018-08-01 14:03 GMT-03:00 Claudio Buffara :

> Não. Estes são os "primos gêmeos" ("twin primes").
>
> Primos consecutivos são os que só têm compostos entre eles. Por exemplo:
> 13 e 17   ou  31 e 37.
>
> 2018-08-01 13:51 GMT-03:00 Arthur Vieira :
>
>> Por primos consecutivos você quer dizer primos que tem diferença de duas
>> unidades?
>>
>> Em 1 de agosto de 2018 12:30, Claudio Buffara 
>> escreveu:
>>
>>> Um dos atrativos da matemática (pelo menos pra mim) é a existência de
>>> problemas que estão em aberto há décadas (ou séculos) mas cujos enunciados
>>> podem ser facilmente compreendidos por alunos de ensino fundamental. Três
>>> exemplos são a conjectura de Goldbach, a conjectura de que existe uma
>>> infinidade de primos gêmeos, e a conjectura de Collatz.
>>>
>>> É interessante que existem 3 problemas elementares cujos enunciados são
>>> parecidos com os das conjecturas acima:
>>> 1) Prove que todo número natural maior do que 11 pode ser escrito como
>>> a soma de dois números compostos;
>>> 2) Encontre todos os “primos trigêmeos” (trios de números primos que
>>> diferem por 2, tais como 3, 5 e 7);
>>> 3) O primeiro termo de uma sequência é 10. Cada termo seguinte é igual
>>> à metade do termo anterior, se este for par, ou 7 unidades maior do que o
>>> termo anterior, se este for ímpar. Qual o 2018º termo da sequência?
>>>
>>> Como vocês podem verificar, os três problemas são fáceis, ainda que,
>>> pra resolvê-los, sejam necessários um mínimo de raciocínio e alguma
>>> experimentação.
>>>
>>> Mas o que eu quero saber é se um aluno normal de 7o ou 8o ano (de 12 a
>>> 14 anos de idade, em média) seria capaz de resolver tais problemas.
>>> O que vocês acham?
>>>
>>> E será que um aluno de 6o ano (11-12 anos) seria capaz de explicar
>>> porque a soma de dois números primos consecutivos não pode ser igual ao
>>> dobro de um número primo?
>>>
>>> OBS: Todos estes problemas envolvem apenas conceitos que são vistos
>>> antes do 6o ano: operações com números naturais e números pares, ímpares,
>>> primos e compostos.
>>>
>>> []s,
>>> Claudio.
>>>
>>>
>>> --
>>> Esta mensagem foi verificada pelo sistema de antivírus e
>>> acredita-se estar livre de perigo.
>>
>>
>>
>> --
>> Esta mensagem foi verificada pelo sistema de antivírus e
>> acredita-se estar livre de perigo.
>
>
>

-- 
Esta mensagem foi verificada pelo sistema de antiv�rus e
 acredita-se estar livre de perigo.



[obm-l] Re: [obm-l] Re: [obm-l] problemas fáceis pro ensino fundamental

2018-08-01 Por tôpico Claudio Buffara
Não. Estes são os "primos gêmeos" ("twin primes").

Primos consecutivos são os que só têm compostos entre eles. Por exemplo: 13
e 17   ou  31 e 37.

2018-08-01 13:51 GMT-03:00 Arthur Vieira :

> Por primos consecutivos você quer dizer primos que tem diferença de duas
> unidades?
>
> Em 1 de agosto de 2018 12:30, Claudio Buffara 
> escreveu:
>
>> Um dos atrativos da matemática (pelo menos pra mim) é a existência de
>> problemas que estão em aberto há décadas (ou séculos) mas cujos enunciados
>> podem ser facilmente compreendidos por alunos de ensino fundamental. Três
>> exemplos são a conjectura de Goldbach, a conjectura de que existe uma
>> infinidade de primos gêmeos, e a conjectura de Collatz.
>>
>> É interessante que existem 3 problemas elementares cujos enunciados são
>> parecidos com os das conjecturas acima:
>> 1) Prove que todo número natural maior do que 11 pode ser escrito como a
>> soma de dois números compostos;
>> 2) Encontre todos os “primos trigêmeos” (trios de números primos que
>> diferem por 2, tais como 3, 5 e 7);
>> 3) O primeiro termo de uma sequência é 10. Cada termo seguinte é igual à
>> metade do termo anterior, se este for par, ou 7 unidades maior do que o
>> termo anterior, se este for ímpar. Qual o 2018º termo da sequência?
>>
>> Como vocês podem verificar, os três problemas são fáceis, ainda que, pra
>> resolvê-los, sejam necessários um mínimo de raciocínio e alguma
>> experimentação.
>>
>> Mas o que eu quero saber é se um aluno normal de 7o ou 8o ano (de 12 a 14
>> anos de idade, em média) seria capaz de resolver tais problemas.
>> O que vocês acham?
>>
>> E será que um aluno de 6o ano (11-12 anos) seria capaz de explicar porque
>> a soma de dois números primos consecutivos não pode ser igual ao dobro de
>> um número primo?
>>
>> OBS: Todos estes problemas envolvem apenas conceitos que são vistos antes
>> do 6o ano: operações com números naturais e números pares, ímpares, primos
>> e compostos.
>>
>> []s,
>> Claudio.
>>
>>
>> --
>> Esta mensagem foi verificada pelo sistema de antivírus e
>> acredita-se estar livre de perigo.
>
>
>
> --
> Esta mensagem foi verificada pelo sistema de antivírus e
> acredita-se estar livre de perigo.

-- 
Esta mensagem foi verificada pelo sistema de antiv�rus e
 acredita-se estar livre de perigo.



[obm-l] Re: [obm-l] problemas fáceis pro ensino fundamental

2018-08-01 Por tôpico Arthur Vieira
Por primos consecutivos você quer dizer primos que tem diferença de duas
unidades?

Em 1 de agosto de 2018 12:30, Claudio Buffara 
escreveu:

> Um dos atrativos da matemática (pelo menos pra mim) é a existência de
> problemas que estão em aberto há décadas (ou séculos) mas cujos enunciados
> podem ser facilmente compreendidos por alunos de ensino fundamental. Três
> exemplos são a conjectura de Goldbach, a conjectura de que existe uma
> infinidade de primos gêmeos, e a conjectura de Collatz.
>
> É interessante que existem 3 problemas elementares cujos enunciados são
> parecidos com os das conjecturas acima:
> 1) Prove que todo número natural maior do que 11 pode ser escrito como a
> soma de dois números compostos;
> 2) Encontre todos os “primos trigêmeos” (trios de números primos que
> diferem por 2, tais como 3, 5 e 7);
> 3) O primeiro termo de uma sequência é 10. Cada termo seguinte é igual à
> metade do termo anterior, se este for par, ou 7 unidades maior do que o
> termo anterior, se este for ímpar. Qual o 2018º termo da sequência?
>
> Como vocês podem verificar, os três problemas são fáceis, ainda que, pra
> resolvê-los, sejam necessários um mínimo de raciocínio e alguma
> experimentação.
>
> Mas o que eu quero saber é se um aluno normal de 7o ou 8o ano (de 12 a 14
> anos de idade, em média) seria capaz de resolver tais problemas.
> O que vocês acham?
>
> E será que um aluno de 6o ano (11-12 anos) seria capaz de explicar porque
> a soma de dois números primos consecutivos não pode ser igual ao dobro de
> um número primo?
>
> OBS: Todos estes problemas envolvem apenas conceitos que são vistos antes
> do 6o ano: operações com números naturais e números pares, ímpares, primos
> e compostos.
>
> []s,
> Claudio.
>
>
> --
> Esta mensagem foi verificada pelo sistema de antivírus e
> acredita-se estar livre de perigo.

-- 
Esta mensagem foi verificada pelo sistema de antiv�rus e
 acredita-se estar livre de perigo.



Re: [obm-l] Problemas selecionados

2018-05-18 Por tôpico Daniel Quevedo
Sim, olhei rápido não percebi b/a^2 que tem que ter um algarismo. Está de
fato correta a solução

Em sex, 18 de mai de 2018 às 19:53, Pedro José 
escreveu:

> Boa noite!
> Daniel,
> observe com calma a solução do colega. Ele não considerou a como um
> algarismo. Note que a solução apresentada por ele foi para a = 143.
> Acontecerá novamente para a=142857143 É mais uma infininixade de vezes.
> Mas sempre b/a^2=7 e portanto, único.
> Saudações,
> PJMS
>
> Em Sex, 18 de mai de 2018 19:34, Daniel Quevedo 
> escreveu:
>
>>
>> Sim, agora olhei com mais calma e entendi. Está correto
>>
>> Em sex, 18 de mai de 2018 às 19:22, Otávio Araújo <
>> otavio17.ara...@gmail.com> escreveu:
>>
>>> E eu não usei a como um número natural qualquer?
>>>
>>> Em sex, 18 de mai de 2018 19:02, Daniel Quevedo 
>>> escreveu:
>>>
 A minha dúvida foi pq pensei em algo mais Geral, não interpretei a como
 algarismo, mas colo número qualquer. Aí teríamos números como 1001 q é
 divisível por 11. Mas assim acho q o problema não fecha.
 Mas me parece q essa é a resolução correta.
 Obrigado

 Em sex, 18 de mai de 2018 às 18:39, Otávio Araújo <
 otavio17.ara...@gmail.com> escreveu:

> * e é o único valor possível.
>
> Esqueci o "e" kkl
>
> Em sex, 18 de mai de 2018 18:24, Otávio Araújo <
> otavio17.ara...@gmail.com> escreveu:
>
>> Faça o seguinte: seja n o número de algarismos de a. Então b =
>> (10^n+1)*a.   ( * denota multiplicação)
>> então a^2 divide b -> a divide 10^n+1 e tem n algarismos ->  10^n-1<=
>> a <10^n.
>> Dai (10^n+1)/a só pode ser um dos números: 2,3,4,5,6,7,8,9. Usando os
>> critérios de divisibilidade, já podemos descartar 2,3,4,5,6,8,9 pois 
>> estes
>> nunva dividiram 10^n+1, qualquer que seja n natural. Assim só sobra o 7. 
>> E
>> de fato é possível, basta fazer n=3, pois 1001 é divisivel por 7. Nesse
>> caso a=1001/7=143, e b=143143 =143x1001=(143^2)x7. Assim b/a^2 seria 7
>> nesse caso é o único valor possível.
>>
>>
>> Em sex, 18 de mai de 2018 18:01, Daniel Quevedo 
>> escreveu:
>>
>>> Dado um inteiro positivo a > 1, escrito em notação decimal, seja b o
>>> número obtido ao cooocarmos lado a lado duas cópias de a, isto é, b = 
>>> aa.
>>> Sabendo que b é múltiplo de a^2, o número de valores possíveis de b/a^2 
>>> é:
>>>
>>> A) 0
>>> B) 1
>>> C) 2
>>> D) 3
>>> E) mais de 3
>>>
>>>
>>> R: b
>>> --
>>> Fiscal: Daniel Quevedo
>>>
>>> --
>>> Esta mensagem foi verificada pelo sistema de antivírus e
>>> acredita-se estar livre de perigo.
>>
>>
> --
> Esta mensagem foi verificada pelo sistema de antivírus e
> acredita-se estar livre de perigo.

 --
 Fiscal: Daniel Quevedo

 --
 Esta mensagem foi verificada pelo sistema de antivírus e
 acredita-se estar livre de perigo.
>>>
>>>
>>> --
>>> Esta mensagem foi verificada pelo sistema de antivírus e
>>> acredita-se estar livre de perigo.
>>
>> --
>> Fiscal: Daniel Quevedo
>>
>> --
>> Esta mensagem foi verificada pelo sistema de antivírus e
>> acredita-se estar livre de perigo.
>
>
> --
> Esta mensagem foi verificada pelo sistema de antivírus e
> acredita-se estar livre de perigo.

-- 
Fiscal: Daniel Quevedo

-- 
Esta mensagem foi verificada pelo sistema de antiv�rus e
 acredita-se estar livre de perigo.



Re: [obm-l] Problemas selecionados

2018-05-18 Por tôpico Pedro José
Boa noite!
Daniel,
observe com calma a solução do colega. Ele não considerou a como um
algarismo. Note que a solução apresentada por ele foi para a = 143.
Acontecerá novamente para a=142857143 É mais uma infininixade de vezes. Mas
sempre b/a^2=7 e portanto, único.
Saudações,
PJMS

Em Sex, 18 de mai de 2018 19:34, Daniel Quevedo 
escreveu:

>
> Sim, agora olhei com mais calma e entendi. Está correto
>
> Em sex, 18 de mai de 2018 às 19:22, Otávio Araújo <
> otavio17.ara...@gmail.com> escreveu:
>
>> E eu não usei a como um número natural qualquer?
>>
>> Em sex, 18 de mai de 2018 19:02, Daniel Quevedo 
>> escreveu:
>>
>>> A minha dúvida foi pq pensei em algo mais Geral, não interpretei a como
>>> algarismo, mas colo número qualquer. Aí teríamos números como 1001 q é
>>> divisível por 11. Mas assim acho q o problema não fecha.
>>> Mas me parece q essa é a resolução correta.
>>> Obrigado
>>>
>>> Em sex, 18 de mai de 2018 às 18:39, Otávio Araújo <
>>> otavio17.ara...@gmail.com> escreveu:
>>>
 * e é o único valor possível.

 Esqueci o "e" kkl

 Em sex, 18 de mai de 2018 18:24, Otávio Araújo <
 otavio17.ara...@gmail.com> escreveu:

> Faça o seguinte: seja n o número de algarismos de a. Então b =
> (10^n+1)*a.   ( * denota multiplicação)
> então a^2 divide b -> a divide 10^n+1 e tem n algarismos ->  10^n-1<=
> a <10^n.
> Dai (10^n+1)/a só pode ser um dos números: 2,3,4,5,6,7,8,9. Usando os
> critérios de divisibilidade, já podemos descartar 2,3,4,5,6,8,9 pois estes
> nunva dividiram 10^n+1, qualquer que seja n natural. Assim só sobra o 7. E
> de fato é possível, basta fazer n=3, pois 1001 é divisivel por 7. Nesse
> caso a=1001/7=143, e b=143143 =143x1001=(143^2)x7. Assim b/a^2 seria 7
> nesse caso é o único valor possível.
>
>
> Em sex, 18 de mai de 2018 18:01, Daniel Quevedo 
> escreveu:
>
>> Dado um inteiro positivo a > 1, escrito em notação decimal, seja b o
>> número obtido ao cooocarmos lado a lado duas cópias de a, isto é, b = aa.
>> Sabendo que b é múltiplo de a^2, o número de valores possíveis de b/a^2 
>> é:
>>
>> A) 0
>> B) 1
>> C) 2
>> D) 3
>> E) mais de 3
>>
>>
>> R: b
>> --
>> Fiscal: Daniel Quevedo
>>
>> --
>> Esta mensagem foi verificada pelo sistema de antivírus e
>> acredita-se estar livre de perigo.
>
>
 --
 Esta mensagem foi verificada pelo sistema de antivírus e
 acredita-se estar livre de perigo.
>>>
>>> --
>>> Fiscal: Daniel Quevedo
>>>
>>> --
>>> Esta mensagem foi verificada pelo sistema de antivírus e
>>> acredita-se estar livre de perigo.
>>
>>
>> --
>> Esta mensagem foi verificada pelo sistema de antivírus e
>> acredita-se estar livre de perigo.
>
> --
> Fiscal: Daniel Quevedo
>
> --
> Esta mensagem foi verificada pelo sistema de antivírus e
> acredita-se estar livre de perigo.

-- 
Esta mensagem foi verificada pelo sistema de antiv�rus e
 acredita-se estar livre de perigo.



Re: [obm-l] Problemas selecionados

2018-05-18 Por tôpico Otávio Araújo
A resposta permanece somente 7, na verdade já tinha noção do que vc falou.
De fato, se a=(10^(6n+3)+1)/7, b será
(10^(6n+3)+1)^2/7 e a^2 será (10^(6n+3)+1)^2/7^2,  e a razão b/a^2
continuará 7

Em sex, 18 de mai de 2018 19:26, Pedro José  escreveu:

> Boa noite!
> Otávio,
> sua solução foi bela. Mas 10^n mod7 é periódica com período mínimo =6, já
> que 10^n=1 mod7.
> Portanto o que aconteceu para n=3, acontecerá também para n = 9, 15, 21,
> 27...
> Creio que haja uma infinidade de respostas.
> Saudações,
> PJMS
>
>
> Em Sex, 18 de mai de 2018 19:02, Daniel Quevedo 
> escreveu:
>
>> A minha dúvida foi pq pensei em algo mais Geral, não interpretei a como
>> algarismo, mas colo número qualquer. Aí teríamos números como 1001 q é
>> divisível por 11. Mas assim acho q o problema não fecha.
>> Mas me parece q essa é a resolução correta.
>> Obrigado
>>
>> Em sex, 18 de mai de 2018 às 18:39, Otávio Araújo <
>> otavio17.ara...@gmail.com> escreveu:
>>
>>> * e é o único valor possível.
>>>
>>> Esqueci o "e" kkl
>>>
>>> Em sex, 18 de mai de 2018 18:24, Otávio Araújo <
>>> otavio17.ara...@gmail.com> escreveu:
>>>
 Faça o seguinte: seja n o número de algarismos de a. Então b =
 (10^n+1)*a.   ( * denota multiplicação)
 então a^2 divide b -> a divide 10^n+1 e tem n algarismos ->  10^n-1<= a
 <10^n.
 Dai (10^n+1)/a só pode ser um dos números: 2,3,4,5,6,7,8,9. Usando os
 critérios de divisibilidade, já podemos descartar 2,3,4,5,6,8,9 pois estes
 nunva dividiram 10^n+1, qualquer que seja n natural. Assim só sobra o 7. E
 de fato é possível, basta fazer n=3, pois 1001 é divisivel por 7. Nesse
 caso a=1001/7=143, e b=143143 =143x1001=(143^2)x7. Assim b/a^2 seria 7
 nesse caso é o único valor possível.


 Em sex, 18 de mai de 2018 18:01, Daniel Quevedo 
 escreveu:

> Dado um inteiro positivo a > 1, escrito em notação decimal, seja b o
> número obtido ao cooocarmos lado a lado duas cópias de a, isto é, b = aa.
> Sabendo que b é múltiplo de a^2, o número de valores possíveis de b/a^2 é:
>
> A) 0
> B) 1
> C) 2
> D) 3
> E) mais de 3
>
>
> R: b
> --
> Fiscal: Daniel Quevedo
>
> --
> Esta mensagem foi verificada pelo sistema de antivírus e
> acredita-se estar livre de perigo.


>>> --
>>> Esta mensagem foi verificada pelo sistema de antivírus e
>>> acredita-se estar livre de perigo.
>>
>> --
>> Fiscal: Daniel Quevedo
>>
>> --
>> Esta mensagem foi verificada pelo sistema de antivírus e
>> acredita-se estar livre de perigo.
>
>
> --
> Esta mensagem foi verificada pelo sistema de antivírus e
> acredita-se estar livre de perigo.

-- 
Esta mensagem foi verificada pelo sistema de antiv�rus e
 acredita-se estar livre de perigo.



Re: [obm-l] Problemas selecionados

2018-05-18 Por tôpico Otávio Araújo
De boas

Em sex, 18 de mai de 2018 19:33, Pedro José  escreveu:

> Desculpe-me, o problema se relaciona ao fator do múltiplo e não às
> ocorrências de a. Portanto, só há uma solução.
> Correto.
>
> Em Sex, 18 de mai de 2018 19:16, Pedro José 
> escreveu:
>
>> Boa noite!
>> Otávio,
>> sua solução foi bela. Mas 10^n mod7 é periódica com período mínimo =6, já
>> que 10^n=1 mod7.
>> Portanto o que aconteceu para n=3, acontecerá também para n = 9, 15, 21,
>> 27...
>> Creio que haja uma infinidade de respostas.
>> Saudações,
>> PJMS
>>
>>
>> Em Sex, 18 de mai de 2018 19:02, Daniel Quevedo 
>> escreveu:
>>
>>> A minha dúvida foi pq pensei em algo mais Geral, não interpretei a como
>>> algarismo, mas colo número qualquer. Aí teríamos números como 1001 q é
>>> divisível por 11. Mas assim acho q o problema não fecha.
>>> Mas me parece q essa é a resolução correta.
>>> Obrigado
>>>
>>> Em sex, 18 de mai de 2018 às 18:39, Otávio Araújo <
>>> otavio17.ara...@gmail.com> escreveu:
>>>
 * e é o único valor possível.

 Esqueci o "e" kkl

 Em sex, 18 de mai de 2018 18:24, Otávio Araújo <
 otavio17.ara...@gmail.com> escreveu:

> Faça o seguinte: seja n o número de algarismos de a. Então b =
> (10^n+1)*a.   ( * denota multiplicação)
> então a^2 divide b -> a divide 10^n+1 e tem n algarismos ->  10^n-1<=
> a <10^n.
> Dai (10^n+1)/a só pode ser um dos números: 2,3,4,5,6,7,8,9. Usando os
> critérios de divisibilidade, já podemos descartar 2,3,4,5,6,8,9 pois estes
> nunva dividiram 10^n+1, qualquer que seja n natural. Assim só sobra o 7. E
> de fato é possível, basta fazer n=3, pois 1001 é divisivel por 7. Nesse
> caso a=1001/7=143, e b=143143 =143x1001=(143^2)x7. Assim b/a^2 seria 7
> nesse caso é o único valor possível.
>
>
> Em sex, 18 de mai de 2018 18:01, Daniel Quevedo 
> escreveu:
>
>> Dado um inteiro positivo a > 1, escrito em notação decimal, seja b o
>> número obtido ao cooocarmos lado a lado duas cópias de a, isto é, b = aa.
>> Sabendo que b é múltiplo de a^2, o número de valores possíveis de b/a^2 
>> é:
>>
>> A) 0
>> B) 1
>> C) 2
>> D) 3
>> E) mais de 3
>>
>>
>> R: b
>> --
>> Fiscal: Daniel Quevedo
>>
>> --
>> Esta mensagem foi verificada pelo sistema de antivírus e
>> acredita-se estar livre de perigo.
>
>
 --
 Esta mensagem foi verificada pelo sistema de antivírus e
 acredita-se estar livre de perigo.
>>>
>>> --
>>> Fiscal: Daniel Quevedo
>>>
>>> --
>>> Esta mensagem foi verificada pelo sistema de antivírus e
>>> acredita-se estar livre de perigo.
>>
>>
> --
> Esta mensagem foi verificada pelo sistema de antivírus e
> acredita-se estar livre de perigo.

-- 
Esta mensagem foi verificada pelo sistema de antiv�rus e
 acredita-se estar livre de perigo.



Re: [obm-l] Problemas selecionados

2018-05-18 Por tôpico Pedro José
Boa noite!
Não havia prestado atenção no enunciado e julgará que fosse a quantidade de
soluções a e não do quociente b/a^2. Está correto.
É que para a há uma infinidade de soluções. Porém b/a^2 é constante.
Saudações,
PJMS


Em Sex, 18 de mai de 2018 19:22, Otávio Araújo 
escreveu:

> E eu não usei a como um número natural qualquer?
>
> Em sex, 18 de mai de 2018 19:02, Daniel Quevedo 
> escreveu:
>
>> A minha dúvida foi pq pensei em algo mais Geral, não interpretei a como
>> algarismo, mas colo número qualquer. Aí teríamos números como 1001 q é
>> divisível por 11. Mas assim acho q o problema não fecha.
>> Mas me parece q essa é a resolução correta.
>> Obrigado
>>
>> Em sex, 18 de mai de 2018 às 18:39, Otávio Araújo <
>> otavio17.ara...@gmail.com> escreveu:
>>
>>> * e é o único valor possível.
>>>
>>> Esqueci o "e" kkl
>>>
>>> Em sex, 18 de mai de 2018 18:24, Otávio Araújo <
>>> otavio17.ara...@gmail.com> escreveu:
>>>
 Faça o seguinte: seja n o número de algarismos de a. Então b =
 (10^n+1)*a.   ( * denota multiplicação)
 então a^2 divide b -> a divide 10^n+1 e tem n algarismos ->  10^n-1<= a
 <10^n.
 Dai (10^n+1)/a só pode ser um dos números: 2,3,4,5,6,7,8,9. Usando os
 critérios de divisibilidade, já podemos descartar 2,3,4,5,6,8,9 pois estes
 nunva dividiram 10^n+1, qualquer que seja n natural. Assim só sobra o 7. E
 de fato é possível, basta fazer n=3, pois 1001 é divisivel por 7. Nesse
 caso a=1001/7=143, e b=143143 =143x1001=(143^2)x7. Assim b/a^2 seria 7
 nesse caso é o único valor possível.


 Em sex, 18 de mai de 2018 18:01, Daniel Quevedo 
 escreveu:

> Dado um inteiro positivo a > 1, escrito em notação decimal, seja b o
> número obtido ao cooocarmos lado a lado duas cópias de a, isto é, b = aa.
> Sabendo que b é múltiplo de a^2, o número de valores possíveis de b/a^2 é:
>
> A) 0
> B) 1
> C) 2
> D) 3
> E) mais de 3
>
>
> R: b
> --
> Fiscal: Daniel Quevedo
>
> --
> Esta mensagem foi verificada pelo sistema de antivírus e
> acredita-se estar livre de perigo.


>>> --
>>> Esta mensagem foi verificada pelo sistema de antivírus e
>>> acredita-se estar livre de perigo.
>>
>> --
>> Fiscal: Daniel Quevedo
>>
>> --
>> Esta mensagem foi verificada pelo sistema de antivírus e
>> acredita-se estar livre de perigo.
>
>
> --
> Esta mensagem foi verificada pelo sistema de antivírus e
> acredita-se estar livre de perigo.

-- 
Esta mensagem foi verificada pelo sistema de antiv�rus e
 acredita-se estar livre de perigo.



Re: [obm-l] Problemas selecionados

2018-05-18 Por tôpico Daniel Quevedo
Sim, agora olhei com mais calma e entendi. Está correto

Em sex, 18 de mai de 2018 às 19:22, Otávio Araújo 
escreveu:

> E eu não usei a como um número natural qualquer?
>
> Em sex, 18 de mai de 2018 19:02, Daniel Quevedo 
> escreveu:
>
>> A minha dúvida foi pq pensei em algo mais Geral, não interpretei a como
>> algarismo, mas colo número qualquer. Aí teríamos números como 1001 q é
>> divisível por 11. Mas assim acho q o problema não fecha.
>> Mas me parece q essa é a resolução correta.
>> Obrigado
>>
>> Em sex, 18 de mai de 2018 às 18:39, Otávio Araújo <
>> otavio17.ara...@gmail.com> escreveu:
>>
>>> * e é o único valor possível.
>>>
>>> Esqueci o "e" kkl
>>>
>>> Em sex, 18 de mai de 2018 18:24, Otávio Araújo <
>>> otavio17.ara...@gmail.com> escreveu:
>>>
 Faça o seguinte: seja n o número de algarismos de a. Então b =
 (10^n+1)*a.   ( * denota multiplicação)
 então a^2 divide b -> a divide 10^n+1 e tem n algarismos ->  10^n-1<= a
 <10^n.
 Dai (10^n+1)/a só pode ser um dos números: 2,3,4,5,6,7,8,9. Usando os
 critérios de divisibilidade, já podemos descartar 2,3,4,5,6,8,9 pois estes
 nunva dividiram 10^n+1, qualquer que seja n natural. Assim só sobra o 7. E
 de fato é possível, basta fazer n=3, pois 1001 é divisivel por 7. Nesse
 caso a=1001/7=143, e b=143143 =143x1001=(143^2)x7. Assim b/a^2 seria 7
 nesse caso é o único valor possível.


 Em sex, 18 de mai de 2018 18:01, Daniel Quevedo 
 escreveu:

> Dado um inteiro positivo a > 1, escrito em notação decimal, seja b o
> número obtido ao cooocarmos lado a lado duas cópias de a, isto é, b = aa.
> Sabendo que b é múltiplo de a^2, o número de valores possíveis de b/a^2 é:
>
> A) 0
> B) 1
> C) 2
> D) 3
> E) mais de 3
>
>
> R: b
> --
> Fiscal: Daniel Quevedo
>
> --
> Esta mensagem foi verificada pelo sistema de antivírus e
> acredita-se estar livre de perigo.


>>> --
>>> Esta mensagem foi verificada pelo sistema de antivírus e
>>> acredita-se estar livre de perigo.
>>
>> --
>> Fiscal: Daniel Quevedo
>>
>> --
>> Esta mensagem foi verificada pelo sistema de antivírus e
>> acredita-se estar livre de perigo.
>
>
> --
> Esta mensagem foi verificada pelo sistema de antivírus e
> acredita-se estar livre de perigo.

-- 
Fiscal: Daniel Quevedo

-- 
Esta mensagem foi verificada pelo sistema de antiv�rus e
 acredita-se estar livre de perigo.



Re: [obm-l] Problemas selecionados

2018-05-18 Por tôpico Pedro José
Desculpe-me, o problema se relaciona ao fator do múltiplo e não às
ocorrências de a. Portanto, só há uma solução.
Correto.

Em Sex, 18 de mai de 2018 19:16, Pedro José  escreveu:

> Boa noite!
> Otávio,
> sua solução foi bela. Mas 10^n mod7 é periódica com período mínimo =6, já
> que 10^n=1 mod7.
> Portanto o que aconteceu para n=3, acontecerá também para n = 9, 15, 21,
> 27...
> Creio que haja uma infinidade de respostas.
> Saudações,
> PJMS
>
>
> Em Sex, 18 de mai de 2018 19:02, Daniel Quevedo 
> escreveu:
>
>> A minha dúvida foi pq pensei em algo mais Geral, não interpretei a como
>> algarismo, mas colo número qualquer. Aí teríamos números como 1001 q é
>> divisível por 11. Mas assim acho q o problema não fecha.
>> Mas me parece q essa é a resolução correta.
>> Obrigado
>>
>> Em sex, 18 de mai de 2018 às 18:39, Otávio Araújo <
>> otavio17.ara...@gmail.com> escreveu:
>>
>>> * e é o único valor possível.
>>>
>>> Esqueci o "e" kkl
>>>
>>> Em sex, 18 de mai de 2018 18:24, Otávio Araújo <
>>> otavio17.ara...@gmail.com> escreveu:
>>>
 Faça o seguinte: seja n o número de algarismos de a. Então b =
 (10^n+1)*a.   ( * denota multiplicação)
 então a^2 divide b -> a divide 10^n+1 e tem n algarismos ->  10^n-1<= a
 <10^n.
 Dai (10^n+1)/a só pode ser um dos números: 2,3,4,5,6,7,8,9. Usando os
 critérios de divisibilidade, já podemos descartar 2,3,4,5,6,8,9 pois estes
 nunva dividiram 10^n+1, qualquer que seja n natural. Assim só sobra o 7. E
 de fato é possível, basta fazer n=3, pois 1001 é divisivel por 7. Nesse
 caso a=1001/7=143, e b=143143 =143x1001=(143^2)x7. Assim b/a^2 seria 7
 nesse caso é o único valor possível.


 Em sex, 18 de mai de 2018 18:01, Daniel Quevedo 
 escreveu:

> Dado um inteiro positivo a > 1, escrito em notação decimal, seja b o
> número obtido ao cooocarmos lado a lado duas cópias de a, isto é, b = aa.
> Sabendo que b é múltiplo de a^2, o número de valores possíveis de b/a^2 é:
>
> A) 0
> B) 1
> C) 2
> D) 3
> E) mais de 3
>
>
> R: b
> --
> Fiscal: Daniel Quevedo
>
> --
> Esta mensagem foi verificada pelo sistema de antivírus e
> acredita-se estar livre de perigo.


>>> --
>>> Esta mensagem foi verificada pelo sistema de antivírus e
>>> acredita-se estar livre de perigo.
>>
>> --
>> Fiscal: Daniel Quevedo
>>
>> --
>> Esta mensagem foi verificada pelo sistema de antivírus e
>> acredita-se estar livre de perigo.
>
>

-- 
Esta mensagem foi verificada pelo sistema de antiv�rus e
 acredita-se estar livre de perigo.



Re: [obm-l] Problemas selecionados

2018-05-18 Por tôpico Pedro José
Boa noite!
Otávio,
sua solução foi bela. Mas 10^n mod7 é periódica com período mínimo =6, já
que 10^n=1 mod7.
Portanto o que aconteceu para n=3, acontecerá também para n = 9, 15, 21,
27...
Creio que haja uma infinidade de respostas.
Saudações,
PJMS


Em Sex, 18 de mai de 2018 19:02, Daniel Quevedo 
escreveu:

> A minha dúvida foi pq pensei em algo mais Geral, não interpretei a como
> algarismo, mas colo número qualquer. Aí teríamos números como 1001 q é
> divisível por 11. Mas assim acho q o problema não fecha.
> Mas me parece q essa é a resolução correta.
> Obrigado
>
> Em sex, 18 de mai de 2018 às 18:39, Otávio Araújo <
> otavio17.ara...@gmail.com> escreveu:
>
>> * e é o único valor possível.
>>
>> Esqueci o "e" kkl
>>
>> Em sex, 18 de mai de 2018 18:24, Otávio Araújo 
>> escreveu:
>>
>>> Faça o seguinte: seja n o número de algarismos de a. Então b =
>>> (10^n+1)*a.   ( * denota multiplicação)
>>> então a^2 divide b -> a divide 10^n+1 e tem n algarismos ->  10^n-1<= a
>>> <10^n.
>>> Dai (10^n+1)/a só pode ser um dos números: 2,3,4,5,6,7,8,9. Usando os
>>> critérios de divisibilidade, já podemos descartar 2,3,4,5,6,8,9 pois estes
>>> nunva dividiram 10^n+1, qualquer que seja n natural. Assim só sobra o 7. E
>>> de fato é possível, basta fazer n=3, pois 1001 é divisivel por 7. Nesse
>>> caso a=1001/7=143, e b=143143 =143x1001=(143^2)x7. Assim b/a^2 seria 7
>>> nesse caso é o único valor possível.
>>>
>>>
>>> Em sex, 18 de mai de 2018 18:01, Daniel Quevedo 
>>> escreveu:
>>>
 Dado um inteiro positivo a > 1, escrito em notação decimal, seja b o
 número obtido ao cooocarmos lado a lado duas cópias de a, isto é, b = aa.
 Sabendo que b é múltiplo de a^2, o número de valores possíveis de b/a^2 é:

 A) 0
 B) 1
 C) 2
 D) 3
 E) mais de 3


 R: b
 --
 Fiscal: Daniel Quevedo

 --
 Esta mensagem foi verificada pelo sistema de antivírus e
 acredita-se estar livre de perigo.
>>>
>>>
>> --
>> Esta mensagem foi verificada pelo sistema de antivírus e
>> acredita-se estar livre de perigo.
>
> --
> Fiscal: Daniel Quevedo
>
> --
> Esta mensagem foi verificada pelo sistema de antivírus e
> acredita-se estar livre de perigo.

-- 
Esta mensagem foi verificada pelo sistema de antiv�rus e
 acredita-se estar livre de perigo.



Re: [obm-l] Problemas selecionados

2018-05-18 Por tôpico Otávio Araújo
De nada

Em sex, 18 de mai de 2018 19:02, Daniel Quevedo 
escreveu:

> A minha dúvida foi pq pensei em algo mais Geral, não interpretei a como
> algarismo, mas colo número qualquer. Aí teríamos números como 1001 q é
> divisível por 11. Mas assim acho q o problema não fecha.
> Mas me parece q essa é a resolução correta.
> Obrigado
>
> Em sex, 18 de mai de 2018 às 18:39, Otávio Araújo <
> otavio17.ara...@gmail.com> escreveu:
>
>> * e é o único valor possível.
>>
>> Esqueci o "e" kkl
>>
>> Em sex, 18 de mai de 2018 18:24, Otávio Araújo 
>> escreveu:
>>
>>> Faça o seguinte: seja n o número de algarismos de a. Então b =
>>> (10^n+1)*a.   ( * denota multiplicação)
>>> então a^2 divide b -> a divide 10^n+1 e tem n algarismos ->  10^n-1<= a
>>> <10^n.
>>> Dai (10^n+1)/a só pode ser um dos números: 2,3,4,5,6,7,8,9. Usando os
>>> critérios de divisibilidade, já podemos descartar 2,3,4,5,6,8,9 pois estes
>>> nunva dividiram 10^n+1, qualquer que seja n natural. Assim só sobra o 7. E
>>> de fato é possível, basta fazer n=3, pois 1001 é divisivel por 7. Nesse
>>> caso a=1001/7=143, e b=143143 =143x1001=(143^2)x7. Assim b/a^2 seria 7
>>> nesse caso é o único valor possível.
>>>
>>>
>>> Em sex, 18 de mai de 2018 18:01, Daniel Quevedo 
>>> escreveu:
>>>
 Dado um inteiro positivo a > 1, escrito em notação decimal, seja b o
 número obtido ao cooocarmos lado a lado duas cópias de a, isto é, b = aa.
 Sabendo que b é múltiplo de a^2, o número de valores possíveis de b/a^2 é:

 A) 0
 B) 1
 C) 2
 D) 3
 E) mais de 3


 R: b
 --
 Fiscal: Daniel Quevedo

 --
 Esta mensagem foi verificada pelo sistema de antivírus e
 acredita-se estar livre de perigo.
>>>
>>>
>> --
>> Esta mensagem foi verificada pelo sistema de antivírus e
>> acredita-se estar livre de perigo.
>
> --
> Fiscal: Daniel Quevedo
>
> --
> Esta mensagem foi verificada pelo sistema de antivírus e
> acredita-se estar livre de perigo.

-- 
Esta mensagem foi verificada pelo sistema de antiv�rus e
 acredita-se estar livre de perigo.



Re: [obm-l] Problemas selecionados

2018-05-18 Por tôpico Otávio Araújo
E eu não usei a como um número natural qualquer?

Em sex, 18 de mai de 2018 19:02, Daniel Quevedo 
escreveu:

> A minha dúvida foi pq pensei em algo mais Geral, não interpretei a como
> algarismo, mas colo número qualquer. Aí teríamos números como 1001 q é
> divisível por 11. Mas assim acho q o problema não fecha.
> Mas me parece q essa é a resolução correta.
> Obrigado
>
> Em sex, 18 de mai de 2018 às 18:39, Otávio Araújo <
> otavio17.ara...@gmail.com> escreveu:
>
>> * e é o único valor possível.
>>
>> Esqueci o "e" kkl
>>
>> Em sex, 18 de mai de 2018 18:24, Otávio Araújo 
>> escreveu:
>>
>>> Faça o seguinte: seja n o número de algarismos de a. Então b =
>>> (10^n+1)*a.   ( * denota multiplicação)
>>> então a^2 divide b -> a divide 10^n+1 e tem n algarismos ->  10^n-1<= a
>>> <10^n.
>>> Dai (10^n+1)/a só pode ser um dos números: 2,3,4,5,6,7,8,9. Usando os
>>> critérios de divisibilidade, já podemos descartar 2,3,4,5,6,8,9 pois estes
>>> nunva dividiram 10^n+1, qualquer que seja n natural. Assim só sobra o 7. E
>>> de fato é possível, basta fazer n=3, pois 1001 é divisivel por 7. Nesse
>>> caso a=1001/7=143, e b=143143 =143x1001=(143^2)x7. Assim b/a^2 seria 7
>>> nesse caso é o único valor possível.
>>>
>>>
>>> Em sex, 18 de mai de 2018 18:01, Daniel Quevedo 
>>> escreveu:
>>>
 Dado um inteiro positivo a > 1, escrito em notação decimal, seja b o
 número obtido ao cooocarmos lado a lado duas cópias de a, isto é, b = aa.
 Sabendo que b é múltiplo de a^2, o número de valores possíveis de b/a^2 é:

 A) 0
 B) 1
 C) 2
 D) 3
 E) mais de 3


 R: b
 --
 Fiscal: Daniel Quevedo

 --
 Esta mensagem foi verificada pelo sistema de antivírus e
 acredita-se estar livre de perigo.
>>>
>>>
>> --
>> Esta mensagem foi verificada pelo sistema de antivírus e
>> acredita-se estar livre de perigo.
>
> --
> Fiscal: Daniel Quevedo
>
> --
> Esta mensagem foi verificada pelo sistema de antivírus e
> acredita-se estar livre de perigo.

-- 
Esta mensagem foi verificada pelo sistema de antiv�rus e
 acredita-se estar livre de perigo.



Re: [obm-l] Problemas selecionados

2018-05-18 Por tôpico Daniel Quevedo
A minha dúvida foi pq pensei em algo mais Geral, não interpretei a como
algarismo, mas colo número qualquer. Aí teríamos números como 1001 q é
divisível por 11. Mas assim acho q o problema não fecha.
Mas me parece q essa é a resolução correta.
Obrigado

Em sex, 18 de mai de 2018 às 18:39, Otávio Araújo 
escreveu:

> * e é o único valor possível.
>
> Esqueci o "e" kkl
>
> Em sex, 18 de mai de 2018 18:24, Otávio Araújo 
> escreveu:
>
>> Faça o seguinte: seja n o número de algarismos de a. Então b =
>> (10^n+1)*a.   ( * denota multiplicação)
>> então a^2 divide b -> a divide 10^n+1 e tem n algarismos ->  10^n-1<= a
>> <10^n.
>> Dai (10^n+1)/a só pode ser um dos números: 2,3,4,5,6,7,8,9. Usando os
>> critérios de divisibilidade, já podemos descartar 2,3,4,5,6,8,9 pois estes
>> nunva dividiram 10^n+1, qualquer que seja n natural. Assim só sobra o 7. E
>> de fato é possível, basta fazer n=3, pois 1001 é divisivel por 7. Nesse
>> caso a=1001/7=143, e b=143143 =143x1001=(143^2)x7. Assim b/a^2 seria 7
>> nesse caso é o único valor possível.
>>
>>
>> Em sex, 18 de mai de 2018 18:01, Daniel Quevedo 
>> escreveu:
>>
>>> Dado um inteiro positivo a > 1, escrito em notação decimal, seja b o
>>> número obtido ao cooocarmos lado a lado duas cópias de a, isto é, b = aa.
>>> Sabendo que b é múltiplo de a^2, o número de valores possíveis de b/a^2 é:
>>>
>>> A) 0
>>> B) 1
>>> C) 2
>>> D) 3
>>> E) mais de 3
>>>
>>>
>>> R: b
>>> --
>>> Fiscal: Daniel Quevedo
>>>
>>> --
>>> Esta mensagem foi verificada pelo sistema de antivírus e
>>> acredita-se estar livre de perigo.
>>
>>
> --
> Esta mensagem foi verificada pelo sistema de antivírus e
> acredita-se estar livre de perigo.

-- 
Fiscal: Daniel Quevedo

-- 
Esta mensagem foi verificada pelo sistema de antiv�rus e
 acredita-se estar livre de perigo.



Re: [obm-l] Problemas selecionados

2018-05-18 Por tôpico Otávio Araújo
* 10^(n-1)<=a<10^n


Esqueci dos parênteses tbm kkk

Em sex, 18 de mai de 2018 18:28, Otávio Araújo 
escreveu:

> * e é o único valor possível.
>
> Esqueci o "e" kkl
>
> Em sex, 18 de mai de 2018 18:24, Otávio Araújo 
> escreveu:
>
>> Faça o seguinte: seja n o número de algarismos de a. Então b =
>> (10^n+1)*a.   ( * denota multiplicação)
>> então a^2 divide b -> a divide 10^n+1 e tem n algarismos ->  10^n-1<= a
>> <10^n.
>> Dai (10^n+1)/a só pode ser um dos números: 2,3,4,5,6,7,8,9. Usando os
>> critérios de divisibilidade, já podemos descartar 2,3,4,5,6,8,9 pois estes
>> nunva dividiram 10^n+1, qualquer que seja n natural. Assim só sobra o 7. E
>> de fato é possível, basta fazer n=3, pois 1001 é divisivel por 7. Nesse
>> caso a=1001/7=143, e b=143143 =143x1001=(143^2)x7. Assim b/a^2 seria 7
>> nesse caso é o único valor possível.
>>
>>
>> Em sex, 18 de mai de 2018 18:01, Daniel Quevedo 
>> escreveu:
>>
>>> Dado um inteiro positivo a > 1, escrito em notação decimal, seja b o
>>> número obtido ao cooocarmos lado a lado duas cópias de a, isto é, b = aa.
>>> Sabendo que b é múltiplo de a^2, o número de valores possíveis de b/a^2 é:
>>>
>>> A) 0
>>> B) 1
>>> C) 2
>>> D) 3
>>> E) mais de 3
>>>
>>>
>>> R: b
>>> --
>>> Fiscal: Daniel Quevedo
>>>
>>> --
>>> Esta mensagem foi verificada pelo sistema de antivírus e
>>> acredita-se estar livre de perigo.
>>
>>

-- 
Esta mensagem foi verificada pelo sistema de antiv�rus e
 acredita-se estar livre de perigo.



Re: [obm-l] Problemas selecionados

2018-05-18 Por tôpico Otávio Araújo
* e é o único valor possível.

Esqueci o "e" kkl

Em sex, 18 de mai de 2018 18:24, Otávio Araújo 
escreveu:

> Faça o seguinte: seja n o número de algarismos de a. Então b =
> (10^n+1)*a.   ( * denota multiplicação)
> então a^2 divide b -> a divide 10^n+1 e tem n algarismos ->  10^n-1<= a
> <10^n.
> Dai (10^n+1)/a só pode ser um dos números: 2,3,4,5,6,7,8,9. Usando os
> critérios de divisibilidade, já podemos descartar 2,3,4,5,6,8,9 pois estes
> nunva dividiram 10^n+1, qualquer que seja n natural. Assim só sobra o 7. E
> de fato é possível, basta fazer n=3, pois 1001 é divisivel por 7. Nesse
> caso a=1001/7=143, e b=143143 =143x1001=(143^2)x7. Assim b/a^2 seria 7
> nesse caso é o único valor possível.
>
>
> Em sex, 18 de mai de 2018 18:01, Daniel Quevedo 
> escreveu:
>
>> Dado um inteiro positivo a > 1, escrito em notação decimal, seja b o
>> número obtido ao cooocarmos lado a lado duas cópias de a, isto é, b = aa.
>> Sabendo que b é múltiplo de a^2, o número de valores possíveis de b/a^2 é:
>>
>> A) 0
>> B) 1
>> C) 2
>> D) 3
>> E) mais de 3
>>
>>
>> R: b
>> --
>> Fiscal: Daniel Quevedo
>>
>> --
>> Esta mensagem foi verificada pelo sistema de antivírus e
>> acredita-se estar livre de perigo.
>
>

-- 
Esta mensagem foi verificada pelo sistema de antiv�rus e
 acredita-se estar livre de perigo.



Re: [obm-l] Problemas selecionados

2018-05-18 Por tôpico Otávio Araújo
Faça o seguinte: seja n o número de algarismos de a. Então b = (10^n+1)*a.
 ( * denota multiplicação)
então a^2 divide b -> a divide 10^n+1 e tem n algarismos ->  10^n-1<= a
<10^n.
Dai (10^n+1)/a só pode ser um dos números: 2,3,4,5,6,7,8,9. Usando os
critérios de divisibilidade, já podemos descartar 2,3,4,5,6,8,9 pois estes
nunva dividiram 10^n+1, qualquer que seja n natural. Assim só sobra o 7. E
de fato é possível, basta fazer n=3, pois 1001 é divisivel por 7. Nesse
caso a=1001/7=143, e b=143143 =143x1001=(143^2)x7. Assim b/a^2 seria 7
nesse caso é o único valor possível.


Em sex, 18 de mai de 2018 18:01, Daniel Quevedo 
escreveu:

> Dado um inteiro positivo a > 1, escrito em notação decimal, seja b o
> número obtido ao cooocarmos lado a lado duas cópias de a, isto é, b = aa.
> Sabendo que b é múltiplo de a^2, o número de valores possíveis de b/a^2 é:
>
> A) 0
> B) 1
> C) 2
> D) 3
> E) mais de 3
>
>
> R: b
> --
> Fiscal: Daniel Quevedo
>
> --
> Esta mensagem foi verificada pelo sistema de antivírus e
> acredita-se estar livre de perigo.

-- 
Esta mensagem foi verificada pelo sistema de antiv�rus e
 acredita-se estar livre de perigo.



Re: [obm-l] Re: [obm-l] Re: [obm-l] Re: [obm-l] Problemas interessantes de análise complexa

2018-03-27 Por tôpico Artur Costa Steiner
acho que vou comprar esse livro. Eu tenho Complex Made Simple, de David UlrichArtur Costa Steiner Em 27 de mar de 2018 15:52, Claudio Buffara  escreveu:

A rigidez à qual eu me referia me parece ter mais a ver com o fato de que uma função analítica, por também ser conforme, transforma um "quadrado infinitesimal" em outro "quadrado infinitesimal", enquanto que uma função que é apenas real-diferenciável (no sentido da análise no R^n, olhando C como R^2) transforma um tal quadrado num "paralelogramo infinitesimal", que em geral, nem retângulo é.Isso tá bem ilustrado no livro Visual Complex Analysis, que eu mencionei antes.

[]s,Claudio.2018-03-27 14:12 GMT-03:00 Bernardo Freitas Paulo da Costa :2018-03-27 13:36 GMT-03:00 Claudio Buffara :
> Os problemas 1, 3 e 4 me parecem ser consequências da "rigidez" que a
> diferenciabilidade complexa impõe às funções analíticas e que, pra mim, está
> longe de ser algo intuitivo.

É, a estrutura complexa é muito impressionante.  Parte da rigidez é
puramente algébrica (como abaixo), mas existem fenômenos para os quais
eu não encontro um análogo algébrico legal (como o próximo |f| <= |g|
...)

> Por exemplo, no problema 1, se g(z) = exp(z), então a conclusão decorre do
> teorema de Liouville.
> No caso geral, temos que lidar com os zeros de g e Liouville não se aplica
> (pelo menos não diretamente).
> No entanto, se g(z) = 0, então f(z) = 0 pela desigualdade |f(z)| <= |g(z)|.
> Será que essa desigualdade garante que os zeros de g(z) são singularidades
> removíveis de f(z)/g(z)?

Sim: a forma canônica multiplicativa de f e g em torno de um zero,
mais a desigualdade, dá que a ordem de f é pelo menos igual à de g.  O
fato de haver uma ordem *inteira* de anulação é o que eu chamo de
"algebrização".

Abraços,
--
Bernardo Freitas Paulo da Costa

--
Esta mensagem foi verificada pelo sistema de antivírus e
 acredita-se estar livre de perigo.


=
Instru�ões para entrar na lista, sair da lista e usar a lista em
http://www.mat.puc-rio.br/~obmlistas/obm-l.html
=

--
Esta mensagem foi verificada pelo sistema de antiv�rus e 
 acredita-se estar livre de perigo.

--
Esta mensagem foi verificada pelo sistema de antivírus e 
 acredita-se estar livre de perigo.




[obm-l] Re: [obm-l] Re: [obm-l] Problemas interessantes de análise complexa

2018-03-27 Por tôpico Artur Costa Steiner
Em Ter, 27 de mar de 2018 13:50, Claudio Buffara 
escreveu:

> Os problemas 1, 3 e 4 me parecem ser consequências da "rigidez" que a
> diferenciabilidade complexa impõe às funções analíticas e que, pra mim,
> está longe de ser algo intuitivo.
>
> Por exemplo, no problema 1, se g(z) = exp(z), então a conclusão decorre do
> teorema de Liouville.
> No caso geral, temos que lidar com os zeros de g e Liouville não se aplica
> (pelo menos não diretamente).
> No entanto, se g(z) = 0, então f(z) = 0 pela desigualdade |f(z)| <= |g(z)|.
> Será que essa desigualdade garante que os zeros de g(z) são singularidades
> removíveis de f(z)/g(z)?
>

Garante sjm, porque f/g é limitada (por 1) perto dos zeros de g. E isto
garante que os zeros sejam singularidades removíveis

>
> No problema 2, fazendo w = 1/z, obtemos o polinômio w^(n+1) + 2w - 1 e o
> problema passa a ser o de provar que este tem uma única raiz no interior de
> D(0,1).
> (também é preciso mostrar que P(z) não tem raízes com |z| = 1, mas isso é
> relativamente simples: se P(z) = 0 e |z| = 1, então z^n(z-2) = 1 ==> |z-2|
> = 1 ==> z = 1. Mas P(1) = -2 <> 0).
> O teorema de Rouché diz que, se f e g forem analíticas no fecho de D(0,1)
> e que se |g(w)| < |f(w)| para |w| = 1, então f e f+g têm o mesmo número de
> raízes no interior de D(0,1).
> Tomemos f(w) = 2w - 0,5  e  g(w) = w^(n+1) - 0,5.
> Então:
> f(w) tem uma única raiz (igual a 0,25) no interior de D(0,1);
> f(w) + g(w) = w^(n+1) + 2w - 1;
> para |w| = 1,
> |g(w)| = |w^(n+1) - 0,5| <= 1 + 0,5 = 1,5, com igualdade sss w for uma
> raiz (n+1)-ésima de -1,
> |f(w)| = |2w - 0,5| >= 2 - 0,5 = 1,5, com igualdade sss w = 1.
> Como 1 não é uma raiz (n+1)-ésima de -1, teremos sempre a desigualdade
> estrita |g(w)| < |f(w)| para |w| = 1.
> Assim, as condições do teorema de Rouché são satisfeitas e, portanto,
> w^(n+1) + 2w - 1 tem o mesmo número de raízes (a saber, 1) que 2w - 0,5 no
> interior de D(0,1).
>

OK!

E para determinar a integral em função do único zero em fora de D(0,1),
podemos invocar o teorema das raízes interiores e exteriores. Como edta
raiz é simples, o resíduo de 1/p nela é 1/p'(r)

Artur

>
> []s,
> Claudio.
>
>
>
>
>
>
>
>
> 2018-03-21 16:32 GMT-03:00 Artur Steiner :
>
>> 1) Mostre que, se f e g são funções inteiras tais que |f(z)| <= |g(z)|
>> para todo complexo z, então, também para todo z, f(z) = k g(z), onde k é
>> uma constante complexa.
>>
>> 2) Mostre que o polinômio P(z) = z^n (z - 2) - 1, n inteiro positivo, tem
>> exatamente n raízes (contando multiplicidades) no disco aberto D(0, 1).
>>
>> 3) Mostre que se f é inteira e injetora, então f é um mapeamento afim não
>> constante (logo, f é bijetora)
>>
>> 4) Mostre que uma função inteira é uniformemente contínua no plano se, e
>> somente se, for um mapeamento afim.
>>
>> Artur
>>
>> Enviado do meu iPad(
>> --
>> Esta mensagem foi verificada pelo sistema de antivírus e
>>  acredita-se estar livre de perigo.
>>
>>
>> =
>> Instru�ões para entrar na lista, sair da lista e usar a lista em
>> http://www.mat.puc-rio.br/~obmlistas/obm-l.html
>> =
>>
>
>
> --
> Esta mensagem foi verificada pelo sistema de antivírus e
> acredita-se estar livre de perigo.

-- 
Esta mensagem foi verificada pelo sistema de antiv�rus e
 acredita-se estar livre de perigo.



[obm-l] Re: [obm-l] Re: [obm-l] Re: [obm-l] Problemas interessantes de análise complexa

2018-03-27 Por tôpico Claudio Buffara
 A rigidez à qual eu me referia me parece ter mais a ver com o fato de que
uma função analítica, por também ser conforme, transforma um "quadrado
infinitesimal" em outro "quadrado infinitesimal", enquanto que uma função
que é apenas real-diferenciável (no sentido da análise no R^n, olhando C
como R^2) transforma um tal quadrado num "paralelogramo infinitesimal", que
em geral, nem retângulo é.
Isso tá bem ilustrado no livro Visual Complex Analysis, que eu mencionei
antes.

[]s,
Claudio.


2018-03-27 14:12 GMT-03:00 Bernardo Freitas Paulo da Costa <
bernardo...@gmail.com>:

> 2018-03-27 13:36 GMT-03:00 Claudio Buffara :
> > Os problemas 1, 3 e 4 me parecem ser consequências da "rigidez" que a
> > diferenciabilidade complexa impõe às funções analíticas e que, pra mim,
> está
> > longe de ser algo intuitivo.
>
> É, a estrutura complexa é muito impressionante.  Parte da rigidez é
> puramente algébrica (como abaixo), mas existem fenômenos para os quais
> eu não encontro um análogo algébrico legal (como o próximo |f| <= |g|
> ...)
>
> > Por exemplo, no problema 1, se g(z) = exp(z), então a conclusão decorre
> do
> > teorema de Liouville.
> > No caso geral, temos que lidar com os zeros de g e Liouville não se
> aplica
> > (pelo menos não diretamente).
> > No entanto, se g(z) = 0, então f(z) = 0 pela desigualdade |f(z)| <=
> |g(z)|.
> > Será que essa desigualdade garante que os zeros de g(z) são
> singularidades
> > removíveis de f(z)/g(z)?
>
> Sim: a forma canônica multiplicativa de f e g em torno de um zero,
> mais a desigualdade, dá que a ordem de f é pelo menos igual à de g.  O
> fato de haver uma ordem *inteira* de anulação é o que eu chamo de
> "algebrização".
>
> Abraços,
> --
> Bernardo Freitas Paulo da Costa
>
> --
> Esta mensagem foi verificada pelo sistema de antivírus e
>  acredita-se estar livre de perigo.
>
>
> =
> Instru�ões para entrar na lista, sair da lista e usar a lista em
> http://www.mat.puc-rio.br/~obmlistas/obm-l.html
> =
>

-- 
Esta mensagem foi verificada pelo sistema de antiv�rus e
 acredita-se estar livre de perigo.



[obm-l] Re: [obm-l] Re: [obm-l] Problemas interessantes de análise complexa

2018-03-27 Por tôpico Bernardo Freitas Paulo da Costa
2018-03-27 13:36 GMT-03:00 Claudio Buffara :
> Os problemas 1, 3 e 4 me parecem ser consequências da "rigidez" que a
> diferenciabilidade complexa impõe às funções analíticas e que, pra mim, está
> longe de ser algo intuitivo.

É, a estrutura complexa é muito impressionante.  Parte da rigidez é
puramente algébrica (como abaixo), mas existem fenômenos para os quais
eu não encontro um análogo algébrico legal (como o próximo |f| <= |g|
...)

> Por exemplo, no problema 1, se g(z) = exp(z), então a conclusão decorre do
> teorema de Liouville.
> No caso geral, temos que lidar com os zeros de g e Liouville não se aplica
> (pelo menos não diretamente).
> No entanto, se g(z) = 0, então f(z) = 0 pela desigualdade |f(z)| <= |g(z)|.
> Será que essa desigualdade garante que os zeros de g(z) são singularidades
> removíveis de f(z)/g(z)?

Sim: a forma canônica multiplicativa de f e g em torno de um zero,
mais a desigualdade, dá que a ordem de f é pelo menos igual à de g.  O
fato de haver uma ordem *inteira* de anulação é o que eu chamo de
"algebrização".

Abraços,
-- 
Bernardo Freitas Paulo da Costa

-- 
Esta mensagem foi verificada pelo sistema de antiv�rus e
 acredita-se estar livre de perigo.


=
Instru��es para entrar na lista, sair da lista e usar a lista em
http://www.mat.puc-rio.br/~obmlistas/obm-l.html
=


[obm-l] Re: [obm-l] Problemas interessantes de análise complexa

2018-03-27 Por tôpico Claudio Buffara
Os problemas 1, 3 e 4 me parecem ser consequências da "rigidez" que a
diferenciabilidade complexa impõe às funções analíticas e que, pra mim,
está longe de ser algo intuitivo.

Por exemplo, no problema 1, se g(z) = exp(z), então a conclusão decorre do
teorema de Liouville.
No caso geral, temos que lidar com os zeros de g e Liouville não se aplica
(pelo menos não diretamente).
No entanto, se g(z) = 0, então f(z) = 0 pela desigualdade |f(z)| <= |g(z)|.
Será que essa desigualdade garante que os zeros de g(z) são singularidades
removíveis de f(z)/g(z)?

No problema 2, fazendo w = 1/z, obtemos o polinômio w^(n+1) + 2w - 1 e o
problema passa a ser o de provar que este tem uma única raiz no interior de
D(0,1).
(também é preciso mostrar que P(z) não tem raízes com |z| = 1, mas isso é
relativamente simples: se P(z) = 0 e |z| = 1, então z^n(z-2) = 1 ==> |z-2|
= 1 ==> z = 1. Mas P(1) = -2 <> 0).
O teorema de Rouché diz que, se f e g forem analíticas no fecho de D(0,1) e
que se |g(w)| < |f(w)| para |w| = 1, então f e f+g têm o mesmo número de
raízes no interior de D(0,1).
Tomemos f(w) = 2w - 0,5  e  g(w) = w^(n+1) - 0,5.
Então:
f(w) tem uma única raiz (igual a 0,25) no interior de D(0,1);
f(w) + g(w) = w^(n+1) + 2w - 1;
para |w| = 1,
|g(w)| = |w^(n+1) - 0,5| <= 1 + 0,5 = 1,5, com igualdade sss w for uma raiz
(n+1)-ésima de -1,
|f(w)| = |2w - 0,5| >= 2 - 0,5 = 1,5, com igualdade sss w = 1.
Como 1 não é uma raiz (n+1)-ésima de -1, teremos sempre a desigualdade
estrita |g(w)| < |f(w)| para |w| = 1.
Assim, as condições do teorema de Rouché são satisfeitas e, portanto,
w^(n+1) + 2w - 1 tem o mesmo número de raízes (a saber, 1) que 2w - 0,5 no
interior de D(0,1).

[]s,
Claudio.








2018-03-21 16:32 GMT-03:00 Artur Steiner :

> 1) Mostre que, se f e g são funções inteiras tais que |f(z)| <= |g(z)|
> para todo complexo z, então, também para todo z, f(z) = k g(z), onde k é
> uma constante complexa.
>
> 2) Mostre que o polinômio P(z) = z^n (z - 2) - 1, n inteiro positivo, tem
> exatamente n raízes (contando multiplicidades) no disco aberto D(0, 1).
>
> 3) Mostre que se f é inteira e injetora, então f é um mapeamento afim não
> constante (logo, f é bijetora)
>
> 4) Mostre que uma função inteira é uniformemente contínua no plano se, e
> somente se, for um mapeamento afim.
>
> Artur
>
> Enviado do meu iPad(
> --
> Esta mensagem foi verificada pelo sistema de antivírus e
>  acredita-se estar livre de perigo.
>
>
> =
> Instru�ões para entrar na lista, sair da lista e usar a lista em
> http://www.mat.puc-rio.br/~obmlistas/obm-l.html
> =
>

-- 
Esta mensagem foi verificada pelo sistema de antiv�rus e
 acredita-se estar livre de perigo.



Re: [obm-l] Problemas interessantes

2013-08-25 Por tôpico Bernardo Freitas Paulo da Costa
2013/8/25 Benedito :
> Eduardo,
>
> A sua observação faz sentido. O que falta é a vírgula !!!:
>
> Um triângulo equilátero de lado 2012 está dividido em 2012  triângulos
> equiláteros menores, de lado 1.
Continua errado. As áreas não batem.

Eu acho que é "divida o triângulo de lado 2012 em montes de
trianglinhos de lado 1 com lados paralelos aos originais. Só tem um
jeito de fazer isso. E o problema começa na frase seguinte."

Dica: fazer esta mesma operação (e botar as mesmas formigas, mas
menos) em triângulos de lado 1 e 2.

Abraços,
-- 
Bernardo Freitas Paulo da Costa

-- 
Esta mensagem foi verificada pelo sistema de antivírus e
 acredita-se estar livre de perigo.


=
Instruções para entrar na lista, sair da lista e usar a lista em
http://www.mat.puc-rio.br/~obmlistas/obm-l.html
=


Re: [obm-l] Problemas interessantes

2013-08-24 Por tôpico Eduardo Wilner
Um triângulo  equilátero de lado nse divide em ntriângulos de lado 1 ???!!!

 




 De: Benedito 
Para: obm-l@mat.puc-rio.br 
Enviadas: Quinta-feira, 22 de Agosto de 2013 4:39
Assunto: [obm-l] Problemas interessantes
 


Segue dois problemas interessantes.
Benedito
 
Problema 1
Um triângulo equilátero de lado 2012 está dividido em 2012  triângulos 
equiláteros menores de lado 1
mediante paralelas ao seus lados. Em cada vértice de um triângulo menor há uma 
formiga. No mesmo instante, 
todas as formigas começam a caminhar com a mesma velocidade pelas retas da 
triangulação.
Ao chegar  a outro vértice giram 60º ou 120º à esquerda ou à direita e  seguem 
movendo-se.
Determinar se é possível que este movimento se desenvolva para sempre sem ter  
nunca duas
formigas em um mesmo vértice de um triângulo menor.
 
Problema 2
Associar aos vértices de um polígono convexo de 33 lados os números inteiros de 
1 a 33, sem repetir, e em seguida, associar aos lados do polígono a soma dos 
números de seus extremos. 
O objetivo é que os números associados aos lados sejam 33 inteiros consecutivos 
ordenados.
-- 
Esta mensagem foi verificada pelo sistema de antivírus e 
acredita-se estar livre de perigo. 
-- 
Esta mensagem foi verificada pelo sistema de antivírus e
 acredita-se estar livre de perigo.



Re: [obm-l] Problemas interessantes

2013-08-23 Por tôpico PONCE
 Obrigado Benedito,
 pelos belos problemas. 
 LUIZ PONCE 
 On Qui 22/08/13 04:39 , "Benedito" bened...@ufrnet.br sent:
Segue dois problemas interessantes.

Benedito
Problema 1

Um triângulo equilátero de lado 2012 está dividido em 2012 
triângulos equiláteros menores de lado 1

mediante paralelas ao seus lados. Em cada vértice de um triângulo
menor há uma formiga. No mesmo instante, 

todas as formigas começam a caminhar com a mesma velocidade pelas
retas da triangulação.

Ao chegar  a outro vértice giram 60º ou 120º à esquerda ou à
direita e  seguem movendo-se.

Determinar se é possível que este movimento se desenvolva para
sempre sem ter  nunca duas

formigas em um mesmo vértice de um triângulo menor.
Problema 2

Associar aos vértices de um polígono convexo de 33 lados os
números inteiros de 1 a 33, sem repetir, e em seguida, associar aos
lados do polígono a soma dos números de seus extremos. 

O objetivo é que os números associados aos lados sejam 33 inteiros
consecutivos ordenados.
 -- 
 Esta mensagem foi verificada pelo sistema de antivírus e  
  acredita-se estar livre de perigo. 

-- 
Esta mensagem foi verificada pelo sistema de antiv�rus e
 acredita-se estar livre de perigo.



Re: [obm-l] problemas dificeis

2012-10-16 Por tôpico terence thirteen
Em 15 de outubro de 2012 21:16, Heitor Bueno Ponchio Xavier
 escreveu:
> Gostaria de ajuda nos seguintes problemas:
>
> 01. Encontre todos os pares ordenados (m,n) em que m e n são inteiros
> positivos tais que (n³+1)/(mn-1) é um inteiro.
>
> 02. Seja p um número primo. Prove que existe um número primo q tal que, para
> todo inteiro n, o número n^p - p não é divisível por

Ambos os problemas foram de IMO.
1 - 
http://www.artofproblemsolving.com/Forum/viewtopic.php?p=6413&sid=e21096674ffec73c290f9e181560c748#p6413

2- 
http://www.artofproblemsolving.com/Forum/viewtopic.php?p=266&sid=e21096674ffec73c290f9e181560c748#p266

-- 
/**/
神が祝福

Torres

=
Instru��es para entrar na lista, sair da lista e usar a lista em
http://www.mat.puc-rio.br/~obmlistas/obm-l.html
=


Re: [obm-l] Problemas dificeis

2012-03-21 Por tôpico Heitor Bueno Ponchio Xavier
Obrigado a todos!

Em 21 de março de 2012 23:40, marcelo rufino de oliveira <
marcelo_ruf...@hotmail.com> escreveu:

>  Na 2ª questão faça assim:
>
> y1 = x1
> y2 = x2 + 1
> y3 = x3 + 2
> y4 = x4 + 3
> y5 = x5 + 4
>
> Assim, escolher x1, x2, x3, x4, x5, x6 inteiros de modo que
> 1<=x1<=x2<=x3<=x4<=x5<=6 é equivalente a escolher 1 <= y1< y2 < y3 < y4 <
> y5 <= 10.
> Para tanto, basta escolher 5 números de 1 a 10, ou seja, esta quantidade é
> igual a C(10, 5) =  252.
>
> Marcelo Rufino de Oliveira
>
> --------------
> From: joao_maldona...@hotmail.com
> To: obm-l@mat.puc-rio.br
> Subject: RE: [obm-l] Problemas dificeis
> Date: Wed, 21 Mar 2012 01:19:01 -0300
>
>
> Para o b pense assim
> Sendo a, b, c, d, e, f   a quantidade de vezes que aparecem os numeros 1,
> 2, 3, 4, 5, 6 na quina (x1, x2, x3, x4, x5) respectivamente
> Temos que o problema se resume a encontar as solucoes nao negativas de
> a+b+c+d+e+f=5
> que  eh nada mais que C(10, 6) =210
>
>
> Se nao errei em nenhuma passagem acho q eh isso
>
> []'s
> joao
>
> --
> Date: Mon, 19 Mar 2012 21:47:45 -0300
> Subject: [obm-l] Problemas dificeis
> From: heitor.iyp...@gmail.com
> To: obm-l@mat.puc-rio.br
>
> 1-Dados 2n pontos no espaço,n>1, prove que:
> i) Se eles forem ligados por n²+1 segmentos Mostrque no minimo um
> triangulo é formado.
> ii) é possivel ligar 2n pontos por meio de n² segmentos sem que qualquer
> triangulo seja formado.
>
> 2- Quantas são as soluções inteiras de:
> 1<=x1<=x2<=x3<=x4<=x5<=6
>


RE: [obm-l] Problemas dificeis

2012-03-21 Por tôpico marcelo rufino de oliveira

Na 2ª questão faça assim:

y1 = x1
y2 = x2 + 1
y3 = x3 + 2
y4 = x4 + 3
y5 = x5 + 4

Assim, escolher x1, x2, x3, x4, x5, x6 inteiros de modo que 
1<=x1<=x2<=x3<=x4<=x5<=6 é equivalente a escolher 1 <= y1< y2 < y3 < y4 < y5 <= 
10.
Para tanto, basta escolher 5 números de 1 a 10, ou seja, esta quantidade é 
igual a C(10, 5) =  252.

Marcelo Rufino de Oliveira

From: joao_maldona...@hotmail.com
To: obm-l@mat.puc-rio.br
Subject: RE: [obm-l] Problemas dificeis
Date: Wed, 21 Mar 2012 01:19:01 -0300






Para o b pense assim
Sendo a, b, c, d, e, f   a quantidade de vezes que aparecem os numeros 1, 2, 3, 
4, 5, 6 na quina (x1, x2, x3, x4, x5) respectivamente
Temos que o problema se resume a encontar as solucoes nao negativas de
a+b+c+d+e+f=5
que  eh nada mais que C(10, 6) =210


Se nao errei em nenhuma passagem acho q eh isso  

[]'s
joao

Date: Mon, 19 Mar 2012 21:47:45 -0300
Subject: [obm-l] Problemas dificeis
From: heitor.iyp...@gmail.com
To: obm-l@mat.puc-rio.br

1-Dados 2n pontos no espaço,n>1, prove que:
i) Se eles forem ligados por n²+1 segmentos Mostrque no minimo um triangulo é 
formado.
ii) é possivel ligar 2n pontos por meio de n² segmentos sem que qualquer 
triangulo seja formado.


2- Quantas são as soluções inteiras de:
1<=x1<=x2<=x3<=x4<=x5<=6

  

RE: [obm-l] Problemas dificeis

2012-03-21 Por tôpico João Maldonado
Realmente  o erro  foi meu :D
A quantidade de solucoes de a+b+c+d+e+f=5 é C(10, 5)=252 e nao 210, hehe

[]s
Joao

Date: Wed, 21 Mar 2012 11:14:59 -0300
Subject: Re: [obm-l] Problemas dificeis
From: heitor.iyp...@gmail.com
To: obm-l@mat.puc-rio.br

João o gabarito ta dando 252

2012/3/21 João Maldonado 




Para o b pense assim
Sendo a, b, c, d, e, f   a quantidade de vezes que aparecem os numeros 1, 2, 3, 
4, 5, 6 na quina (x1, x2, x3, x4, x5) respectivamente
Temos que o problema se resume a encontar as solucoes nao negativas de

a+b+c+d+e+f=5
que  eh nada mais que C(10, 6) =210


Se nao errei em nenhuma passagem acho q eh isso  

[]'s
joao

Date: Mon, 19 Mar 2012 21:47:45 -0300
Subject: [obm-l] Problemas dificeis

From: heitor.iyp...@gmail.com
To: obm-l@mat.puc-rio.br

1-Dados 2n pontos no espaço,n>1, prove que:

i) Se eles forem ligados por n²+1 segmentos Mostrque no minimo um triangulo é 
formado.
ii) é possivel ligar 2n pontos por meio de n² segmentos sem que qualquer 
triangulo seja formado.


2- Quantas são as soluções inteiras de:
1<=x1<=x2<=x3<=x4<=x5<=6
  


  

Re: [obm-l] Problemas dificeis

2012-03-21 Por tôpico Heitor Bueno Ponchio Xavier
João o gabarito ta dando 252

2012/3/21 João Maldonado 

> Para o b pense assim
> Sendo a, b, c, d, e, f   a quantidade de vezes que aparecem os numeros 1,
> 2, 3, 4, 5, 6 na quina (x1, x2, x3, x4, x5) respectivamente
> Temos que o problema se resume a encontar as solucoes nao negativas de
> a+b+c+d+e+f=5
> que  eh nada mais que C(10, 6) =210
>
>
> Se nao errei em nenhuma passagem acho q eh isso
>
> []'s
> joao
>
> --
> Date: Mon, 19 Mar 2012 21:47:45 -0300
> Subject: [obm-l] Problemas dificeis
> From: heitor.iyp...@gmail.com
> To: obm-l@mat.puc-rio.br
>
>
> 1-Dados 2n pontos no espaço,n>1, prove que:
> i) Se eles forem ligados por n²+1 segmentos Mostrque no minimo um
> triangulo é formado.
> ii) é possivel ligar 2n pontos por meio de n² segmentos sem que qualquer
> triangulo seja formado.
>
> 2- Quantas são as soluções inteiras de:
> 1<=x1<=x2<=x3<=x4<=x5<=6
>


RE: [obm-l] Problemas dificeis

2012-03-20 Por tôpico João Maldonado
Para o b pense assim
Sendo a, b, c, d, e, f   a quantidade de vezes que aparecem os numeros 1, 2, 3, 
4, 5, 6 na quina (x1, x2, x3, x4, x5) respectivamente
Temos que o problema se resume a encontar as solucoes nao negativas de
a+b+c+d+e+f=5
que  eh nada mais que C(10, 6) =210


Se nao errei em nenhuma passagem acho q eh isso  

[]'s
joao

Date: Mon, 19 Mar 2012 21:47:45 -0300
Subject: [obm-l] Problemas dificeis
From: heitor.iyp...@gmail.com
To: obm-l@mat.puc-rio.br

1-Dados 2n pontos no espaço,n>1, prove que:
i) Se eles forem ligados por n²+1 segmentos Mostrque no minimo um triangulo é 
formado.
ii) é possivel ligar 2n pontos por meio de n² segmentos sem que qualquer 
triangulo seja formado.


2- Quantas são as soluções inteiras de:
1<=x1<=x2<=x3<=x4<=x5<=6
  

Re: [obm-l] Problemas dificeis

2012-03-19 Por tôpico terence thirteen
Procure por Teorema de Turán para o primeiro problema.

Em 19 de março de 2012 21:47, Heitor Bueno Ponchio Xavier
 escreveu:
> 1-Dados 2n pontos no espaço,n>1, prove que:
> i) Se eles forem ligados por n²+1 segmentos Mostrque no minimo um triangulo
> é formado.
> ii) é possivel ligar 2n pontos por meio de n² segmentos sem que qualquer
> triangulo seja formado.
>
> 2- Quantas são as soluções inteiras de:
> 1<=x1<=x2<=x3<=x4<=x5<=6



-- 
/**/
神が祝福

Torres

=
Instru��es para entrar na lista, sair da lista e usar a lista em
http://www.mat.puc-rio.br/~obmlistas/obm-l.html
=


RE: [obm-l] Problemas de Geometria - Morgado II

2011-11-18 Por tôpico João Maldonado


Tangente é a reta   que intersepta  uma curva em um e  somente um ponto,  
logo ela  deve necessariamente  formar  90º  com a reta  que passa pelo centro 
e pelo ponto de tangênciaComo  as duas retas  que partem   dos dois centros 
formam 90°, elas são  congruentes, daonde vem que os 3 pontos  são colineares.  
Tem como provar por analítica também,  aí fica o desafio.
[]'sJoão

From: marconeborge...@hotmail.com
To: obm-l@mat.puc-rio.br
Subject: RE: [obm-l] Problemas de Geometria - Morgado II
Date: Fri, 18 Nov 2011 22:25:34 +








onde encontro uma justificativa dessa propriedade basica do circulo tangente?
 




From: joao_maldona...@hotmail.com
To: obm-l@mat.puc-rio.br
Subject: RE: [obm-l] Problemas de Geometria - Morgado II
Date: Fri, 18 Nov 2011 17:25:23 -0200





O  primeiro eu fiz por analítica,  acho que fica mais fácil

Sendo  A(0, h), B(a, h), C(b, 0), D(0, 0)
Temos que  AC é perpendicular à BD  -> -h/b - -1/(h/a) ->  h = (ab)^(1/2)


Quanto  ao segundo é só usar a propriedade básica do  círculo tangente  -> há  
uma reta que une o ponto  de tangência  e os centros de quaisquer dois círculos 
tangentes




Sendo  assim,  Seja A o centro do círculo  menor e B o do círculo médio  e O a 
origem (centro do círculo maior).  Seja C a  projeção de A sobre as abssissas.


Temos:
BA=R/2 + r
BC = R/2-r
Logo  AB = sqrt(2Rr)
OA =  R-r  
OB=r


Usando Pitáagras


(R-r)² = 2Rr + r² -> R² =  4Rr ->  r = R/4


[]'s
João




Date: Thu, 17 Nov 2011 18:22:54 -0800
From: aazinco...@yahoo.com.br
Subject: [obm-l] Problemas de Geometria - Morgado II
To: obm-l@mat.puc-rio.br



Boa noite!


Estou com dificuldades para resolver os problemas 113 e 249 do livro Geometria 
II do Morgado.


113) Um trapézio retângulo de bases a e b possui diagonais perpendiculares. 
Quanto mede a altura desse trapézio?


249)  Considere o quadrante de raio R da figura. Calcule a área em vermelho 
(figura em anexo). 

Tentei vários métodos para encontrar o raio da menor circunferência, mas não 
consegui. Alguém poderia dar uma dica para eu destravar?


Muito obrigado!








  

RE: [obm-l] Problemas de Geometria - Morgado II

2011-11-18 Por tôpico marcone augusto araújo borges

onde encontro uma justificativa dessa propriedade basica do circulo tangente?
 



From: joao_maldona...@hotmail.com
To: obm-l@mat.puc-rio.br
Subject: RE: [obm-l] Problemas de Geometria - Morgado II
Date: Fri, 18 Nov 2011 17:25:23 -0200





O  primeiro eu fiz por analítica,  acho que fica mais fácil

Sendo  A(0, h), B(a, h), C(b, 0), D(0, 0)
Temos que  AC é perpendicular à BD  -> -h/b - -1/(h/a) ->  h = (ab)^(1/2)


Quanto  ao segundo é só usar a propriedade básica do  círculo tangente  -> há  
uma reta que une o ponto  de tangência  e os centros de quaisquer dois círculos 
tangentes




Sendo  assim,  Seja A o centro do círculo  menor e B o do círculo médio  e O a 
origem (centro do círculo maior).  Seja C a  projeção de A sobre as abssissas.


Temos:
BA=R/2 + r
BC = R/2-r
Logo  AB = sqrt(2Rr)
OA =  R-r  
OB=r


Usando Pitáagras


(R-r)² = 2Rr + r² -> R² =  4Rr ->  r = R/4


[]'s
João




Date: Thu, 17 Nov 2011 18:22:54 -0800
From: aazinco...@yahoo.com.br
Subject: [obm-l] Problemas de Geometria - Morgado II
To: obm-l@mat.puc-rio.br



Boa noite!


Estou com dificuldades para resolver os problemas 113 e 249 do livro Geometria 
II do Morgado.


113) Um trapézio retângulo de bases a e b possui diagonais perpendiculares. 
Quanto mede a altura desse trapézio?


249)  Considere o quadrante de raio R da figura. Calcule a área em vermelho 
(figura em anexo). 

Tentei vários métodos para encontrar o raio da menor circunferência, mas não 
consegui. Alguém poderia dar uma dica para eu destravar?


Muito obrigado!







  

RE: [obm-l] Problemas de Geometria - Morgado II

2011-11-18 Por tôpico João Maldonado

O  primeiro eu fiz por analítica,  acho que fica mais fácil
Sendo  A(0, h), B(a, h), C(b, 0), D(0, 0)Temos que  AC é perpendicular à BD  -> 
-h/b - -1/(h/a) ->  h = (ab)^(1/2)
Quanto  ao segundo é só usar a propriedade básica do  círculo tangente  -> há  
uma reta que une o ponto  de tangência  e os centros de quaisquer dois círculos 
tangentes

Sendo  assim,  Seja A o centro do círculo  menor e B o do círculo médio  e O a 
origem (centro do círculo maior).  Seja C a  projeção de A sobre as abssissas.
Temos:BA=R/2 + rBC = R/2-rLogo  AB = sqrt(2Rr)OA =  R-r  OB=r
Usando Pitáagras
(R-r)² = 2Rr + r² -> R² =  4Rr ->  r = R/4
[]'sJoão
Date: Thu, 17 Nov 2011 18:22:54 -0800
From: aazinco...@yahoo.com.br
Subject: [obm-l] Problemas de Geometria - Morgado II
To: obm-l@mat.puc-rio.br

Boa noite!
Estou com dificuldades para resolver os problemas 113 e 249 do livro Geometria 
II do Morgado.
113) Um trapézio retângulo de bases a e b possui diagonais perpendiculares. 
Quanto mede a altura desse trapézio?
249)  Considere o quadrante de raio R da figura. Calcule a área em vermelho 
(figura em anexo). 
Tentei vários métodos para encontrar o raio da menor circunferência, mas não 
consegui. Alguém poderia dar uma dica para eu destravar?
Muito obrigado!



  

Re: [obm-l] Problemas de Geometria - Morgado II

2011-11-18 Por tôpico Eduardo Wilner
113) Os triângulos formados com as bases, as diagonais e a altura, h, são 
semelhantes, logo b/h = h/a ,ou, h = sqrt (ab).
Assim a área vale sqrt(ab)(a+b)/2.

Quanto ao 249), não tenho a figura...

[]'s



Re: [obm-l] Problemas(polinomoi- ideias!)

2011-07-06 Por tôpico Johann Dirichlet
Continuando: acho que, quando se faz alguma manipulação algébrica, a
conta falha miseravelmente para graus grandes.

Usando a ideia do Ralph, o polinomio em questão é par ou ímpar. Mas
quando eu abro as contas, usando um "exemplo finito" (uma tentativa do
genero f(x)=ax^2+bx+c), dá muito desencontro de expoentes. Vou mostrar
as contas outro dia (elas sao grandinhas!).

Em 05/07/11, Johann Dirichlet escreveu:
> Eu, na verdade, tentei achar um polinomio que desse certo. E cantei
> vitória antes do tempo...
> E a sua ideia de par-ou-impar matou de vez as esperanças: L^2+1
> aumenta o módulo.
>
> O Marcone tambem me enviou este e-mail corrigido. Eu estou matutando
> nele, e achei alguns exemplos. Ao que me parece, para cada grau de
> polinomio, existe um único polinômio que resolve a equação funcional.
>
> Eu descobri que estes poucos polinomios tem coeficiente líder 1 e
> sub-líder 0. Mas não avencei muito para dar um parecer final.
>
> No mais, acho que x^2+1 me lembra de usar i^2=-1...
>
> Em 04/07/11, Ralph Teixeira escreveu:
>> Como voce disse, se a eh uma raiz de P(x), entao a^2+1 tem que ser raiz
>> de
>> P(x) tambem. Entao se voce pegar as raizes de P(x) e "aplicar" x^2+1
>> nelas,
>> voce ainda tem que cair em raizes. Portanto, dada uma raiz qualquer a,
>> temos
>> que a^2+1, (a^2+1)^2+1, etc. gera varias raizes de P(x). Como P(x) tem
>> que
>> ter um numero finito de raizes distintas, essa sequencia tem que gerar um
>> ciclo, tem que repetir em algum momento.
>>
>> Que ciclo? O caso mais simples seria fazer um ciclo de tamanho 1, ou
>> seja,
>> fazer logo a^2+1=a, para o ciclo soh ter um termo. Foi a ideia que voce
>> colocou... mas nao dah certo -- a condicao deste ciclo ser finita eh
>> NECESSARIA para ter a igualdade pedida, mas ter um ciclo de raizes nao eh
>> SUFICENTE para garantir a igualdade pedida.
>>
>> O Shine botou a bola embaixo do braco e levou para casa: nao ha polinomio
>> com a condicao pedida... :)
>>
>> Mas, olha soh:* o Marcone, que propos o problema original, me mandou um
>> E-mail dizendo que realmente o enunciado original era mesmo
>> p(x^2+1)=(p(x))^2+1, mas por algum motivo ele nao conseguiu colocar a
>> correcao na lista.* Entao ainda ha um problema interessante (mas bem
>> diferente) para fazer.
>>
>> (Eu jah vi isso em algum lugar, mas nao lembro onde...)
>>
>> Abraco,
>> Ralph
>> 2011/7/4 Johann Dirichlet 
>>
>>> Puxa! Mas onde esta o erro da minha solução?
>>>
>>> Anyway, inicialmente pensei em fatorar o dito polinomio.
>>> Creio que ele seja mônico, abrindo a expressão geral o fator máximo é
>>> a^2=a.
>>> Aí, escreve ele na forma deprodutos (x-a_i).. Basicamente, um lado
>>> fica na forma
>>> x^2+1-a_ i, e o outro como (x-a_ i)^2. Supóndo que as raízes são, em
>>> alguma ordem, iguais, dá pra chegar em algum lugar.
>>>
>>>
>>>
>>>
>>> Em 01/07/11, Ralph Teixeira escreveu:
>>> > O raciocínio do Dirichlet mostra que basta achar UM polinômio (não
>>> > constante) que tenha esta propriedade. Afinal, como ele mostrou, se
>>> > p(x)
>>> > serve, então q(x)=(p(x))^2 também serve.
>>> >
>>> > Mas seja lá quem for o polinômio mágico, eu sei que ou ele é um
>>> > polinômio
>>> > par ou ele é ímpar. Afinal, escreva p(x)=P(x)+I(x) onde P(x) tem
>>> > apenas
>>> os
>>> > termos de grau par e I(x) tem apenas os de grau ímpar.
>>> >
>>> > Ora, p(x)^2=(P^2+I^2)+2PI. Note que P^2+I^2 é um polinômio par e 2PI é
>>> > ímpar.
>>> >
>>> > Mas a condição manda que p^2=p(x^2+1), que é uma função par. Então o
>>> termo
>>> > 2PI não pode existir, isto é, P=0 ou I=0. Assim, p(x) é par ou ímpar.
>>> > E
>>> > x^2-x+1 não é um nem outro, então não funcionou...
>>> >
>>> > Então precisamos ainda mostrar que existe UM tal polinômio!
>>> >
>>> > Abraço,
>>> >   Ralph
>>> >
>>> > P.S.: Tem certeza que o enunciado é esse mesmo? Não seria, sei lá,
>>> > p(x^2+1)=(p(x))^2+1 ao invés?
>>> > 2011/7/1 Johann Dirichlet 
>>> >
>>> >> Em 01/07/11, Johann Dirichlet escreveu:
>>> >> > Em 30/06/11, marcone augusto araújo
>>> >> > borges escreveu:
>>> >> >>
>>> >> >> 1) Se p é inteiro primo ímpar,mostre que o numerador da fração
>>> >> >> 1+1/2+1/3+...1/(p-1) é um múltiplo de p.
>>> >> >
>>> >> > 1) Teorema de Wolstenholme, se não me engano...
>>> >> >
>>> >> > Bora lá, usar o velho truque das pontas de Gauss:
>>> >> > 1/k+1/(p-k)=p/(k(p-k));
>>> >> > assim sendo, temos um monte de frações p/(alguma coisa). Esta coisa
>>> >> > não será múltipla de p em momento nenhum, logo nada aniquila este
>>> >> > fator p.
>>> >> >
>>> >> >>
>>> >> >> 2) Mostre que existem infinitos polinômios p(x) com coeficientes
>>> reais
>>> >> >> tais
>>> >> >> que p(x^2+1) = [p(x)]^2.
>>> >>
>>> >> É mais mole do que eu pensei!
>>> >>
>>> >> 1 - Se P e Q são soluções da equação acima, P*Q também será. Óbvio!
>>> >> 2 - Um polinômio possível é x^2-x+1.
>>> >> Como sei? Simples:
>>> >>
>>> >> Se L é um zero de P, então L^2+1 também será.
>>> >> Se eu conseguir L=L^2+1, terei uma solução pronta!
>>> >> Basta abrir o polinom

Re: [obm-l] Problemas(polinomoi- ideias!)

2011-07-05 Por tôpico Johann Dirichlet
Eu, na verdade, tentei achar um polinomio que desse certo. E cantei
vitória antes do tempo...
E a sua ideia de par-ou-impar matou de vez as esperanças: L^2+1
aumenta o módulo.

O Marcone tambem me enviou este e-mail corrigido. Eu estou matutando
nele, e achei alguns exemplos. Ao que me parece, para cada grau de
polinomio, existe um único polinômio que resolve a equação funcional.

Eu descobri que estes poucos polinomios tem coeficiente líder 1 e
sub-líder 0. Mas não avencei muito para dar um parecer final.

No mais, acho que x^2+1 me lembra de usar i^2=-1...

Em 04/07/11, Ralph Teixeira escreveu:
> Como voce disse, se a eh uma raiz de P(x), entao a^2+1 tem que ser raiz de
> P(x) tambem. Entao se voce pegar as raizes de P(x) e "aplicar" x^2+1 nelas,
> voce ainda tem que cair em raizes. Portanto, dada uma raiz qualquer a, temos
> que a^2+1, (a^2+1)^2+1, etc. gera varias raizes de P(x). Como P(x) tem que
> ter um numero finito de raizes distintas, essa sequencia tem que gerar um
> ciclo, tem que repetir em algum momento.
>
> Que ciclo? O caso mais simples seria fazer um ciclo de tamanho 1, ou seja,
> fazer logo a^2+1=a, para o ciclo soh ter um termo. Foi a ideia que voce
> colocou... mas nao dah certo -- a condicao deste ciclo ser finita eh
> NECESSARIA para ter a igualdade pedida, mas ter um ciclo de raizes nao eh
> SUFICENTE para garantir a igualdade pedida.
>
> O Shine botou a bola embaixo do braco e levou para casa: nao ha polinomio
> com a condicao pedida... :)
>
> Mas, olha soh:* o Marcone, que propos o problema original, me mandou um
> E-mail dizendo que realmente o enunciado original era mesmo
> p(x^2+1)=(p(x))^2+1, mas por algum motivo ele nao conseguiu colocar a
> correcao na lista.* Entao ainda ha um problema interessante (mas bem
> diferente) para fazer.
>
> (Eu jah vi isso em algum lugar, mas nao lembro onde...)
>
> Abraco,
> Ralph
> 2011/7/4 Johann Dirichlet 
>
>> Puxa! Mas onde esta o erro da minha solução?
>>
>> Anyway, inicialmente pensei em fatorar o dito polinomio.
>> Creio que ele seja mônico, abrindo a expressão geral o fator máximo é
>> a^2=a.
>> Aí, escreve ele na forma deprodutos (x-a_i).. Basicamente, um lado
>> fica na forma
>> x^2+1-a_ i, e o outro como (x-a_ i)^2. Supóndo que as raízes são, em
>> alguma ordem, iguais, dá pra chegar em algum lugar.
>>
>>
>>
>>
>> Em 01/07/11, Ralph Teixeira escreveu:
>> > O raciocínio do Dirichlet mostra que basta achar UM polinômio (não
>> > constante) que tenha esta propriedade. Afinal, como ele mostrou, se p(x)
>> > serve, então q(x)=(p(x))^2 também serve.
>> >
>> > Mas seja lá quem for o polinômio mágico, eu sei que ou ele é um
>> > polinômio
>> > par ou ele é ímpar. Afinal, escreva p(x)=P(x)+I(x) onde P(x) tem apenas
>> os
>> > termos de grau par e I(x) tem apenas os de grau ímpar.
>> >
>> > Ora, p(x)^2=(P^2+I^2)+2PI. Note que P^2+I^2 é um polinômio par e 2PI é
>> > ímpar.
>> >
>> > Mas a condição manda que p^2=p(x^2+1), que é uma função par. Então o
>> termo
>> > 2PI não pode existir, isto é, P=0 ou I=0. Assim, p(x) é par ou ímpar. E
>> > x^2-x+1 não é um nem outro, então não funcionou...
>> >
>> > Então precisamos ainda mostrar que existe UM tal polinômio!
>> >
>> > Abraço,
>> >   Ralph
>> >
>> > P.S.: Tem certeza que o enunciado é esse mesmo? Não seria, sei lá,
>> > p(x^2+1)=(p(x))^2+1 ao invés?
>> > 2011/7/1 Johann Dirichlet 
>> >
>> >> Em 01/07/11, Johann Dirichlet escreveu:
>> >> > Em 30/06/11, marcone augusto araújo
>> >> > borges escreveu:
>> >> >>
>> >> >> 1) Se p é inteiro primo ímpar,mostre que o numerador da fração
>> >> >> 1+1/2+1/3+...1/(p-1) é um múltiplo de p.
>> >> >
>> >> > 1) Teorema de Wolstenholme, se não me engano...
>> >> >
>> >> > Bora lá, usar o velho truque das pontas de Gauss:
>> >> > 1/k+1/(p-k)=p/(k(p-k));
>> >> > assim sendo, temos um monte de frações p/(alguma coisa). Esta coisa
>> >> > não será múltipla de p em momento nenhum, logo nada aniquila este
>> >> > fator p.
>> >> >
>> >> >>
>> >> >> 2) Mostre que existem infinitos polinômios p(x) com coeficientes
>> reais
>> >> >> tais
>> >> >> que p(x^2+1) = [p(x)]^2.
>> >>
>> >> É mais mole do que eu pensei!
>> >>
>> >> 1 - Se P e Q são soluções da equação acima, P*Q também será. Óbvio!
>> >> 2 - Um polinômio possível é x^2-x+1.
>> >> Como sei? Simples:
>> >>
>> >> Se L é um zero de P, então L^2+1 também será.
>> >> Se eu conseguir L=L^2+1, terei uma solução pronta!
>> >> Basta abrir o polinomio sem medo.
>> >>
>> >>
>> >> P.S.: saber todas as soluções me parece mais desgastante. Aplicando a
>> >> transformação T(L)=L^2+1 um numero finito de vezes, todos os
>> >> polinômios dos pontos fixos são soluções. A treta é saber se não
>> >> escapa nenhum (até porque muitos desses polinomios são fatoráveis, I
>> >> think so).
>> >>
>> >> >>
>> >> >> 3) Uma corda AB,de comprimento constante,desliza sobre uma
>> >> >> semicircunferência determinada por um diâmetro d.
>> >> >> Considere o triângulo cujos vértices são: o ponto médio da corda e
>> >> >> as
>> >> >> 

Re: [obm-l] Problemas(polinomoi- ideias!)

2011-07-04 Por tôpico Ralph Teixeira
Como voce disse, se a eh uma raiz de P(x), entao a^2+1 tem que ser raiz de
P(x) tambem. Entao se voce pegar as raizes de P(x) e "aplicar" x^2+1 nelas,
voce ainda tem que cair em raizes. Portanto, dada uma raiz qualquer a, temos
que a^2+1, (a^2+1)^2+1, etc. gera varias raizes de P(x). Como P(x) tem que
ter um numero finito de raizes distintas, essa sequencia tem que gerar um
ciclo, tem que repetir em algum momento.

Que ciclo? O caso mais simples seria fazer um ciclo de tamanho 1, ou seja,
fazer logo a^2+1=a, para o ciclo soh ter um termo. Foi a ideia que voce
colocou... mas nao dah certo -- a condicao deste ciclo ser finita eh
NECESSARIA para ter a igualdade pedida, mas ter um ciclo de raizes nao eh
SUFICENTE para garantir a igualdade pedida.

O Shine botou a bola embaixo do braco e levou para casa: nao ha polinomio
com a condicao pedida... :)

Mas, olha soh:* o Marcone, que propos o problema original, me mandou um
E-mail dizendo que realmente o enunciado original era mesmo
p(x^2+1)=(p(x))^2+1, mas por algum motivo ele nao conseguiu colocar a
correcao na lista.* Entao ainda ha um problema interessante (mas bem
diferente) para fazer.

(Eu jah vi isso em algum lugar, mas nao lembro onde...)

Abraco,
Ralph
2011/7/4 Johann Dirichlet 

> Puxa! Mas onde esta o erro da minha solução?
>
> Anyway, inicialmente pensei em fatorar o dito polinomio.
> Creio que ele seja mônico, abrindo a expressão geral o fator máximo é
> a^2=a.
> Aí, escreve ele na forma deprodutos (x-a_i).. Basicamente, um lado
> fica na forma
> x^2+1-a_ i, e o outro como (x-a_ i)^2. Supóndo que as raízes são, em
> alguma ordem, iguais, dá pra chegar em algum lugar.
>
>
>
>
> Em 01/07/11, Ralph Teixeira escreveu:
> > O raciocínio do Dirichlet mostra que basta achar UM polinômio (não
> > constante) que tenha esta propriedade. Afinal, como ele mostrou, se p(x)
> > serve, então q(x)=(p(x))^2 também serve.
> >
> > Mas seja lá quem for o polinômio mágico, eu sei que ou ele é um polinômio
> > par ou ele é ímpar. Afinal, escreva p(x)=P(x)+I(x) onde P(x) tem apenas
> os
> > termos de grau par e I(x) tem apenas os de grau ímpar.
> >
> > Ora, p(x)^2=(P^2+I^2)+2PI. Note que P^2+I^2 é um polinômio par e 2PI é
> > ímpar.
> >
> > Mas a condição manda que p^2=p(x^2+1), que é uma função par. Então o
> termo
> > 2PI não pode existir, isto é, P=0 ou I=0. Assim, p(x) é par ou ímpar. E
> > x^2-x+1 não é um nem outro, então não funcionou...
> >
> > Então precisamos ainda mostrar que existe UM tal polinômio!
> >
> > Abraço,
> >   Ralph
> >
> > P.S.: Tem certeza que o enunciado é esse mesmo? Não seria, sei lá,
> > p(x^2+1)=(p(x))^2+1 ao invés?
> > 2011/7/1 Johann Dirichlet 
> >
> >> Em 01/07/11, Johann Dirichlet escreveu:
> >> > Em 30/06/11, marcone augusto araújo
> >> > borges escreveu:
> >> >>
> >> >> 1) Se p é inteiro primo ímpar,mostre que o numerador da fração
> >> >> 1+1/2+1/3+...1/(p-1) é um múltiplo de p.
> >> >
> >> > 1) Teorema de Wolstenholme, se não me engano...
> >> >
> >> > Bora lá, usar o velho truque das pontas de Gauss:
> >> > 1/k+1/(p-k)=p/(k(p-k));
> >> > assim sendo, temos um monte de frações p/(alguma coisa). Esta coisa
> >> > não será múltipla de p em momento nenhum, logo nada aniquila este
> >> > fator p.
> >> >
> >> >>
> >> >> 2) Mostre que existem infinitos polinômios p(x) com coeficientes
> reais
> >> >> tais
> >> >> que p(x^2+1) = [p(x)]^2.
> >>
> >> É mais mole do que eu pensei!
> >>
> >> 1 - Se P e Q são soluções da equação acima, P*Q também será. Óbvio!
> >> 2 - Um polinômio possível é x^2-x+1.
> >> Como sei? Simples:
> >>
> >> Se L é um zero de P, então L^2+1 também será.
> >> Se eu conseguir L=L^2+1, terei uma solução pronta!
> >> Basta abrir o polinomio sem medo.
> >>
> >>
> >> P.S.: saber todas as soluções me parece mais desgastante. Aplicando a
> >> transformação T(L)=L^2+1 um numero finito de vezes, todos os
> >> polinômios dos pontos fixos são soluções. A treta é saber se não
> >> escapa nenhum (até porque muitos desses polinomios são fatoráveis, I
> >> think so).
> >>
> >> >>
> >> >> 3) Uma corda AB,de comprimento constante,desliza sobre uma
> >> >> semicircunferência determinada por um diâmetro d.
> >> >> Considere o triângulo cujos vértices são: o ponto médio da corda e as
> >> >> projeções ortogonais dos seus extremos A e B
> >> >> sobre o diâmetro d.Mostre que ,durante o deslizamento da corda,esse
> >> >> triângulo é sempre isósceles e nunca muda de formato(i.é.,os ângulos
> do
> >> >> triângulo são constantes)
> >> >
> >> > Faz um desenho!
> >> > Diâmetro r;centro O, raio 1; corda AB, tamanho d, médio M; AB
> >> > projetado em r dá XY.
> >> >
> >> > O triangulo AOB é obviamente isósceles.
> >> > Os quadrilateros XOMA e YOMB são inscritíveis de diâmetros OA e OB
> >> > respectivamente (angulos de 90 graus).
> >> >
> >> > Temos OXM=OAM=OBM=OYM, logo XMY é isosceles. E o angulo OBA depende
> >> > unicamente de d.
> >> >
> >> > P.S.: duvido que os triangulos sejam todos congruentes. O angulo XOM
> >> > define o tamanho

Re: [obm-l] Problemas(polinomoi- ideias!)

2011-07-04 Por tôpico Johann Dirichlet
Sobre a sua mesg: por "indução infinita" na derivação, nunca acharemos
um polinômio?

Comments:
Bem, não tô com a mensagem full, mas basicamente o Ralph provou que o
polinômio é par ou ímpar. Não dá pra ser coluna do meio.
Realmente, raiz 0 falha miseravelmente, isto é imediato. Mas, o lance
é: não existe nenhum polinômio que funcione??

Realmente,pensando na minha ideia das raizes, pareece que L^2+1 tem
modulo maior que L (uma desigualdade das medias mataria isso). Ou
seja, não tem como, depois de aplicar L^2+1 um numero finito de vezes,
voltar a L.

Quanto a outra, comecei a atacar hoje.


Em 04/07/11, Carlos Yuzo Shine escreveu:
> Eu acho que não existem infinitos polinômios (a única solução é o polinômio
> nulo). Isso, é claro, falando do problema P(x^2 + 1) = [P(x)]^2.
>
> Antes, lembramos que zero não pode ser raiz de P: como o Ralph já provou,
> isso
> gera infinitas raízes e aí P é o polinômio nulo.
>
> Olha só: derivando P(x^2 + 1) = P(x)^2 dos dois lados encontramos
> 2xP'(x^2+1) =
> 2P(x)P'(x). Aí vou tentar usar uma ideia parecida com a do Ralph. x = 0 só
> pode
> ser raiz de P' (o que atesta o fato de P ser par, que também foi provado).
> Agora
> note que se x diferente de zero é raiz de P' então x^2 + 1 também é. Pena
> que só
> temos o zero. Mas aí fazemos de trás para frente (isto é, aplicamos uma
> inversa
> f de x^2 + 1): troca x = f(0) = i. Então i é raiz de P ou P'. Mas se for
> raiz de
> P, então i^2 + 1 = 0 é raiz e dá problema do mesmo jeito. Então é raiz de
> P'.
> Mas aí é só continuar.
>
> Definindo f melhor: f(r) é a raiz quadrada de r - 1 que tem argumento arg(r
> -
> 1)/2. A minha pergunta é: partindo do r = 0, ele entra em loop?
>
> Eu acho que não. De fato, f é injetiva: f(r) = f(s) implica (f(r))^2 + 1 =
> (f(s))^2 + 1 que é o mesmo que r = s. Então se a sequência n_0 = 0 e n_k =
> f(n_{k-1}) é periódica, ela é puramente periódica. Mas 0 não é periódica
> para
> f^{-1}(x) = x^2 + 1, então acabou (eu acho).
>
>
> Isso faz sentido?
>
> []'s
> Shine
>
>
> - Original Message 
> From: Johann Dirichlet 
> To: obm-l@mat.puc-rio.br
> Sent: Mon, July 4, 2011 12:46:11 PM
> Subject: Re: [obm-l] Problemas(polinomoi- ideias!)
>
> Puxa! Mas onde esta o erro da minha solução?
>
> Anyway, inicialmente pensei em fatorar o dito polinomio.
> Creio que ele seja mônico, abrindo a expressão geral o fator máximo é a^2=a.
> Aí, escreve ele na forma deprodutos (x-a_i).. Basicamente, um lado
> fica na forma
> x^2+1-a_ i, e o outro como (x-a_ i)^2. Supóndo que as raízes são, em
> alguma ordem, iguais, dá pra chegar em algum lugar.
>
>
>
>
> Em 01/07/11, Ralph Teixeira escreveu:
>> O raciocínio do Dirichlet mostra que basta achar UM polinômio (não
>> constante) que tenha esta propriedade. Afinal, como ele mostrou, se p(x)
>> serve, então q(x)=(p(x))^2 também serve.
>>
>> Mas seja lá quem for o polinômio mágico, eu sei que ou ele é um polinômio
>> par ou ele é ímpar. Afinal, escreva p(x)=P(x)+I(x) onde P(x) tem apenas os
>> termos de grau par e I(x) tem apenas os de grau ímpar.
>>
>> Ora, p(x)^2=(P^2+I^2)+2PI. Note que P^2+I^2 é um polinômio par e 2PI é
>> ímpar.
>>
>> Mas a condição manda que p^2=p(x^2+1), que é uma função par. Então o termo
>> 2PI não pode existir, isto é, P=0 ou I=0. Assim, p(x) é par ou ímpar. E
>> x^2-x+1 não é um nem outro, então não funcionou...
>>
>> Então precisamos ainda mostrar que existe UM tal polinômio!
>>
>> Abraço,
>>   Ralph
>>
>> P.S.: Tem certeza que o enunciado é esse mesmo? Não seria, sei lá,
>> p(x^2+1)=(p(x))^2+1 ao invés?
>> 2011/7/1 Johann Dirichlet 
>>
>>> Em 01/07/11, Johann Dirichlet escreveu:
>>> > Em 30/06/11, marcone augusto araújo
>>> > borges escreveu:
>>> >>
>>> >> 1) Se p é inteiro primo ímpar,mostre que o numerador da fração
>>> >> 1+1/2+1/3+...1/(p-1) é um múltiplo de p.
>>> >
>>> > 1) Teorema de Wolstenholme, se não me engano...
>>> >
>>> > Bora lá, usar o velho truque das pontas de Gauss:
>>> > 1/k+1/(p-k)=p/(k(p-k));
>>> > assim sendo, temos um monte de frações p/(alguma coisa). Esta coisa
>>> > não será múltipla de p em momento nenhum, logo nada aniquila este
>>> > fator p.
>>> >
>>> >>
>>> >> 2) Mostre que existem infinitos polinômios p(x) com coeficientes reais
>>> >> tais
>>> >> que p(x^2+1) = [p(x)]^2.
>>>
>>> É mais mole do que eu pensei!
>>>
>>> 1 - Se P e Q são soluções da equação 

Re: [obm-l] Problemas(polinomoi- ideias!)

2011-07-04 Por tôpico Carlos Yuzo Shine
Eu acho que não existem infinitos polinômios (a única solução é o polinômio 
nulo). Isso, é claro, falando do problema P(x^2 + 1) = [P(x)]^2.

Antes, lembramos que zero não pode ser raiz de P: como o Ralph já provou, isso 
gera infinitas raízes e aí P é o polinômio nulo.

Olha só: derivando P(x^2 + 1) = P(x)^2 dos dois lados encontramos 2xP'(x^2+1) = 
2P(x)P'(x). Aí vou tentar usar uma ideia parecida com a do Ralph. x = 0 só pode 
ser raiz de P' (o que atesta o fato de P ser par, que também foi provado). 
Agora 
note que se x diferente de zero é raiz de P' então x^2 + 1 também é. Pena que 
só 
temos o zero. Mas aí fazemos de trás para frente (isto é, aplicamos uma inversa 
f de x^2 + 1): troca x = f(0) = i. Então i é raiz de P ou P'. Mas se for raiz 
de 
P, então i^2 + 1 = 0 é raiz e dá problema do mesmo jeito. Então é raiz de P'. 
Mas aí é só continuar.

Definindo f melhor: f(r) é a raiz quadrada de r - 1 que tem argumento arg(r - 
1)/2. A minha pergunta é: partindo do r = 0, ele entra em loop?

Eu acho que não. De fato, f é injetiva: f(r) = f(s) implica (f(r))^2 + 1 = 
(f(s))^2 + 1 que é o mesmo que r = s. Então se a sequência n_0 = 0 e n_k = 
f(n_{k-1}) é periódica, ela é puramente periódica. Mas 0 não é periódica para 
f^{-1}(x) = x^2 + 1, então acabou (eu acho).


Isso faz sentido?

[]'s
Shine


- Original Message 
From: Johann Dirichlet 
To: obm-l@mat.puc-rio.br
Sent: Mon, July 4, 2011 12:46:11 PM
Subject: Re: [obm-l] Problemas(polinomoi- ideias!)

Puxa! Mas onde esta o erro da minha solução?

Anyway, inicialmente pensei em fatorar o dito polinomio.
Creio que ele seja mônico, abrindo a expressão geral o fator máximo é a^2=a.
Aí, escreve ele na forma deprodutos (x-a_i).. Basicamente, um lado
fica na forma
x^2+1-a_ i, e o outro como (x-a_ i)^2. Supóndo que as raízes são, em
alguma ordem, iguais, dá pra chegar em algum lugar.




Em 01/07/11, Ralph Teixeira escreveu:
> O raciocínio do Dirichlet mostra que basta achar UM polinômio (não
> constante) que tenha esta propriedade. Afinal, como ele mostrou, se p(x)
> serve, então q(x)=(p(x))^2 também serve.
>
> Mas seja lá quem for o polinômio mágico, eu sei que ou ele é um polinômio
> par ou ele é ímpar. Afinal, escreva p(x)=P(x)+I(x) onde P(x) tem apenas os
> termos de grau par e I(x) tem apenas os de grau ímpar.
>
> Ora, p(x)^2=(P^2+I^2)+2PI. Note que P^2+I^2 é um polinômio par e 2PI é
> ímpar.
>
> Mas a condição manda que p^2=p(x^2+1), que é uma função par. Então o termo
> 2PI não pode existir, isto é, P=0 ou I=0. Assim, p(x) é par ou ímpar. E
> x^2-x+1 não é um nem outro, então não funcionou...
>
> Então precisamos ainda mostrar que existe UM tal polinômio!
>
> Abraço,
>   Ralph
>
> P.S.: Tem certeza que o enunciado é esse mesmo? Não seria, sei lá,
> p(x^2+1)=(p(x))^2+1 ao invés?
> 2011/7/1 Johann Dirichlet 
>
>> Em 01/07/11, Johann Dirichlet escreveu:
>> > Em 30/06/11, marcone augusto araújo
>> > borges escreveu:
>> >>
>> >> 1) Se p é inteiro primo ímpar,mostre que o numerador da fração
>> >> 1+1/2+1/3+...1/(p-1) é um múltiplo de p.
>> >
>> > 1) Teorema de Wolstenholme, se não me engano...
>> >
>> > Bora lá, usar o velho truque das pontas de Gauss:
>> > 1/k+1/(p-k)=p/(k(p-k));
>> > assim sendo, temos um monte de frações p/(alguma coisa). Esta coisa
>> > não será múltipla de p em momento nenhum, logo nada aniquila este
>> > fator p.
>> >
>> >>
>> >> 2) Mostre que existem infinitos polinômios p(x) com coeficientes reais
>> >> tais
>> >> que p(x^2+1) = [p(x)]^2.
>>
>> É mais mole do que eu pensei!
>>
>> 1 - Se P e Q são soluções da equação acima, P*Q também será. Óbvio!
>> 2 - Um polinômio possível é x^2-x+1.
>> Como sei? Simples:
>>
>> Se L é um zero de P, então L^2+1 também será.
>> Se eu conseguir L=L^2+1, terei uma solução pronta!
>> Basta abrir o polinomio sem medo.
>>
>>
>> P.S.: saber todas as soluções me parece mais desgastante. Aplicando a
>> transformação T(L)=L^2+1 um numero finito de vezes, todos os
>> polinômios dos pontos fixos são soluções. A treta é saber se não
>> escapa nenhum (até porque muitos desses polinomios são fatoráveis, I
>> think so).
>>
>> >>
>> >> 3) Uma corda AB,de comprimento constante,desliza sobre uma
>> >> semicircunferência determinada por um diâmetro d.
>> >> Considere o triângulo cujos vértices são: o ponto médio da corda e as
>> >> projeções ortogonais dos seus extremos A e B
>> >> sobre o diâmetro d.Mostre que ,durante o deslizamento da corda,esse
>> >> triângulo é sempre isósceles e nunca muda de formato(i.é.,os ângulos 

Re: [obm-l] Problemas(polinomoi- ideias!)

2011-07-04 Por tôpico Johann Dirichlet
Puxa! Mas onde esta o erro da minha solução?

Anyway, inicialmente pensei em fatorar o dito polinomio.
Creio que ele seja mônico, abrindo a expressão geral o fator máximo é a^2=a.
Aí, escreve ele na forma deprodutos (x-a_i).. Basicamente, um lado
fica na forma
x^2+1-a_ i, e o outro como (x-a_ i)^2. Supóndo que as raízes são, em
alguma ordem, iguais, dá pra chegar em algum lugar.




Em 01/07/11, Ralph Teixeira escreveu:
> O raciocínio do Dirichlet mostra que basta achar UM polinômio (não
> constante) que tenha esta propriedade. Afinal, como ele mostrou, se p(x)
> serve, então q(x)=(p(x))^2 também serve.
>
> Mas seja lá quem for o polinômio mágico, eu sei que ou ele é um polinômio
> par ou ele é ímpar. Afinal, escreva p(x)=P(x)+I(x) onde P(x) tem apenas os
> termos de grau par e I(x) tem apenas os de grau ímpar.
>
> Ora, p(x)^2=(P^2+I^2)+2PI. Note que P^2+I^2 é um polinômio par e 2PI é
> ímpar.
>
> Mas a condição manda que p^2=p(x^2+1), que é uma função par. Então o termo
> 2PI não pode existir, isto é, P=0 ou I=0. Assim, p(x) é par ou ímpar. E
> x^2-x+1 não é um nem outro, então não funcionou...
>
> Então precisamos ainda mostrar que existe UM tal polinômio!
>
> Abraço,
>   Ralph
>
> P.S.: Tem certeza que o enunciado é esse mesmo? Não seria, sei lá,
> p(x^2+1)=(p(x))^2+1 ao invés?
> 2011/7/1 Johann Dirichlet 
>
>> Em 01/07/11, Johann Dirichlet escreveu:
>> > Em 30/06/11, marcone augusto araújo
>> > borges escreveu:
>> >>
>> >> 1) Se p é inteiro primo ímpar,mostre que o numerador da fração
>> >> 1+1/2+1/3+...1/(p-1) é um múltiplo de p.
>> >
>> > 1) Teorema de Wolstenholme, se não me engano...
>> >
>> > Bora lá, usar o velho truque das pontas de Gauss:
>> > 1/k+1/(p-k)=p/(k(p-k));
>> > assim sendo, temos um monte de frações p/(alguma coisa). Esta coisa
>> > não será múltipla de p em momento nenhum, logo nada aniquila este
>> > fator p.
>> >
>> >>
>> >> 2) Mostre que existem infinitos polinômios p(x) com coeficientes reais
>> >> tais
>> >> que p(x^2+1) = [p(x)]^2.
>>
>> É mais mole do que eu pensei!
>>
>> 1 - Se P e Q são soluções da equação acima, P*Q também será. Óbvio!
>> 2 - Um polinômio possível é x^2-x+1.
>> Como sei? Simples:
>>
>> Se L é um zero de P, então L^2+1 também será.
>> Se eu conseguir L=L^2+1, terei uma solução pronta!
>> Basta abrir o polinomio sem medo.
>>
>>
>> P.S.: saber todas as soluções me parece mais desgastante. Aplicando a
>> transformação T(L)=L^2+1 um numero finito de vezes, todos os
>> polinômios dos pontos fixos são soluções. A treta é saber se não
>> escapa nenhum (até porque muitos desses polinomios são fatoráveis, I
>> think so).
>>
>> >>
>> >> 3) Uma corda AB,de comprimento constante,desliza sobre uma
>> >> semicircunferência determinada por um diâmetro d.
>> >> Considere o triângulo cujos vértices são: o ponto médio da corda e as
>> >> projeções ortogonais dos seus extremos A e B
>> >> sobre o diâmetro d.Mostre que ,durante o deslizamento da corda,esse
>> >> triângulo é sempre isósceles e nunca muda de formato(i.é.,os ângulos do
>> >> triângulo são constantes)
>> >
>> > Faz um desenho!
>> > Diâmetro r;centro O, raio 1; corda AB, tamanho d, médio M; AB
>> > projetado em r dá XY.
>> >
>> > O triangulo AOB é obviamente isósceles.
>> > Os quadrilateros XOMA e YOMB são inscritíveis de diâmetros OA e OB
>> > respectivamente (angulos de 90 graus).
>> >
>> > Temos OXM=OAM=OBM=OYM, logo XMY é isosceles. E o angulo OBA depende
>> > unicamente de d.
>> >
>> > P.S.: duvido que os triangulos sejam todos congruentes. O angulo XOM
>> > define o tamanho de XM.
>> >
>> >>
>> >> Meus agradecimentos por qualquer esclarecimento.
>> >
>> >
>> > --
>> > /**/
>> > 神が祝福
>> >
>> > Torres
>> >
>>
>>
>> --
>> /**/
>> 神が祝福
>>
>> Torres
>>
>> =
>> Instru�ões para entrar na lista, sair da lista e usar a lista em
>> http://www.mat.puc-rio.br/~obmlistas/obm-l.html
>> =
>>
>


-- 
/**/
神が祝福

Torres

=
Instru��es para entrar na lista, sair da lista e usar a lista em
http://www.mat.puc-rio.br/~obmlistas/obm-l.html
=


Re: [obm-l] Problemas(ajuda)

2011-07-01 Por tôpico Ralph Teixeira
Melhorando aos poucos, ainda usando as ideias do Dirichlet: p(x) não pode
ser ímpar. Se fosse, 0 seria raiz. Mas então 0^2+1=1 seria raiz, e 1^2+1=2
seria raiz, e 2^2+1=5 seria raiz... e p(x) não pode ter infinitas raízes.
Então estamos à procura de um polinômio **par** p(x) tal que
p(x^2+1)=[p(x)]^2.

Aliás, esse raciocínio mostra que esse p(x) não pode ter nenhuma raiz real
-- se tiver uma raiz real x, terá infinitas, já que x^2+1>x para todo x
real.

(Por enquanto, fico com a terrível impressão de que tal polinômio não
existe... Alguém achou o dito cujo?)
2011/7/1 Ralph Teixeira 

> O raciocínio do Dirichlet mostra que basta achar UM polinômio (não
> constante) que tenha esta propriedade. Afinal, como ele mostrou, se p(x)
> serve, então q(x)=(p(x))^2 também serve.
>
> Mas seja lá quem for o polinômio mágico, eu sei que ou ele é um polinômio
> par ou ele é ímpar. Afinal, escreva p(x)=P(x)+I(x) onde P(x) tem apenas os
> termos de grau par e I(x) tem apenas os de grau ímpar.
>
> Ora, p(x)^2=(P^2+I^2)+2PI. Note que P^2+I^2 é um polinômio par e 2PI é
> ímpar.
>
> Mas a condição manda que p^2=p(x^2+1), que é uma função par. Então o termo
> 2PI não pode existir, isto é, P=0 ou I=0. Assim, p(x) é par ou ímpar. E
> x^2-x+1 não é um nem outro, então não funcionou...
>
> Então precisamos ainda mostrar que existe UM tal polinômio!
>
> Abraço,
>   Ralph
>
> P.S.: Tem certeza que o enunciado é esse mesmo? Não seria, sei lá,
> p(x^2+1)=(p(x))^2+1 ao invés?
> 2011/7/1 Johann Dirichlet 
>
>> Em 01/07/11, Johann Dirichlet escreveu:
>> > Em 30/06/11, marcone augusto araújo
>> > borges escreveu:
>> >>
>> >> 1) Se p é inteiro primo ímpar,mostre que o numerador da fração
>> >> 1+1/2+1/3+...1/(p-1) é um múltiplo de p.
>> >
>> > 1) Teorema de Wolstenholme, se não me engano...
>> >
>> > Bora lá, usar o velho truque das pontas de Gauss:
>> 1/k+1/(p-k)=p/(k(p-k));
>> > assim sendo, temos um monte de frações p/(alguma coisa). Esta coisa
>> > não será múltipla de p em momento nenhum, logo nada aniquila este
>> > fator p.
>> >
>> >>
>> >> 2) Mostre que existem infinitos polinômios p(x) com coeficientes reais
>> >> tais
>> >> que p(x^2+1) = [p(x)]^2.
>>
>> É mais mole do que eu pensei!
>>
>> 1 - Se P e Q são soluções da equação acima, P*Q também será. Óbvio!
>> 2 - Um polinômio possível é x^2-x+1.
>> Como sei? Simples:
>>
>> Se L é um zero de P, então L^2+1 também será.
>> Se eu conseguir L=L^2+1, terei uma solução pronta!
>> Basta abrir o polinomio sem medo.
>>
>>
>> P.S.: saber todas as soluções me parece mais desgastante. Aplicando a
>> transformação T(L)=L^2+1 um numero finito de vezes, todos os
>> polinômios dos pontos fixos são soluções. A treta é saber se não
>> escapa nenhum (até porque muitos desses polinomios são fatoráveis, I
>> think so).
>>
>> >>
>> >> 3) Uma corda AB,de comprimento constante,desliza sobre uma
>> >> semicircunferência determinada por um diâmetro d.
>> >> Considere o triângulo cujos vértices são: o ponto médio da corda e as
>> >> projeções ortogonais dos seus extremos A e B
>> >> sobre o diâmetro d.Mostre que ,durante o deslizamento da corda,esse
>> >> triângulo é sempre isósceles e nunca muda de formato(i.é.,os ângulos do
>> >> triângulo são constantes)
>> >
>> > Faz um desenho!
>> > Diâmetro r;centro O, raio 1; corda AB, tamanho d, médio M; AB
>> > projetado em r dá XY.
>> >
>> > O triangulo AOB é obviamente isósceles.
>> > Os quadrilateros XOMA e YOMB são inscritíveis de diâmetros OA e OB
>> > respectivamente (angulos de 90 graus).
>> >
>> > Temos OXM=OAM=OBM=OYM, logo XMY é isosceles. E o angulo OBA depende
>> > unicamente de d.
>> >
>> > P.S.: duvido que os triangulos sejam todos congruentes. O angulo XOM
>> > define o tamanho de XM.
>> >
>> >>
>> >> Meus agradecimentos por qualquer esclarecimento.
>> >
>> >
>> > --
>> > /**/
>> > 神が祝福
>> >
>> > Torres
>> >
>>
>>
>> --
>> /**/
>> 神が祝福
>>
>> Torres
>>
>> =
>> Instru�ões para entrar na lista, sair da lista e usar a lista em
>> http://www.mat.puc-rio.br/~obmlistas/obm-l.html
>> =
>>
>
>


Re: [obm-l] Problemas(ajuda)

2011-07-01 Por tôpico Ralph Teixeira
O raciocínio do Dirichlet mostra que basta achar UM polinômio (não
constante) que tenha esta propriedade. Afinal, como ele mostrou, se p(x)
serve, então q(x)=(p(x))^2 também serve.

Mas seja lá quem for o polinômio mágico, eu sei que ou ele é um polinômio
par ou ele é ímpar. Afinal, escreva p(x)=P(x)+I(x) onde P(x) tem apenas os
termos de grau par e I(x) tem apenas os de grau ímpar.

Ora, p(x)^2=(P^2+I^2)+2PI. Note que P^2+I^2 é um polinômio par e 2PI é
ímpar.

Mas a condição manda que p^2=p(x^2+1), que é uma função par. Então o termo
2PI não pode existir, isto é, P=0 ou I=0. Assim, p(x) é par ou ímpar. E
x^2-x+1 não é um nem outro, então não funcionou...

Então precisamos ainda mostrar que existe UM tal polinômio!

Abraço,
  Ralph

P.S.: Tem certeza que o enunciado é esse mesmo? Não seria, sei lá,
p(x^2+1)=(p(x))^2+1 ao invés?
2011/7/1 Johann Dirichlet 

> Em 01/07/11, Johann Dirichlet escreveu:
> > Em 30/06/11, marcone augusto araújo
> > borges escreveu:
> >>
> >> 1) Se p é inteiro primo ímpar,mostre que o numerador da fração
> >> 1+1/2+1/3+...1/(p-1) é um múltiplo de p.
> >
> > 1) Teorema de Wolstenholme, se não me engano...
> >
> > Bora lá, usar o velho truque das pontas de Gauss: 1/k+1/(p-k)=p/(k(p-k));
> > assim sendo, temos um monte de frações p/(alguma coisa). Esta coisa
> > não será múltipla de p em momento nenhum, logo nada aniquila este
> > fator p.
> >
> >>
> >> 2) Mostre que existem infinitos polinômios p(x) com coeficientes reais
> >> tais
> >> que p(x^2+1) = [p(x)]^2.
>
> É mais mole do que eu pensei!
>
> 1 - Se P e Q são soluções da equação acima, P*Q também será. Óbvio!
> 2 - Um polinômio possível é x^2-x+1.
> Como sei? Simples:
>
> Se L é um zero de P, então L^2+1 também será.
> Se eu conseguir L=L^2+1, terei uma solução pronta!
> Basta abrir o polinomio sem medo.
>
>
> P.S.: saber todas as soluções me parece mais desgastante. Aplicando a
> transformação T(L)=L^2+1 um numero finito de vezes, todos os
> polinômios dos pontos fixos são soluções. A treta é saber se não
> escapa nenhum (até porque muitos desses polinomios são fatoráveis, I
> think so).
>
> >>
> >> 3) Uma corda AB,de comprimento constante,desliza sobre uma
> >> semicircunferência determinada por um diâmetro d.
> >> Considere o triângulo cujos vértices são: o ponto médio da corda e as
> >> projeções ortogonais dos seus extremos A e B
> >> sobre o diâmetro d.Mostre que ,durante o deslizamento da corda,esse
> >> triângulo é sempre isósceles e nunca muda de formato(i.é.,os ângulos do
> >> triângulo são constantes)
> >
> > Faz um desenho!
> > Diâmetro r;centro O, raio 1; corda AB, tamanho d, médio M; AB
> > projetado em r dá XY.
> >
> > O triangulo AOB é obviamente isósceles.
> > Os quadrilateros XOMA e YOMB são inscritíveis de diâmetros OA e OB
> > respectivamente (angulos de 90 graus).
> >
> > Temos OXM=OAM=OBM=OYM, logo XMY é isosceles. E o angulo OBA depende
> > unicamente de d.
> >
> > P.S.: duvido que os triangulos sejam todos congruentes. O angulo XOM
> > define o tamanho de XM.
> >
> >>
> >> Meus agradecimentos por qualquer esclarecimento.
> >
> >
> > --
> > /**/
> > 神が祝福
> >
> > Torres
> >
>
>
> --
> /**/
> 神が祝福
>
> Torres
>
> =
> Instru�ões para entrar na lista, sair da lista e usar a lista em
> http://www.mat.puc-rio.br/~obmlistas/obm-l.html
> =
>


Re: [obm-l] Problemas(ajuda)

2011-07-01 Por tôpico Johann Dirichlet
Em 01/07/11, Johann Dirichlet escreveu:
> Em 30/06/11, marcone augusto araújo
> borges escreveu:
>>
>> 1) Se p é inteiro primo ímpar,mostre que o numerador da fração
>> 1+1/2+1/3+...1/(p-1) é um múltiplo de p.
>
> 1) Teorema de Wolstenholme, se não me engano...
>
> Bora lá, usar o velho truque das pontas de Gauss: 1/k+1/(p-k)=p/(k(p-k));
> assim sendo, temos um monte de frações p/(alguma coisa). Esta coisa
> não será múltipla de p em momento nenhum, logo nada aniquila este
> fator p.
>
>>
>> 2) Mostre que existem infinitos polinômios p(x) com coeficientes reais
>> tais
>> que p(x^2+1) = [p(x)]^2.

É mais mole do que eu pensei!

1 - Se P e Q são soluções da equação acima, P*Q também será. Óbvio!
2 - Um polinômio possível é x^2-x+1.
Como sei? Simples:

Se L é um zero de P, então L^2+1 também será.
Se eu conseguir L=L^2+1, terei uma solução pronta!
Basta abrir o polinomio sem medo.


P.S.: saber todas as soluções me parece mais desgastante. Aplicando a
transformação T(L)=L^2+1 um numero finito de vezes, todos os
polinômios dos pontos fixos são soluções. A treta é saber se não
escapa nenhum (até porque muitos desses polinomios são fatoráveis, I
think so).

>>
>> 3) Uma corda AB,de comprimento constante,desliza sobre uma
>> semicircunferência determinada por um diâmetro d.
>> Considere o triângulo cujos vértices são: o ponto médio da corda e as
>> projeções ortogonais dos seus extremos A e B
>> sobre o diâmetro d.Mostre que ,durante o deslizamento da corda,esse
>> triângulo é sempre isósceles e nunca muda de formato(i.é.,os ângulos do
>> triângulo são constantes)
>
> Faz um desenho!
> Diâmetro r;centro O, raio 1; corda AB, tamanho d, médio M; AB
> projetado em r dá XY.
>
> O triangulo AOB é obviamente isósceles.
> Os quadrilateros XOMA e YOMB são inscritíveis de diâmetros OA e OB
> respectivamente (angulos de 90 graus).
>
> Temos OXM=OAM=OBM=OYM, logo XMY é isosceles. E o angulo OBA depende
> unicamente de d.
>
> P.S.: duvido que os triangulos sejam todos congruentes. O angulo XOM
> define o tamanho de XM.
>
>>
>> Meus agradecimentos por qualquer esclarecimento. 
>> 
>
>
> --
> /**/
> 神が祝福
>
> Torres
>


-- 
/**/
神が祝福

Torres

=
Instru��es para entrar na lista, sair da lista e usar a lista em
http://www.mat.puc-rio.br/~obmlistas/obm-l.html
=


Re: [obm-l] Problemas(ajuda)

2011-07-01 Por tôpico Johann Dirichlet
Em 30/06/11, marcone augusto araújo
borges escreveu:
>
> 1) Se p é inteiro primo ímpar,mostre que o numerador da fração
> 1+1/2+1/3+...1/(p-1) é um múltiplo de p.

1) Teorema de Wolstenholme, se não me engano...

Bora lá, usar o velho truque das pontas de Gauss: 1/k+1/(p-k)=p/(k(p-k));
assim sendo, temos um monte de frações p/(alguma coisa). Esta coisa
não será múltipla de p em momento nenhum, logo nada aniquila este
fator p.

>
> 2) Mostre que existem infinitos polinômios p(x) com coeficientes reais tais
> que p(x^2+1) = [p(x)]^2.
>
> 3) Uma corda AB,de comprimento constante,desliza sobre uma
> semicircunferência determinada por um diâmetro d.
> Considere o triângulo cujos vértices são: o ponto médio da corda e as
> projeções ortogonais dos seus extremos A e B
> sobre o diâmetro d.Mostre que ,durante o deslizamento da corda,esse
> triângulo é sempre isósceles e nunca muda de formato(i.é.,os ângulos do
> triângulo são constantes)

Faz um desenho!
Diâmetro r;centro O, raio 1; corda AB, tamanho d, médio M; AB
projetado em r dá XY.

O triangulo AOB é obviamente isósceles.
Os quadrilateros XOMA e YOMB são inscritíveis de diâmetros OA e OB
respectivamente (angulos de 90 graus).

Temos OXM=OAM=OBM=OYM, logo XMY é isosceles. E o angulo OBA depende
unicamente de d.

P.S.: duvido que os triangulos sejam todos congruentes. O angulo XOM
define o tamanho de XM.

>
> Meus agradecimentos por qualquer esclarecimento.  
> 


-- 
/**/
神が祝福

Torres

=
Instru��es para entrar na lista, sair da lista e usar a lista em
http://www.mat.puc-rio.br/~obmlistas/obm-l.html
=


Re: [obm-l] Problemas de Geometria Plana

2010-04-21 Por tôpico Eduardo Wilner
No segundo problema, dimensionalmente pode-se descartar B), C) e D).
Compare
 a expressão da área do triângulo em função de p e r com aquela 
em 
função da altura e da hipotenusa  ( que no caso é 2R).

Abraços

Wilner

---
 Em dom, 18/4/10, adriano emidio 
 escreveu:

De: adriano emidio 

Assunto: [obm-l] Problemas de 
Geometria Plana
Para: obm-l@mat.puc-rio.br
Data: Domingo, 18 de 
Abril de 2010, 14:38

Não consigo resolver esses dois problemas, quer dizer 
encontrar uma resposta dentre as propostas.
O
 problema é:  Uma expressão que dá o lado do eneágono regular, em 
função das diagonais a, b e c, com a < b < c, é:
A)
 (c^2+b^2)/aB) cb/aC) (c^2-b^2)/aD) 
(c+b)^2/aE) (c-b)^2/a 
Apliquei o 
teorema de Ptolomeu em vários quadriláteros, mas um deles forneceu: L= 
ab/(a+c) e as relações: a^2 = bL+ L^2 , b^2 = ac+L^2 e c^2= bc+ L^2.
Segundo
 problema: Um triângulo retângulo de perímetro 2p está inscrito num 
círculo de raio R e circunscrito a um círculo de raio r. Uma expressão 
que dá a altura relativa à hipotenusa do triângulo, em termos de p, r e R
 é?
A) pr/RB) 
(p+r)/RC) R/prD) R/(p+r)E)
 2pr/R
Usei o fato de que o 
produto dos catetos deste triângulo é igual ao poduto da hipotenusa pela
 medida da altura relativa à mesma e não sai! quer dizer não encontro 
uma resposta!                      



  

Re: [obm-l] PROBLEMAS IDIOTAS!

2010-01-25 Por tôpico Rogerio Ponce
Ola' Jorge,
eu mesmo ja' havia enviado a seguinte solucao dos trens (em junho de 2004):

- Problema dos trens -

Se nosso trem estivesse parado , veriamos um trem em sentido oposto na taxa
media de 1 trem a cada 24 h.

Como estamos nos movimentando com a mesma velocidade media, mas em sentido
oposto, veremos um novo trem a cada 24h/2 = 12hs, em media.

Ja' que a viagem dura 6 dias inteiros, veriamos 12 trens, mas o ultimo
tambem nao vale, pois estaria saindo da estacao.

Portanto o total encontrado seria de 12-1=11 trens.


- Livros na estante -

E' razoavel que ambos estejam com a lombada para fora, e que o primeiro
livro esteja do lado esquerdo.

Portanto, entre a primeira folha do primeiro, e a primeira folha do segundo,
existem as outras 69 folhas do segundo livro, alem de sua contracapa, e a
capa do primeiro livro, ou seja, existem
69*2=138 paginas impressas, (fora a capa e a contracapa).


- Caramujo no muro -

Ao longo do primeiro dia, ele chega a 3 metros de altura, e durante a noite
ele escorrega.
Ao longo do segundo dia, ele chega a 4 metros de altura,...
...
Ao longo do decimo dia, ele chega a 12 metros de altura.

Portanto, o caramujo levou 9 dias inteiros e alguns quebrados.
Pode ser que o caramujo conte pros seus amigos que levou somente 9 dias, mas
eu prefiro arredondar isso para 10 dias.


[]'s
Rogerio Ponce





2010/1/23 Jorge Luis Rodrigues e Silva Luis 

> ... por motivo ignorado ainda se encontram em aberto na lista...
>
> Um trem sai diàriamente de A para B. O mesmo ocorre no sentido contrário. A
>
> distância é percorrida em 6 dias. Se um trem sair de A, quantos trens
> vindos
> de B ele irá ultrapassar antes de lá chegar?
> Resp: Nenhum trem, já que não passou à frente de nenhum deles...
>
> Mas, se vocês viajassem num desses trens, quantos encontrariam correndo em
> sentido oposto, não incluindo o que tivesse chegado quando eu partisse e o
> que tivesse partido quando eu chegasse?
> Resp: Inacreditavelmente, todos erram novamente...
>
> Dois volumes de uma obra estão ordenadamente na estante, sendo o 1º com 50
> folhas e o 2º com 70. Quantas páginas impressas estão entre a primeira
> folha
> do 1º volume e a primeira do 2º volume?  (Campeão de Erros!)
>
> Um caramujo resolve subir um muro de 12 metros de altura da seguinte
> maneira: durante o dia ele sobe 3 metros e durante a noite desce 2 metros.
> Sabendo-se que iniciou a subida da base, ao amanhecer do 1º dia, quantos
> dias gastará o caramujo para chegar ao topo?
> Pasmem! Todos são unânimes em afirmar que o caramujo chegará ao topo no
> décimo dia. Incrível, não!
>
>
> 
>


RE: [obm-l] PROBLEMAS IDIOTAS!

2010-01-25 Por tôpico Vitor Paschoal

Eu acho que no último problema, o dos caramujos, são dez dias( dez dias e nove 
noites) e não doze como o pessoal tá afirmando, desse jeito ele vai avançará 30 
metros nas manhãs e regredirá 18 metros nas noites, dando um deslocamento de 12 
metros.
 


From: jorgelrs1...@hotmail.com
To: obm-l@mat.puc-rio.br
Subject: [obm-l] PROBLEMAS IDIOTAS!
Date: Sat, 23 Jan 2010 18:31:21 +



Olá! Artur Steiner...Se o Ralph Teixeira desistir da administração da lista, 
seu nome será uma ótima indicação bem como o Paulo Santa Rita...Apesar de não 
chegar a ser um Instituto Pestalozzi, gostaria de discutir alguns problemas 
idiotas e suas resoluções estúpidas...
 
Um rei queria livrar-se de seu primeiro-ministro escrevendo "vá" nos dois 
papéis a serem sorteados. Entretanto,o ministro conseguiu ficar. Como? A 
resposta de autores consagrados é que o rei foi obrigado a ficar com o 
ministro, para não denunciar o que tinha feito. Ora, bolas! Se o ministro 
queimou um dos papéis sorteados sem olhar e o outro papel estava escrito "vá", 
subtende-se que o papel sorteado e queimado pelo ministro era o que estava 
escrito "fique". Logo, não havia necessidade do rei revelar sua farsa. 
Concordam!
 
Pasmem! Vejam abaixo outros problemas imbecis, que por motivo ignorado ainda se 
encontram em aberto na lista...
 
Um trem sai diàriamente de A para B. O mesmo ocorre no sentido contrário. A 
distância é percorrida em 6 dias. Se um trem sair de A, quantos trens vindos 
de B ele irá ultrapassar antes de lá chegar?
Resp: Nenhum trem, já que não passou à frente de nenhum deles...

Mas, se vocês viajassem num desses trens, quantos encontrariam correndo em 
sentido oposto, não incluindo o que tivesse chegado quando eu partisse e o 
que tivesse partido quando eu chegasse?
Resp: Inacreditavelmente, todos erram novamente...

Dois volumes de uma obra estão ordenadamente na estante, sendo o 1º com 50 
folhas e o 2º com 70. Quantas páginas impressas estão entre a primeira folha 
do 1º volume e a primeira do 2º volume?  (Campeão de Erros!)

Um caramujo resolve subir um muro de 12 metros de altura da seguinte 
maneira: durante o dia ele sobe 3 metros e durante a noite desce 2 metros. 
Sabendo-se que iniciou a subida da base, ao amanhecer do 1º dia, quantos 
dias gastará o caramujo para chegar ao topo?
Pasmem! Todos são unânimes em afirmar que o caramujo chegará ao topo no décimo 
dia. Incrível, não!

Afinal! Qual a irracionalidade em recorrer aos caprichos da moeda ao invés de 
preferir uma escolha livre entre duas equipes favoritas?
 
 
Divirtam-se!
 
 
 



Quer brincar com as suas fotos e fazer álbuns divertidos? Clique aqui e saiba 
como.   
_
Agora é fácil compartilhar fotos no Messenger: solte todas na janelinha. Veja 
como!
http://www.windowslive.com.br/public/tip.aspx/view/77?product=2&ocid=CRM-WindowsLive:dicaCompartilhamentoFotos:Tagline:WLCRM:On:WL:pt-BR:Messenger

RE: [obm-l] Problemas matematicos de meu cotidiano(que eu n consigo resolver)

2009-12-06 Por tôpico Giovane Ferreira

Prezados amigos da OBM

Gostaria de saber como faço para conseguir que o MEC avalie um material de 
apoio que eu desenvolvi na area de trigonometria.

Abraço 

Professor Giovane
 
> Date: Sat, 5 Dec 2009 20:50:24 -0200
> Subject: Re: [obm-l] Problemas matematicos de meu cotidiano(que eu n consigo 
> resolver)
> From: ralp...@gmail.com
> To: obm-l@mat.puc-rio.br
> 
> Oi, Thiago. Seja bem-vindo! Que bom primeiro E-mail!
> 
> O seu segundo problema tem uma solucao bem legal (mas um pouco
> misteriosa), e uma mais bracal (mas mais geral). Vamos a elas:
> 
> SOLUCAO 1: MAGICA E RAPIDA, MAS SOH SERVE EM UNS POUCOS PROBLEMAS
> Sejam A e B os numeros que "eu" tiro no D8 e D12 (e seja X=A+B), e
> seja Y o numero que "voce" tira no seu D20. Entao os possiveis
> resultados de nosso experimento eh o conjunto das ternas ordenadas
> (A,B,Y) onde A vai de 1 a 8, B de 1 a 12 e Y vai de 1 a 20. De fato,
> no total, sao 8.12.20=1920 possibilidades para o nosso experimento.
> 
> Note uma simetria interessante neste "espaco amostral": para cada
> possibilidade (A,B,Y) existe uma possibilidade igualmente provavel
> (9-A,13-B,21-Y), certo?
> 
> Bom, agora a chave eh voce perceber que eu ganho em EXATAMENTE UMA
> destas duas possibilidades! Afinal,"eu ganhar" com a segunda terna
> ordenada significa:
> 
> (9-A)+(13-B)>21-Y sse
> 22-A-B>21-Y sse
> A+B A+B<=Y
> (esta ultima equivalencia vem do fato de estarmos trabalhando apenas
> com inteiros, entao M 
> isto eh, GANHAR com (9-A,13-B,21-Y) significa PERDER ou EMPATAR com
> (A,B,Y); e vice-versa!
> 
> Preste atencao: eu separei todas as possibilidades em pares
> **igualmente provaveis**, onde eu ganho em uma e apenas uma das duas
> hipoteses do par. Em outras palavras: para cada situacao em que eu
> ganho, existe uma outra situacao **com exatamente a mesma
> probabilidade** onde eu nao ganho.
> 
> Entao Pr(eu ganho)=0.5=50% (!!).
> --//--
> Agora, qual a probabilidade de a gente empatar? O truque agora eh
> perceber que minha soma A+B eh um numero entre 2 e 20 (que NAO SAO
> igualmente provaveis, mas isto nao vai importar aqui). Finja que eu
> jah fiz o meu lancamento. Agora, qual a chance de voce "acertar"
> exatamente o meu numero com o seu d20? Ora, nao interessa o que eu
> tirei: a chance eh 1/20=5%.
> 
> Entao Pr(empate)=0.05=5% ()
> --//--
> Enfim, Pr(eu perco)=100%-50%-5%=45%.
> --//--
> Conclusao: neste jogo, prefiro d8+d12 do que d20. :)
> 
> SOLUCAO 2: NO BRACO MESMO, MAS RESOLVE QUALQUER PROBLEMA DESTE TIPO
> A pessoa ("eu") que joga os dados de 12 e de 8 vai rolar uma soma X.
> Esta soma X pode dar 2, 3, 4, ..., 20.
> 
> Agora, soh porque sao 19 valores, nao significa 1/19 para cada!
> Supondo que os dados sejam justos e independentes, temos 12.8=96
> maneiras IGUALMENTE PROVAVEIS de rola-los. Destas:
> 
> -- Apenas 1+1 dah soma 2, entao Pr(X=2)=1/96;
> -- Apenas 1+2 e 2+1 dao 3, entao Pr(X=3)=2/96;
> -- Apenas 1+3, 2+2 e 3+1 dao 4, entao Pr(X=4)=3/96;
> ...
> 
> Continuando este raciocinio, chegamos aa seguinte tabela de
> probabilidades para X:
> 
> a: 02 03 04 05 06 07 08 09 10 11 12 13 14 15 16 17 18 19 20
> Pr(X=a): 01 02 03 04 05 06 07 08 08 08 08 08 07 06 05 04 03 02 01
> (toda esta linha eh SOBRE 96; note como a soma destes numeros eh 96 de
> fato!)
> 
> (Obs.: a partir da soma 10 a probabilidade para de subir (jah que nao
> dah para tirar 9+1). Depois, do 14 em diante, ela cai; por exemplo,
> para 14 temos apenas as possibilidades 2+12, 3+13,..., 8+6, sao 7.)
> 
> Agora, se eu tirar 02, eu tenho 1/20 de chance de ganhar (voce teria
> de tirar 1); se eu tirar 03, eu tenho 2/20 de chance de ganhar (voce
> tiraria 1 ou 2); e assim por diante. Entao:
> 
> Pr(eu ganhar)=Pr(eu ganhar com meu 2)+Pr(eu ganhar com meu
> 3)+...+Pr(eu ganhar com meu 20)=
> =Pr(X=2).1/20+Pr(X=3).2/20+Pr(X=4).3/20+...+Pr(X=20).19/20=
> =(1.1+2.2+3.3+4.4+5.5+6.6+7.7+8.8+8.9+8.10+8.11+8.12+7.13+6.14+5.15+4.16+3.17+2.18+1.19)/(96.20)=960/(96.20)=50%
> 
> (se voce nao quer fazer esta conta no braco, vah comer mingau quente;
> assim, voce aprende a ir das beiradas para dentro, juntando os termos
> assim: 1.1+1.19; 2.2+2.18; etc...; esse eh o velho truque do Gauss
> quando ele ainda era um fedelho :) :) :) )
> 
> Analogamente:
> 
> Pr(EU 
> perder)=(1.18+2.17+3.16+4.15+5.14+6.13+7.12+8.11+8.10+8.9+8.8+8.7+7.6+6.5+5.4+4.3+3.2+2.1)/(96.20)=45%
> (e o truque do mingau funciona de novo para acelerar as contas!)
> 
> Enfim:
> 
> Pr(Empatar)=(1.1+2.1+3.1+...+4.1+3.1+2.1+1.1)/(96.20)=96/(96.20)=1/20=5%
> 
> (claro, porque a soma dos numeros daquela tabela tinha que ser 96)
> 
> Abraco,
> Ralph
> 
> 2009/1

Re: [obm-l] Problemas matematicos de meu cotidiano(que eu n consigo resolver)

2009-12-05 Por tôpico Ralph Teixeira
Oi, Thiago. Seja bem-vindo! Que bom primeiro E-mail!

O seu segundo problema tem uma solucao bem legal (mas um pouco
misteriosa), e uma mais bracal (mas mais geral). Vamos a elas:

SOLUCAO 1: MAGICA E RAPIDA, MAS SOH SERVE EM UNS POUCOS PROBLEMAS
Sejam A e B os numeros que "eu" tiro no D8 e D12 (e seja X=A+B), e
seja Y o numero que "voce" tira no seu D20. Entao os possiveis
resultados de nosso experimento eh o conjunto das ternas ordenadas
(A,B,Y) onde A vai de 1 a 8, B de 1 a 12 e Y vai de 1 a 20. De fato,
no total, sao 8.12.20=1920 possibilidades para o nosso experimento.

Note uma simetria interessante neste "espaco amostral": para cada
possibilidade (A,B,Y) existe uma possibilidade igualmente provavel
(9-A,13-B,21-Y), certo?

Bom, agora a chave eh voce perceber que eu ganho em EXATAMENTE UMA
destas duas possibilidades! Afinal,"eu ganhar" com a segunda terna
ordenada significa:

(9-A)+(13-B)>21-Y sse
22-A-B>21-Y sse
A+B:
> Bom, esse 'e o meu primeiro e-mail aqui na lista entao vou fazer uma
> apresentacaozinha (meus acentos nesse computador nao funcionam)... Eu me
> chamo Thiago, do nivel 2, do estado de SP, e acho que eh isso...
> Mas vamos direto ao assunto...
>
> Eu fui almocar e no restaurante que fui, tinham 3 geladeiras. A temperatura
> delas variava sempre entre -2 e -5 graus C. Sempre que ele chegava em mais
> ou menos -2,5 , ele acionava algum gerador que fazia com que a temperatura
> dele caisse a -5 graus. E sempre que alguem abria a geladeira, a temperatura
> caia at'e onde o gerador fosse ligado. Sabendo disso, eu propus a minha mae:
> Um garcom, que esta com pressa, precisa que a bebida do cliente gele o mais
> rapidamente. Quando ele bateu o olho, uma geladeira estava a -2,5 graus,
> outra em 3,5 graus e a outra em 5 graus. Qual geladeira ele devera por para
> que gele o mais rapidamente? (leve em considera'cao que ele devera abrir a
> geladeira para por o refrigerante e que a de -2,5 esta ligando o motor. Nao
> se sabe se a 3,5 esta subindo ou descendo).
> --
> Eu estava jogando dados com o meu irmao (ganha quem tira o maior numero), e
> nos temos aqueles dados de RPG: temos um dado de 20, um de 12 e o um de 8.
> Uma pessoa fica com um dado de 20 e outra fica com um de 12 e um de 8.
> Que dados nos teriamos mais chance de ganha? Antes eu pensei assim: O numero
> minimo do dado de 20 'e 1, e o numero minimo do de 12 e de 8 'e 2, entao o
> de 12 e 8 deve ser melhor. Mas depois eu pensei assim: A chance do de 20
> tirar 20 'e de 100/20 enquanto a do 12 e 8 'e de 100/98. Ja a chance do 20
> tirar 19 'e de 100/20, enquanto a do 12 e 8 'e de 100/49...
> Afinal, qual combinacao de dados que 'e melhor?
>
> Desde ja' agradeco
>
>
>
> 
> Windows 7: agora com recursos que economizam bateria. Clique para conhecer.

=
Instruções para entrar na lista, sair da lista e usar a lista em
http://www.mat.puc-rio.br/~obmlistas/obm-l.html
=


Re: [obm-l] Problemas de Sangaku

2009-05-15 Por tôpico Carlos Nehab

Oi, DENISSON

Desculpe-me pois, ululantemente, padeço do mesmo mal...

Por favor aguarde o fim de semana para postar a solução do sandaku 
proposto. 


Abraços,
Nehab


Denisson escreveu:
Nehab, é interessante como nunca acertam meu nome :) É Denisson, não 
Denilson hehehehe




2009/5/14 Carlos Nehab >


Oi, Santa Rita,

O problema do problema é efetivamente evitar o sistema que você
mencionou, que é do terceiro grau...
Aliás, os problemas de geometria ditos "quadráticos" são quase
sempre triviais. 
Os bons problemas, em 90% dos casos, são quase sempre "cubicos".


To atracado com o problema, tentando  uma soluao geométrica. 
Guenta  a mão Denilson... :-)


Abraços
Nehab

Paulo Santa Rita escreveu:

Ola Denisson e demais
colegas desta lista ... OBM-L,

Eu acho que voce queria dizer : "trace a reta BP e a reta PC", certo ?
Se for assim, a sua questao e  simples, pois, fazendo  AP=X,  e facil
ver que o triangulo ABP tem catetos 1 e X e o triangulo PCD tem
catetos 1 e 1-X. Isto implica que suas hipotenusas estao determinadas
pelo teorema de pitagoras. Ora, estas hipotenusas sao precisamente
dois dos lados do triangulo BPC, sendo o terceiro lado igual a 1.
Conhecemos portanto todos os lados dos tres triangulos  ( em funcao de
AP=X.).  Lancando mao da expressao :

R = ( (p(p-a)(p-b)(p-c) )^0.5 / p

onde "p" e o semi-perimetro do triangulo de lados "a", "b" e "c",
obtemos os raios Ri dos circulos inscritos nos triangulos. Deste como
conhecemos R1, R2 e R3 em funcao de X. Com as expressos dos raios (
funcao de X ) que obtivemos acima,  fazemos :

R1 = R2/2
R2/2 = R3/3

Caso o sistema acima admita uma ou varias solucoes, trata-se da
resposta a sua segunda pergunta. E logico que pode haver um caminho
mais elegante, partindo, por exemplo, de uma construcao geometrica
auxiliar. Fica a sugestao... Confesso, entretanto, que o que me levou
a lhe responder, apresentando uma solucao "forca bruta" como esta,
foi sobretudo outro motivo ...

O que sao estes "Problemas de  Sangaku" ? Nunca ouvi alguem falar sobre 
isso.

Um abraco a todos !
PSR, 51405091108

  

2009/5/14 Denisson  :
Cerca de seis anos atrás eu tive um professor que era fascinado pelos
problemas de sangaku. Alguns eu conseguia fazer, mas confesso que a maioria
ficava à espera da resposta na próxima semana. Vou mostrar um em particular
que eu não consegui fazer:

Considere um quadrado ABCD de lado 1 e tome um ponto P qualquer entre A e D.
Trace a reta BP e a reta BC. Inscreva um círculo em cada um dos 3 triângulos
ABP (de raio R2), BPC (de raio R3) e PCD (de raio R1). Encontre o raio dos 3
círculos em função da medida AP. Em particular, existe alguma posição do
ponto P sobre AD tal que R1:R2:R3 tome o valor 1:2:3?

Se não surgirem respostas posto a solução daqui a alguns dias.

--
Denisson


=
Instru��es para entrar na lista, sair da lista e usar a lista em
http://www.mat.puc-rio.br/~obmlistas/obm-l.html

=






--
Denisson





Re: [obm-l] Problemas de Sangaku

2009-05-15 Por tôpico Carlos Nehab
Puxa ! 

Que ótimo!  Terei fim de semana  
Graças a você, desatraquei :-)


Grande abraço,
Nehab

Paulo Santa Rita escreveu:

Ola Denisson, Nehab e demais
colegas desta lista ... OBM-L,

Pessoal, penso que e natural que esta convivencia informal que
cultivamos aqui inevitavelmente nos leva a desenvolver certa simpatia
por algumas pessoas... Estou seriamente preocupado com o nosso amigo
Nehab, pois, pelo que estou sabendo das ultimas mensagens, ele esta em
seu escritorio, atracado com um violento Sangaku. Alguem tem noticias
dele ?

Bom, amenidades a parte, vamos tentar domar este violento sangaku. Vou
usar a figura sugerida pelo Denisson, na mensagem abaixo.

Seja AP=X. Sejam tambem :

1)  T1, T2 e T3 os pontos de tangencia do circulo ( raio R2 ) inscrito
no traingulo ABP respectivamente nos lados AB, AP e PB
2)  Q1, Q2 e Q3 os pontos de tangencia do circulo ( raio R1) inscrito
no triangulo PCD respectivamente nos lados CD, DP e PC.

R3 e o raio do circulo inscrito no triangulo BPC

E facil ver que a area do triangulo BPC e sempre 1/2, independente do
valor de AP=X que escolhermos. Sabemos que esta area pode ser expressa
assim :

1/2 = R3 *( semi-perimetro do traingulo BPC )

E quem e o semi-perimetro do traingulo BPC ? Vejamos :

PB = PT3  + T3B = X - R2 + 1 - R2 = 1 + X -2*R2
PC = PQ3 +  Q3C = (1-X) - R1 + 1 - R1 = 2 - X - 2*R1
BC = 1

Logo : semi-perimetro = (4 -2*R1 - 2*R2)/2 = 2 - R1 - R2. E daqui segue :

R3*(2 - R1 - R2) = 1/2   => 2 - ( 1/2*R3 ) = R1 + R2

Ou seja, a relacao acima entre os 3 raios independe do valor AP=X que
escolhermos.

Agora vamos a pergunta dois :

Existe  X  tal que R1:R2:R3 = 1:2:3 ?

Se existir um tal X entao R1 + R2 + R3 / 6 = R3/3. Substituindo R1 +
R2 por 2 - ( 1/2*R3), ficara :

2 - (1/2*R3) + R3 = 2*R3   => R3 = (2 - raiz_quad(2) ) / 2
( a outra raiz nao serve por ser maior que 1 )

E facil ver que este valor so ocorre quando AP=X=0, reduzindo-se um
dos circulos a um ponto. Assim, nao existe X > 0 que torne  R1:R2:R3 =
1:2:3.

Um Abraco a todos !
PSR, 61505090B21





  

2009/5/14 Carlos Nehab 

Oi, Santa Rita,

O problema do problema é efetivamente evitar o sistema que você mencionou,
que é do terceiro grau...
Aliás, os problemas de geometria ditos "quadráticos" são quase sempre
triviais.
Os bons problemas, em 90% dos casos, são quase sempre "cubicos".

To atracado com o problema, tentando  uma soluao geométrica.  Guenta  a
mão Denilson... :-)

Abraços
Nehab




  

2009/5/14 Denisson :
Cerca de seis anos atrás eu tive um professor que era fascinado pelos
problemas de sangaku. Alguns eu conseguia fazer, mas confesso que a
maioria
ficava à espera da resposta na próxima semana. Vou mostrar um em
particular
que eu não consegui fazer:

Considere um quadrado ABCD de lado 1 e tome um ponto P qualquer entre A e
D.
Trace a reta BP e a reta PC. Inscreva um círculo em cada um dos 3
triângulos
ABP (de raio R2), BPC (de raio R3) e PCD (de raio R1). Encontre o raio dos
3
círculos em função da medida AP. Em particular, existe alguma posição do
ponto P sobre AD tal que R1:R2:R3 tome o valor 1:2:3?

Se não surgirem respostas posto a solução daqui a alguns dias.

--
Denisson



=
Instru��es para entrar na lista, sair da lista e usar a lista em
http://www.mat.puc-rio.br/~obmlistas/obm-l.html
=

  




Re: [obm-l] Problemas de Sangaku

2009-05-15 Por tôpico Paulo Santa Rita
Ola Denisson, Nehab e demais
colegas desta lista ... OBM-L,

Pessoal, penso que e natural que esta convivencia informal que
cultivamos aqui inevitavelmente nos leva a desenvolver certa simpatia
por algumas pessoas... Estou seriamente preocupado com o nosso amigo
Nehab, pois, pelo que estou sabendo das ultimas mensagens, ele esta em
seu escritorio, atracado com um violento Sangaku. Alguem tem noticias
dele ?

Bom, amenidades a parte, vamos tentar domar este violento sangaku. Vou
usar a figura sugerida pelo Denisson, na mensagem abaixo.

Seja AP=X. Sejam tambem :

1)  T1, T2 e T3 os pontos de tangencia do circulo ( raio R2 ) inscrito
no traingulo ABP respectivamente nos lados AB, AP e PB
2)  Q1, Q2 e Q3 os pontos de tangencia do circulo ( raio R1) inscrito
no triangulo PCD respectivamente nos lados CD, DP e PC.

R3 e o raio do circulo inscrito no triangulo BPC

E facil ver que a area do triangulo BPC e sempre 1/2, independente do
valor de AP=X que escolhermos. Sabemos que esta area pode ser expressa
assim :

1/2 = R3 *( semi-perimetro do traingulo BPC )

E quem e o semi-perimetro do traingulo BPC ? Vejamos :

PB = PT3  + T3B = X - R2 + 1 - R2 = 1 + X -2*R2
PC = PQ3 +  Q3C = (1-X) - R1 + 1 - R1 = 2 - X - 2*R1
BC = 1

Logo : semi-perimetro = (4 -2*R1 - 2*R2)/2 = 2 - R1 - R2. E daqui segue :

R3*(2 - R1 - R2) = 1/2   => 2 - ( 1/2*R3 ) = R1 + R2

Ou seja, a relacao acima entre os 3 raios independe do valor AP=X que
escolhermos.

Agora vamos a pergunta dois :

Existe  X  tal que R1:R2:R3 = 1:2:3 ?

Se existir um tal X entao R1 + R2 + R3 / 6 = R3/3. Substituindo R1 +
R2 por 2 - ( 1/2*R3), ficara :

2 - (1/2*R3) + R3 = 2*R3   => R3 = (2 - raiz_quad(2) ) / 2
( a outra raiz nao serve por ser maior que 1 )

E facil ver que este valor so ocorre quando AP=X=0, reduzindo-se um
dos circulos a um ponto. Assim, nao existe X > 0 que torne  R1:R2:R3 =
1:2:3.

Um Abraco a todos !
PSR, 61505090B21





> 2009/5/14 Carlos Nehab 
>
> Oi, Santa Rita,
>
> O problema do problema é efetivamente evitar o sistema que você mencionou,
> que é do terceiro grau...
> Aliás, os problemas de geometria ditos "quadráticos" são quase sempre
> triviais.
> Os bons problemas, em 90% dos casos, são quase sempre "cubicos".
>
> To atracado com o problema, tentando  uma soluao geométrica.  Guenta  a
> mão Denilson... :-)
>
> Abraços
> Nehab


> 2009/5/14 Denisson :
> Cerca de seis anos atrás eu tive um professor que era fascinado pelos
> problemas de sangaku. Alguns eu conseguia fazer, mas confesso que a
> maioria
> ficava à espera da resposta na próxima semana. Vou mostrar um em
> particular
> que eu não consegui fazer:
>
> Considere um quadrado ABCD de lado 1 e tome um ponto P qualquer entre A e
> D.
> Trace a reta BP e a reta PC. Inscreva um círculo em cada um dos 3
> triângulos
> ABP (de raio R2), BPC (de raio R3) e PCD (de raio R1). Encontre o raio dos
> 3
> círculos em função da medida AP. Em particular, existe alguma posição do
> ponto P sobre AD tal que R1:R2:R3 tome o valor 1:2:3?
>
> Se não surgirem respostas posto a solução daqui a alguns dias.
>
> --
> Denisson

=
Instru��es para entrar na lista, sair da lista e usar a lista em
http://www.mat.puc-rio.br/~obmlistas/obm-l.html
=


Re: [obm-l] Problemas de Sangaku

2009-05-14 Por tôpico Denisson
Nehab, é interessante como nunca acertam meu nome :) É Denisson, não
Denilson hehehehe



2009/5/14 Carlos Nehab 

>  Oi, Santa Rita,
>
> O problema do problema é efetivamente evitar o sistema que você mencionou,
> que é do terceiro grau...
> Aliás, os problemas de geometria ditos "quadráticos" são quase sempre
> triviais.
> Os bons problemas, em 90% dos casos, são quase sempre "cubicos".
>
> To atracado com o problema, tentando  uma soluao geométrica.  Guenta  a mão
> Denilson... :-)
>
> Abraços
> Nehab
>
> Paulo Santa Rita escreveu:
>
> Ola Denisson e demais
> colegas desta lista ... OBM-L,
>
> Eu acho que voce queria dizer : "trace a reta BP e a reta PC", certo ?
> Se for assim, a sua questao e  simples, pois, fazendo  AP=X,  e facil
> ver que o triangulo ABP tem catetos 1 e X e o triangulo PCD tem
> catetos 1 e 1-X. Isto implica que suas hipotenusas estao determinadas
> pelo teorema de pitagoras. Ora, estas hipotenusas sao precisamente
> dois dos lados do triangulo BPC, sendo o terceiro lado igual a 1.
> Conhecemos portanto todos os lados dos tres triangulos  ( em funcao de
> AP=X.).  Lancando mao da expressao :
>
> R = ( (p(p-a)(p-b)(p-c) )^0.5 / p
>
> onde "p" e o semi-perimetro do triangulo de lados "a", "b" e "c",
> obtemos os raios Ri dos circulos inscritos nos triangulos. Deste como
> conhecemos R1, R2 e R3 em funcao de X. Com as expressos dos raios (
> funcao de X ) que obtivemos acima,  fazemos :
>
> R1 = R2/2
> R2/2 = R3/3
>
> Caso o sistema acima admita uma ou varias solucoes, trata-se da
> resposta a sua segunda pergunta. E logico que pode haver um caminho
> mais elegante, partindo, por exemplo, de uma construcao geometrica
> auxiliar. Fica a sugestao... Confesso, entretanto, que o que me levou
> a lhe responder, apresentando uma solucao "forca bruta" como esta,
> foi sobretudo outro motivo ...
>
> O que sao estes "Problemas de  Sangaku" ? Nunca ouvi alguem falar sobre isso.
>
> Um abraco a todos !
> PSR, 51405091108
>
>
>
>  2009/5/14 Denisson  :
> Cerca de seis anos atrás eu tive um professor que era fascinado pelos
> problemas de sangaku. Alguns eu conseguia fazer, mas confesso que a maioria
> ficava à espera da resposta na próxima semana. Vou mostrar um em particular
> que eu não consegui fazer:
>
> Considere um quadrado ABCD de lado 1 e tome um ponto P qualquer entre A e D.
> Trace a reta BP e a reta BC. Inscreva um círculo em cada um dos 3 triângulos
> ABP (de raio R2), BPC (de raio R3) e PCD (de raio R1). Encontre o raio dos 3
> círculos em função da medida AP. Em particular, existe alguma posição do
> ponto P sobre AD tal que R1:R2:R3 tome o valor 1:2:3?
>
> Se não surgirem respostas posto a solução daqui a alguns dias.
>
> --
> Denisson
>
>
>  =
> Instru��es para entrar na lista, sair da lista e usar a lista 
> emhttp://www.mat.puc-rio.br/~obmlistas/obm-l.html 
> 
> =
>
>
>
>
>


-- 
Denisson


Re: [obm-l] Problemas de Sangaku

2009-05-14 Por tôpico Carlos Nehab

Oi, Santa Rita,

O problema do problema é efetivamente evitar o sistema que você 
mencionou, que é do terceiro grau...
Aliás, os problemas de geometria ditos "quadráticos" são quase sempre 
triviais. 
Os bons problemas, em 90% dos casos, são quase sempre "cubicos".


To atracado com o problema, tentando  uma soluao geométrica.  Guenta  a 
mão Denilson... :-)


Abraços
Nehab

Paulo Santa Rita escreveu:

Ola Denisson e demais
colegas desta lista ... OBM-L,

Eu acho que voce queria dizer : "trace a reta BP e a reta PC", certo ?
Se for assim, a sua questao e  simples, pois, fazendo  AP=X,  e facil
ver que o triangulo ABP tem catetos 1 e X e o triangulo PCD tem
catetos 1 e 1-X. Isto implica que suas hipotenusas estao determinadas
pelo teorema de pitagoras. Ora, estas hipotenusas sao precisamente
dois dos lados do triangulo BPC, sendo o terceiro lado igual a 1.
Conhecemos portanto todos os lados dos tres triangulos  ( em funcao de
AP=X.).  Lancando mao da expressao :

R = ( (p(p-a)(p-b)(p-c) )^0.5 / p

onde "p" e o semi-perimetro do triangulo de lados "a", "b" e "c",
obtemos os raios Ri dos circulos inscritos nos triangulos. Deste como
conhecemos R1, R2 e R3 em funcao de X. Com as expressos dos raios (
funcao de X ) que obtivemos acima,  fazemos :

R1 = R2/2
R2/2 = R3/3

Caso o sistema acima admita uma ou varias solucoes, trata-se da
resposta a sua segunda pergunta. E logico que pode haver um caminho
mais elegante, partindo, por exemplo, de uma construcao geometrica
auxiliar. Fica a sugestao... Confesso, entretanto, que o que me levou
a lhe responder, apresentando uma solucao "forca bruta" como esta,
foi sobretudo outro motivo ...

O que sao estes "Problemas de  Sangaku" ? Nunca ouvi alguem falar sobre isso.

Um abraco a todos !
PSR, 51405091108

  

2009/5/14 Denisson :
Cerca de seis anos atrás eu tive um professor que era fascinado pelos
problemas de sangaku. Alguns eu conseguia fazer, mas confesso que a maioria
ficava à espera da resposta na próxima semana. Vou mostrar um em particular
que eu não consegui fazer:

Considere um quadrado ABCD de lado 1 e tome um ponto P qualquer entre A e D.
Trace a reta BP e a reta BC. Inscreva um círculo em cada um dos 3 triângulos
ABP (de raio R2), BPC (de raio R3) e PCD (de raio R1). Encontre o raio dos 3
círculos em função da medida AP. Em particular, existe alguma posição do
ponto P sobre AD tal que R1:R2:R3 tome o valor 1:2:3?

Se não surgirem respostas posto a solução daqui a alguns dias.

--
Denisson



=
Instru��es para entrar na lista, sair da lista e usar a lista em
http://www.mat.puc-rio.br/~obmlistas/obm-l.html
=

  




Re: [obm-l] Problemas de Sangaku

2009-05-14 Por tôpico Denisson
Estou quase um spammer :P

Bem, no ensino médio um professor sempre trazia esses problemas. E o
objetivo era sempre achar a solução mais simples, em geral traçando alguma
reta auxiliar ou traçando circulos. Bem, eu acho eles legais :) Dá uma
olhada lá pra ver se te interessa também.

2009/5/14 Denisson 

> De fato, a única dificuldade nessa questão são as contas. Mas o objetivo
> era mostrar os problemas de sangaku mesmo que por sinal achei que eram bem
> conhecidos. No link do email anterior tem explicações sobre suas origens.
> Existem outros problemas de sangaku e alguns deles tem um grau de
> dificuldade bem grande... É um bom exercício em geral :)
>
>
>
>
> 2009/5/14 Denisson 
>
>> http://www.rpm.org.br/conheca/49/1/sangaku.htm
>>
>> 2009/5/14 Paulo Santa Rita 
>>
>> Ola Denisson e demais
>>> colegas desta lista ... OBM-L,
>>>
>>> Eu acho que voce queria dizer : "trace a reta BP e a reta PC", certo ?
>>> Se for assim, a sua questao e  simples, pois, fazendo  AP=X,  e facil
>>> ver que o triangulo ABP tem catetos 1 e X e o triangulo PCD tem
>>> catetos 1 e 1-X. Isto implica que suas hipotenusas estao determinadas
>>> pelo teorema de pitagoras. Ora, estas hipotenusas sao precisamente
>>> dois dos lados do triangulo BPC, sendo o terceiro lado igual a 1.
>>> Conhecemos portanto todos os lados dos tres triangulos  ( em funcao de
>>> AP=X.).  Lancando mao da expressao :
>>>
>>> R = ( (p(p-a)(p-b)(p-c) )^0.5 / p
>>>
>>> onde "p" e o semi-perimetro do triangulo de lados "a", "b" e "c",
>>> obtemos os raios Ri dos circulos inscritos nos triangulos. Deste como
>>> conhecemos R1, R2 e R3 em funcao de X. Com as expressos dos raios (
>>> funcao de X ) que obtivemos acima,  fazemos :
>>>
>>> R1 = R2/2
>>> R2/2 = R3/3
>>>
>>> Caso o sistema acima admita uma ou varias solucoes, trata-se da
>>> resposta a sua segunda pergunta. E logico que pode haver um caminho
>>> mais elegante, partindo, por exemplo, de uma construcao geometrica
>>> auxiliar. Fica a sugestao... Confesso, entretanto, que o que me levou
>>> a lhe responder, apresentando uma solucao "forca bruta" como esta,
>>> foi sobretudo outro motivo ...
>>>
>>> O que sao estes "Problemas de  Sangaku" ? Nunca ouvi alguem falar sobre
>>> isso.
>>>
>>> Um abraco a todos !
>>> PSR, 51405091108
>>>
>>> > 2009/5/14 Denisson :
>>> > Cerca de seis anos atrás eu tive um professor que era fascinado pelos
>>> > problemas de sangaku. Alguns eu conseguia fazer, mas confesso que a
>>> maioria
>>> > ficava à espera da resposta na próxima semana. Vou mostrar um em
>>> particular
>>> > que eu não consegui fazer:
>>> >
>>> > Considere um quadrado ABCD de lado 1 e tome um ponto P qualquer entre A
>>> e D.
>>> > Trace a reta BP e a reta BC. Inscreva um círculo em cada um dos 3
>>> triângulos
>>> > ABP (de raio R2), BPC (de raio R3) e PCD (de raio R1). Encontre o raio
>>> dos 3
>>> > círculos em função da medida AP. Em particular, existe alguma posição
>>> do
>>> > ponto P sobre AD tal que R1:R2:R3 tome o valor 1:2:3?
>>> >
>>> > Se não surgirem respostas posto a solução daqui a alguns dias.
>>> >
>>> > --
>>> > Denisson
>>>
>>> =
>>> Instruções para entrar na lista, sair da lista e usar a lista em
>>> http://www.mat.puc-rio.br/~obmlistas/obm-l.html
>>> =
>>>
>>
>>
>>
>> --
>> Denisson
>>
>>
>
>
> --
> Denisson
>
>


-- 
Denisson


Re: [obm-l] Problemas de Sangaku

2009-05-14 Por tôpico Denisson
De fato, a única dificuldade nessa questão são as contas. Mas o objetivo era
mostrar os problemas de sangaku mesmo que por sinal achei que eram bem
conhecidos. No link do email anterior tem explicações sobre suas origens.
Existem outros problemas de sangaku e alguns deles tem um grau de
dificuldade bem grande... É um bom exercício em geral :)



2009/5/14 Denisson 

> http://www.rpm.org.br/conheca/49/1/sangaku.htm
>
> 2009/5/14 Paulo Santa Rita 
>
> Ola Denisson e demais
>> colegas desta lista ... OBM-L,
>>
>> Eu acho que voce queria dizer : "trace a reta BP e a reta PC", certo ?
>> Se for assim, a sua questao e  simples, pois, fazendo  AP=X,  e facil
>> ver que o triangulo ABP tem catetos 1 e X e o triangulo PCD tem
>> catetos 1 e 1-X. Isto implica que suas hipotenusas estao determinadas
>> pelo teorema de pitagoras. Ora, estas hipotenusas sao precisamente
>> dois dos lados do triangulo BPC, sendo o terceiro lado igual a 1.
>> Conhecemos portanto todos os lados dos tres triangulos  ( em funcao de
>> AP=X.).  Lancando mao da expressao :
>>
>> R = ( (p(p-a)(p-b)(p-c) )^0.5 / p
>>
>> onde "p" e o semi-perimetro do triangulo de lados "a", "b" e "c",
>> obtemos os raios Ri dos circulos inscritos nos triangulos. Deste como
>> conhecemos R1, R2 e R3 em funcao de X. Com as expressos dos raios (
>> funcao de X ) que obtivemos acima,  fazemos :
>>
>> R1 = R2/2
>> R2/2 = R3/3
>>
>> Caso o sistema acima admita uma ou varias solucoes, trata-se da
>> resposta a sua segunda pergunta. E logico que pode haver um caminho
>> mais elegante, partindo, por exemplo, de uma construcao geometrica
>> auxiliar. Fica a sugestao... Confesso, entretanto, que o que me levou
>> a lhe responder, apresentando uma solucao "forca bruta" como esta,
>> foi sobretudo outro motivo ...
>>
>> O que sao estes "Problemas de  Sangaku" ? Nunca ouvi alguem falar sobre
>> isso.
>>
>> Um abraco a todos !
>> PSR, 51405091108
>>
>> > 2009/5/14 Denisson :
>> > Cerca de seis anos atrás eu tive um professor que era fascinado pelos
>> > problemas de sangaku. Alguns eu conseguia fazer, mas confesso que a
>> maioria
>> > ficava à espera da resposta na próxima semana. Vou mostrar um em
>> particular
>> > que eu não consegui fazer:
>> >
>> > Considere um quadrado ABCD de lado 1 e tome um ponto P qualquer entre A
>> e D.
>> > Trace a reta BP e a reta BC. Inscreva um círculo em cada um dos 3
>> triângulos
>> > ABP (de raio R2), BPC (de raio R3) e PCD (de raio R1). Encontre o raio
>> dos 3
>> > círculos em função da medida AP. Em particular, existe alguma posição do
>> > ponto P sobre AD tal que R1:R2:R3 tome o valor 1:2:3?
>> >
>> > Se não surgirem respostas posto a solução daqui a alguns dias.
>> >
>> > --
>> > Denisson
>>
>> =
>> Instruções para entrar na lista, sair da lista e usar a lista em
>> http://www.mat.puc-rio.br/~obmlistas/obm-l.html
>> =
>>
>
>
>
> --
> Denisson
>
>


-- 
Denisson


Re: [obm-l] Problemas de Sangaku

2009-05-14 Por tôpico Denisson
http://www.rpm.org.br/conheca/49/1/sangaku.htm

2009/5/14 Paulo Santa Rita 

> Ola Denisson e demais
> colegas desta lista ... OBM-L,
>
> Eu acho que voce queria dizer : "trace a reta BP e a reta PC", certo ?
> Se for assim, a sua questao e  simples, pois, fazendo  AP=X,  e facil
> ver que o triangulo ABP tem catetos 1 e X e o triangulo PCD tem
> catetos 1 e 1-X. Isto implica que suas hipotenusas estao determinadas
> pelo teorema de pitagoras. Ora, estas hipotenusas sao precisamente
> dois dos lados do triangulo BPC, sendo o terceiro lado igual a 1.
> Conhecemos portanto todos os lados dos tres triangulos  ( em funcao de
> AP=X.).  Lancando mao da expressao :
>
> R = ( (p(p-a)(p-b)(p-c) )^0.5 / p
>
> onde "p" e o semi-perimetro do triangulo de lados "a", "b" e "c",
> obtemos os raios Ri dos circulos inscritos nos triangulos. Deste como
> conhecemos R1, R2 e R3 em funcao de X. Com as expressos dos raios (
> funcao de X ) que obtivemos acima,  fazemos :
>
> R1 = R2/2
> R2/2 = R3/3
>
> Caso o sistema acima admita uma ou varias solucoes, trata-se da
> resposta a sua segunda pergunta. E logico que pode haver um caminho
> mais elegante, partindo, por exemplo, de uma construcao geometrica
> auxiliar. Fica a sugestao... Confesso, entretanto, que o que me levou
> a lhe responder, apresentando uma solucao "forca bruta" como esta,
> foi sobretudo outro motivo ...
>
> O que sao estes "Problemas de  Sangaku" ? Nunca ouvi alguem falar sobre
> isso.
>
> Um abraco a todos !
> PSR, 51405091108
>
> > 2009/5/14 Denisson :
> > Cerca de seis anos atrás eu tive um professor que era fascinado pelos
> > problemas de sangaku. Alguns eu conseguia fazer, mas confesso que a
> maioria
> > ficava à espera da resposta na próxima semana. Vou mostrar um em
> particular
> > que eu não consegui fazer:
> >
> > Considere um quadrado ABCD de lado 1 e tome um ponto P qualquer entre A e
> D.
> > Trace a reta BP e a reta BC. Inscreva um círculo em cada um dos 3
> triângulos
> > ABP (de raio R2), BPC (de raio R3) e PCD (de raio R1). Encontre o raio
> dos 3
> > círculos em função da medida AP. Em particular, existe alguma posição do
> > ponto P sobre AD tal que R1:R2:R3 tome o valor 1:2:3?
> >
> > Se não surgirem respostas posto a solução daqui a alguns dias.
> >
> > --
> > Denisson
>
> =
> Instruções para entrar na lista, sair da lista e usar a lista em
> http://www.mat.puc-rio.br/~obmlistas/obm-l.html
> =
>



-- 
Denisson


Re: [obm-l] Problemas de Sangaku

2009-05-14 Por tôpico Paulo Santa Rita
Ola Denisson e demais
colegas desta lista ... OBM-L,

Eu acho que voce queria dizer : "trace a reta BP e a reta PC", certo ?
Se for assim, a sua questao e  simples, pois, fazendo  AP=X,  e facil
ver que o triangulo ABP tem catetos 1 e X e o triangulo PCD tem
catetos 1 e 1-X. Isto implica que suas hipotenusas estao determinadas
pelo teorema de pitagoras. Ora, estas hipotenusas sao precisamente
dois dos lados do triangulo BPC, sendo o terceiro lado igual a 1.
Conhecemos portanto todos os lados dos tres triangulos  ( em funcao de
AP=X.).  Lancando mao da expressao :

R = ( (p(p-a)(p-b)(p-c) )^0.5 / p

onde "p" e o semi-perimetro do triangulo de lados "a", "b" e "c",
obtemos os raios Ri dos circulos inscritos nos triangulos. Deste como
conhecemos R1, R2 e R3 em funcao de X. Com as expressos dos raios (
funcao de X ) que obtivemos acima,  fazemos :

R1 = R2/2
R2/2 = R3/3

Caso o sistema acima admita uma ou varias solucoes, trata-se da
resposta a sua segunda pergunta. E logico que pode haver um caminho
mais elegante, partindo, por exemplo, de uma construcao geometrica
auxiliar. Fica a sugestao... Confesso, entretanto, que o que me levou
a lhe responder, apresentando uma solucao "forca bruta" como esta,
foi sobretudo outro motivo ...

O que sao estes "Problemas de  Sangaku" ? Nunca ouvi alguem falar sobre isso.

Um abraco a todos !
PSR, 51405091108

> 2009/5/14 Denisson :
> Cerca de seis anos atrás eu tive um professor que era fascinado pelos
> problemas de sangaku. Alguns eu conseguia fazer, mas confesso que a maioria
> ficava à espera da resposta na próxima semana. Vou mostrar um em particular
> que eu não consegui fazer:
>
> Considere um quadrado ABCD de lado 1 e tome um ponto P qualquer entre A e D.
> Trace a reta BP e a reta BC. Inscreva um círculo em cada um dos 3 triângulos
> ABP (de raio R2), BPC (de raio R3) e PCD (de raio R1). Encontre o raio dos 3
> círculos em função da medida AP. Em particular, existe alguma posição do
> ponto P sobre AD tal que R1:R2:R3 tome o valor 1:2:3?
>
> Se não surgirem respostas posto a solução daqui a alguns dias.
>
> --
> Denisson

=
Instru��es para entrar na lista, sair da lista e usar a lista em
http://www.mat.puc-rio.br/~obmlistas/obm-l.html
=


Re: [obm-l] Problemas interessantes de coloração

2008-07-28 Por tôpico Rogerio Ponce
Ola' Otavio e colegas da lista,
sera' que alguem teria uma solucao diferente para o problema 4?

1)
Considere um triangulo isosceles ABC , de base unitaria AB e lados
iguais a sqrt(3) (ou raiz quadrada de 3).
Se os vertices A e B forem da mesma cor, terminamos aqui.
Caso eles tenham cores diferentes, entao pelo menos um deles tem cor
diferente da cor de C.
Sem perda de generalidade, suponhamos que este seja o vertice A.
Seja M o ponto medio de AC. Passando por M, trace o segmento unitario
DE, ortogonal 'a AC, de forma que M tambem seja ponto medio de DE.
Repare que ADE , e CDE formam 2 triangulos equilateros unitarios.
Caso os vertices D e E tenham cores iguais, terminamos aqui. Caso
tenham cores diferentes, um deles tera' a mesma cor que A ou C, pois
sao 4 vertices (lembre-se que A e C tem cores diferentes) e existem
apenas 3 cores.
Logo, ha' dois pontos com a mesma cor, situados a 1 unidade um do outro.

2)
Qualquer que seja a distancia D, sempre podemos construir um triangulo
equilatero de lado D, que necessariamente tera' pelo menos 2 vertices
da mesma cor.

3)
Divida o plano em duas regioes (superior e inferior) com uma reta
horizontal, e marque 3 pontos sobre ela.
Necessariamente pelo menos dois desses pontos serao da mesma cor.
Da esquerda para a direita, chame-os de "A" e "B" .
Seja "L" a distancia entre eles,  "X" a cor de ambos, e "Y" a outra
cor utilizada para pintar o plano.
Ainda sobre essa reta, marque os pontos "C" e "D", ambos 'a direita de
"B" , tal que BC = L e CD = L.
Marque, na parte superior do plano, os vertices superiores E, F e G
dos respectivos triangulos equilateros com bases AB, BC e CD.
Marque, na parte inferior do plano, os vertices inferiores H e I dos
respectivos triangulos equilateros com bases AB e BC.

Se  cor(E)=X , teriamos o triangulo equilatero ABE com seus vertices
da cor X, e a demonstracao terminou. Entao, suponhamos cor(E)=Y.
Da mesma forma, se cor(H)=X , terminamos. Entao, suponhamos cor(H)=Y.
Repare que os pontos E, H e C formam um triangulo equilatero.
Portanto, se cor(C)=Y, terminariamos aqui. Entao, suponhamos cor(C)=X.
Mas, se cor(F)=X ou cor(I)=X, teriamos um triangulo equilatero (BCF ou
BCI) na cor X, e terminariamos a prova. Assim, suponhamos cor(F)=Y e
cor(I )=Y.
Como os pontos F, I e D formam um triangulo equilatero, se cor(D)=Y,
terminariamos aqui.
Entao, suponhamos cor(D)=X.
Logo, se cor(G)=X teriamos o triangulo equilatero GCD com vertices da
mesma cor, e terminariamos. Assim, suponhamos cor(G)=Y.
E neste ponto, verificamos que os pontos E, I e G , que formam um
triangulo equilatero, teriam a mesma cor Y.
Portanto, sempre e' possivel contruirmos um triangulo equilatero com
vertices da mesma cor.

4)
Sejam as cores A e B.
Seja L = (raiz quadrada de 3) / 2
Trace um eixo horizontal (X) de coordenadas.
Basta pintar o plano com faixas verticais alternando-se as cores, da
seguinte forma:
Pinte o intervalo [0, L) com a cor A.  Repare que o intervalo e'
fechado em 0 e aberto em L.
Pinte o intervalo [L , 2L) com a cor B.
Replique isso, indefinidamente, para os dois lados.

5)
Corte 3 linhas paralelas (horizontais) com 9 paralelas verticais,
determinando 9 grupos de 3 pontos alinhados verticalmente.
Cada grupo de 3 pontos tera' necessariamente 2 pontos com a mesma cor.
Como existem somente 2*2*2 = 8 formas diferentes de se pintar um grupo
de 3 pontos com 2 cores, havera' pelo menos dois grupos com a mesma
pintura entre os 9 grupos.
Os pontos com repeticao de cor em cada um desses 2 grupos determinam
os vertices do retangulo procurado.

[]'s
Rogerio Ponce



2008/7/24 Otávio Menezes <[EMAIL PROTECTED]>:
> 1) Pinte o plano com três cores. Prove que há dois pontos com a mesma cor
> situados a exatamente 1 unidade um do outro.
> 2) Pinte o plano com duas cores. Prove que uma dessas cores contém pares de
> pontos a qualquer distância entre si.
> 3) Pinte o plano com duas cores. Prove que existe um triângulo equilátero
> com todos os vértices da mesma cor.
> 4) Mostre que é possível colorir o plano em duas cores de modo que não
> exista um triâmgulo equilátero de lado 1 com todos os vértices da mesma cor.
> 5) Pinte o plano em duas cores. Mostre que existe um retângulo com todos os
> vértice da mesma cor.
>
> Os dois primeiros são muito fáceis, os outros são mais complicados.
>

=
Instruções para entrar na lista, sair da lista e usar a lista em
http://www.mat.puc-rio.br/~obmlistas/obm-l.html
=


Re: [obm-l] Problemas interessantes de coloração

2008-07-28 Por tôpico Rafael Ando
Hm, verdade, nao tinha pensado nisso 0_o

e a solucao do Igor pra questao 4? Se eu fizer cada listra com espessura
sqrt(3)/2 (tem que ser sqrt(3)/2, outro valor nao da certo... eh a altura de
um triangulo equilatero, e se o valor for diferente desse da pra colocar o
triangulo com um dos seus lados na vertical), definindo que cada listra
contem os pontos a sua esquerda (em outras palavras, no plano xy, eu pinto o
ponto (x,y) de azul se o piso de 2x/sqrt(3) for par, e de vermelho caso
contrario), acho que nao tem triangulo equilatero de lado 1 mas posso
estar enganado, nao fiquei pensando muito =/

2008/7/26 Ralph Teixeira <[EMAIL PROTECTED]>

> Mas nao precisa ser o triangulo todo da mesma cor -- bastam os VERTICES
> :)
>
> 2008/7/25 Igor Battazza <[EMAIL PROTECTED]>:
>
> Tambem nao sei se entendi, pois o problema nao diz nada sobre
>> restriçoes a respeito das cores... Se nao tiver restriçoes, na 4),
>> acho que posso colorir o plano em listras alternadas com 2 cores, azul
>> e vermelho por exemplo, de maneira que a espessura de cada listra seja
>> menor do que 1 unidade (1/2 unidade por ex.).
>>
>> 2008/7/25 Rafael Ando <[EMAIL PROTECTED]>:
>>  > Nao sei se entendi direito o 3 e o 5, mas o que me impede de fazer o
>> > seguinte:
>> >
>> > Sejam azul e vermelho as duas cores. Seja A um ponto azul. Entao seja
>> d>0 a
>> > distancia minima de A ate qualquer ponto vermelho. Entao todos o pontos
>> da
>> > circunferencia de centro A e raio r> > triangulo equilatero inscrito nessa circunferencia resolveriam o
>> > problema
>> >
>> > 2008/7/24 Otávio Menezes <[EMAIL PROTECTED]>:
>> >>
>> >> 1) Pinte o plano com três cores. Prove que há dois pontos com a mesma
>> cor
>> >> situados a exatamente 1 unidade um do outro.
>> >> 2) Pinte o plano com duas cores. Prove que uma dessas cores contém
>> pares
>> >> de pontos a qualquer distância entre si.
>> >> 3) Pinte o plano com duas cores. Prove que existe um triângulo
>> equilátero
>> >> com todos os vértices da mesma cor.
>> >> 4) Mostre que é possível colorir o plano em duas cores de modo que não
>> >> exista um triâmgulo equilátero de lado 1 com todos os vértices da mesma
>> cor.
>> >> 5) Pinte o plano em duas cores. Mostre que existe um retângulo com
>> todos
>> >> os vértice da mesma cor.
>> >>
>> >> Os dois primeiros são muito fáceis, os outros são mais complicados.
>> >
>> >
>> >
>> > --
>> > Rafael
>>
>> =
>> Instruções para entrar na lista, sair da lista e usar a lista em
>> http://www.mat.puc-rio.br/~obmlistas/obm-l.html
>> =
>>
>
>


-- 
Rafael


Re: [obm-l] Problemas interessantes de coloração

2008-07-25 Por tôpico Ralph Teixeira
Oi, Rafael -- mas esta distancia minima pode nao existir... Por exemplo, no
plano xy, imagine que pintamos de azul todos os pontos de coordenadas (x,y)
onde ambos x e y sao racionais; todos os outros pontos, onde x ou y sao
irracionais, a gente pinta de vermelho. Entao, escolhido um ponto A azul,
nao existe a distancia minima d>0 que voce citou...

Abraco,
Ralph

2008/7/25 Rafael Ando <[EMAIL PROTECTED]>:

> Nao sei se entendi direito o 3 e o 5, mas o que me impede de fazer o
> seguinte:
>
> Sejam azul e vermelho as duas cores. Seja A um ponto azul. Entao seja d>0 a
> distancia minima de A ate qualquer ponto vermelho. Entao todos o pontos da
> circunferencia de centro A e raio r triangulo equilatero inscrito nessa circunferencia resolveriam o
> problema
>
> 2008/7/24 Otávio Menezes <[EMAIL PROTECTED]>:
>
>> 1) Pinte o plano com três cores. Prove que há dois pontos com a mesma cor
>> situados a exatamente 1 unidade um do outro.
>> 2) Pinte o plano com duas cores. Prove que uma dessas cores contém pares
>> de pontos a qualquer distância entre si.
>> 3) Pinte o plano com duas cores. Prove que existe um triângulo equilátero
>> com todos os vértices da mesma cor.
>> 4) Mostre que é possível colorir o plano em duas cores de modo que não
>> exista um triâmgulo equilátero de lado 1 com todos os vértices da mesma cor.
>> 5) Pinte o plano em duas cores. Mostre que existe um retângulo com todos
>> os vértice da mesma cor.
>>
>> Os dois primeiros são muito fáceis, os outros são mais complicados.
>>
>
>
>
> --
> Rafael
>


Re: [obm-l] Problemas interessantes de coloração

2008-07-25 Por tôpico Ralph Teixeira
Mas nao precisa ser o triangulo todo da mesma cor -- bastam os VERTICES
:)

2008/7/25 Igor Battazza <[EMAIL PROTECTED]>:

> Tambem nao sei se entendi, pois o problema nao diz nada sobre
> restriçoes a respeito das cores... Se nao tiver restriçoes, na 4),
> acho que posso colorir o plano em listras alternadas com 2 cores, azul
> e vermelho por exemplo, de maneira que a espessura de cada listra seja
> menor do que 1 unidade (1/2 unidade por ex.).
>
> 2008/7/25 Rafael Ando <[EMAIL PROTECTED]>:
>  > Nao sei se entendi direito o 3 e o 5, mas o que me impede de fazer o
> > seguinte:
> >
> > Sejam azul e vermelho as duas cores. Seja A um ponto azul. Entao seja d>0
> a
> > distancia minima de A ate qualquer ponto vermelho. Entao todos o pontos
> da
> > circunferencia de centro A e raio r > triangulo equilatero inscrito nessa circunferencia resolveriam o
> > problema
> >
> > 2008/7/24 Otávio Menezes <[EMAIL PROTECTED]>:
> >>
> >> 1) Pinte o plano com três cores. Prove que há dois pontos com a mesma
> cor
> >> situados a exatamente 1 unidade um do outro.
> >> 2) Pinte o plano com duas cores. Prove que uma dessas cores contém pares
> >> de pontos a qualquer distância entre si.
> >> 3) Pinte o plano com duas cores. Prove que existe um triângulo
> equilátero
> >> com todos os vértices da mesma cor.
> >> 4) Mostre que é possível colorir o plano em duas cores de modo que não
> >> exista um triâmgulo equilátero de lado 1 com todos os vértices da mesma
> cor.
> >> 5) Pinte o plano em duas cores. Mostre que existe um retângulo com todos
> >> os vértice da mesma cor.
> >>
> >> Os dois primeiros são muito fáceis, os outros são mais complicados.
> >
> >
> >
> > --
> > Rafael
>
> =
> Instruções para entrar na lista, sair da lista e usar a lista em
> http://www.mat.puc-rio.br/~obmlistas/obm-l.html
> =
>


Re: [obm-l] Problemas interessantes de coloração

2008-07-25 Por tôpico Igor Battazza
Tambem nao sei se entendi, pois o problema nao diz nada sobre
restriçoes a respeito das cores... Se nao tiver restriçoes, na 4),
acho que posso colorir o plano em listras alternadas com 2 cores, azul
e vermelho por exemplo, de maneira que a espessura de cada listra seja
menor do que 1 unidade (1/2 unidade por ex.).

2008/7/25 Rafael Ando <[EMAIL PROTECTED]>:
> Nao sei se entendi direito o 3 e o 5, mas o que me impede de fazer o
> seguinte:
>
> Sejam azul e vermelho as duas cores. Seja A um ponto azul. Entao seja d>0 a
> distancia minima de A ate qualquer ponto vermelho. Entao todos o pontos da
> circunferencia de centro A e raio r triangulo equilatero inscrito nessa circunferencia resolveriam o
> problema
>
> 2008/7/24 Otávio Menezes <[EMAIL PROTECTED]>:
>>
>> 1) Pinte o plano com três cores. Prove que há dois pontos com a mesma cor
>> situados a exatamente 1 unidade um do outro.
>> 2) Pinte o plano com duas cores. Prove que uma dessas cores contém pares
>> de pontos a qualquer distância entre si.
>> 3) Pinte o plano com duas cores. Prove que existe um triângulo equilátero
>> com todos os vértices da mesma cor.
>> 4) Mostre que é possível colorir o plano em duas cores de modo que não
>> exista um triâmgulo equilátero de lado 1 com todos os vértices da mesma cor.
>> 5) Pinte o plano em duas cores. Mostre que existe um retângulo com todos
>> os vértice da mesma cor.
>>
>> Os dois primeiros são muito fáceis, os outros são mais complicados.
>
>
>
> --
> Rafael

=
Instruções para entrar na lista, sair da lista e usar a lista em
http://www.mat.puc-rio.br/~obmlistas/obm-l.html
=


Re: [obm-l] Problemas interessantes de coloração

2008-07-25 Por tôpico Rafael Ando
Nao sei se entendi direito o 3 e o 5, mas o que me impede de fazer o
seguinte:

Sejam azul e vermelho as duas cores. Seja A um ponto azul. Entao seja d>0 a
distancia minima de A ate qualquer ponto vermelho. Entao todos o pontos da
circunferencia de centro A e raio r:

> 1) Pinte o plano com três cores. Prove que há dois pontos com a mesma cor
> situados a exatamente 1 unidade um do outro.
> 2) Pinte o plano com duas cores. Prove que uma dessas cores contém pares de
> pontos a qualquer distância entre si.
> 3) Pinte o plano com duas cores. Prove que existe um triângulo equilátero
> com todos os vértices da mesma cor.
> 4) Mostre que é possível colorir o plano em duas cores de modo que não
> exista um triâmgulo equilátero de lado 1 com todos os vértices da mesma cor.
> 5) Pinte o plano em duas cores. Mostre que existe um retângulo com todos os
> vértice da mesma cor.
>
> Os dois primeiros são muito fáceis, os outros são mais complicados.
>



-- 
Rafael


Re: [obm-l] PROBLEMAS LITERAIS!

2008-05-09 Por tôpico roberto rocco
Talvez a grande problematica da coisa seja o fato dos estudantes não verem os 
"x" e os "y" com incognitas ou variaveis, porém como simples letras.
   
  Tudo de bom
  

Jorge Luis Rodrigues e Silva Luis <[EMAIL PROTECTED]> escreveu:
  .hmmessage P  {  margin:0px;  padding:0px  }  body.hmmessage  {  
FONT-SIZE: 10pt;  FONT-FAMILY:Tahoma  }  .ExternalClass .EC_hmmessage P 
 {padding:0px;}  .ExternalClass body.EC_hmmessage  
{font-size:10pt;font-family:Tahoma;}Olá, Pessoal! 
 
Apesar da simpatia pelos "XIZES & YPSILONS", os estudantes têm grande 
dificuldade em situações do tipo: A distãncia entre duas localidades é "x" 
quilômetros ou "y" metros . Traduzir esta afirmação em equação. Se "x" homens 
fazem "x" embrulhos em "x" segundos, "y" homens farão "y" embrulhos em ..
 
Mário tem x anos. Ele é o pai de Márcia, uma jovem de y anos. Responda usando 
um polinômio. Quantos anos Mário tem a mais que Márcia? Quantos anos Mário terá 
quando Márcia tiver 40 anos? Quantos anos Mário terá quando Márcia tiver a 
idade que hoje ele tem?
 
O administrador de uma fazenda ganha $x no 1° ano, $y no 2º ano, $x+y no 3° 
ano; depois continua a administrar a fazenda nas mesmas condições que no 3° 
ano. Exprimir o seu ganho depois de 9 anos de administração.
 
A propósito, que relação deve haver entre x e y para que a raiz quadrada x/y, 
seja x/y a menos de 1/y?
 
Agora, vem a bomba!!!
 
Comprei um objeto por $x e vendi por $y. Se todo o dinheiro usado na transação 
era falso, quanto deixei de ganhar?
 
Abraços!
  
-
  Receba GRÁTIS as mensagens do Messenger no seu celular quando você estiver 
offline. Conheça o MSN Mobile! Crie já o seu! 

   
-
Abra sua conta no Yahoo! Mail, o único sem limite de espaço para armazenamento! 

Re: [obm-l] Problemas Legais

2007-10-21 Por tôpico Marcelo Salhab Brogliato
Olá Benedito,

problema 1) acredito que n = [(b-1)(b-2)]/2 né?
vamos ver para b=2 ... n=0 ... bom, não é possível.. pois ambos os bolsos
ficaram vazios..
vamos ignorar este caso, entao: b=3 ... n=1 ... 2 bolsos ficaram vazios..

problema 2)
acho que sai pelo principio da casa dos pombos..
mas ainda nao consegui fazer :))

abracos,
Salhab




On 10/21/07, Benedito <[EMAIL PROTECTED]> wrote:
>
>  Problema 1
> Tenho um casaco com  b  bolsos  e  n  moedas de  1  real.Quero distribuir
> as moedas nos  b  bolsos, de maneira que em cada bolso haja uma quantidade
> diferente de reais.
> Se  n = [(p-1).(p-2)]/2,  isto pode ser feito? Como?
>
> Problema 2
> Pinte os números inteiros 1, 2, 3, ..., N  usando três cores, de modo que
> cada cor seja usada para pintar mais do que  N/4  dos inteiros dados.
> Mostre que a equação x = y + z  possui uma solução na qual  x, y, z  foram
> pintados com cores distintas.
>
> Benedito Freire
>


Re: [obm-l] Problemas de matematica! Desculpa! Correçao: p= -0,2q+100

2007-08-29 Por tôpico Marcelo Salhab Brogliato
Olá Claudio,

p = -0,2q + 100
Como temos q passageiros, o valor arrecadado pela empresa é: pq..
logo: pq = -0,2q^2 + 100q
o maximo desta funcao é: q = 100/0,4 = 250
logo: p = -0,2*250 + 100 = 50


x == 2 (mod 5)
y == 4 (mod 5)

x^5 == 2^5 = 32 == 2 (mod 5)
y^5 == 4^5 = 4^2 * 4^2 * 4 == 4 (mod 5)

x^5*y^5 == 8 == 3 (mod 5)
deste modo: x^5y^5 = 5k + 3
deve ser somado 2 para que seja multiplo de 5.. pois: x^5y^5 + 2 = 5(k+1)

abracos,
Salhab


On 8/29/07, jose silva <[EMAIL PROTECTED]> wrote:
> Se possivel, gostaria que me ajudasssem resolver os seguinte problemas
>
> Uma empresa de transporte estabelece, por viagem, o preço individual da
> passagem (p) em funçao da quantidade (q) de passageiros, atraves da relaçao
> p= -0,2q+100, com q maior que zero e menor que quinhentos. Nestas condiçoes,
> para que a quantidade arrecadada pela empresa, em cada viagem, seja maxima,
> o preço da passagem deve ser, em reais, igual a:
>
> Dividindo-se o numero "x" por 5 obtem-se o resto 2. Dividindo-se o numero
> "y" por 5, obtem-se resto 4. O menor numero inteiro, nao negativo, que se
> deve somar ao produto, x elevado a 5 multiplicado por y elevado a 5, para se
> obter um multiplo de 5 e:
>
> Desde ja, muito obrigado!
> Claudio.
>
> _
> Seja um dos primeiros a testar o novo Windows Live Mail Beta- grátis. Acesse
> http://www.ideas.live.com/programpage.aspx?versionId=5d21c51a-b161-4314-9b0e-4911fb2b2e6d
>
> =
> Instruções para entrar na lista, sair da lista e usar a lista em
> http://www.mat.puc-rio.br/~nicolau/olimp/obm-l.html
> =
>

=
Instruções para entrar na lista, sair da lista e usar a lista em
http://www.mat.puc-rio.br/~nicolau/olimp/obm-l.html
=


Re: [obm-l] PROBLEMAS ATÍPICOS!

2007-07-24 Por tôpico Marcelo Salhab Brogliato

Olá Jorge,

questoes dificeis.. hehe.. fiquei um bom tempo filosofando..

1) G = 0,20 * M, G = gasto de harry... M = milhas andadas por harry
obviamente, a semanada S de harry é: S >= G...
como harry pediu pra aumentar para 10 dolares, temos que: S < 10..
assim: G <= S < 10 .. de onde tiramos que: 0,20M < 10 ... M < 50
agora, G' = 0,40 * M ... como M < 50 ... G' < 20 e S' = 10 > S
sabemos que G' <= S'  0,40*M <= 10 ... M <= 25
assim, harry podera andar no maximo 25 milhas.. enquanto antes ele
poderia andar no maximo 50 milhas..
mas nao consegui concluir se ele anda mais ou menos.. :))

2) realmente.. essa aqui me deixou louco.. haha! nao consegui ver mta
relacao da pergunta com o enunciado.. dê alguma dica por favor :)

3) conclui que é a mesma coisa.. hehe
se vc leva 1 sabonete e para 1/2 no outro, temos que vc pagou
(1+1/2)/2 = 3/4 do preco por cada sabonete... agora se vc leva 4 e
paga 3, temos que vc pagou 3/4 do preco por cada sabonete.. logo, é a
mesma coisa.. é isso?

abracos,
Salhab





On 7/24/07, Jorge Luis Rodrigues e Silva Luis <[EMAIL PROTECTED]> wrote:

Olá, Pessoal!

Numa tentativa de limitar a utilização do carro da família pelo seu filho
Harry, Tom e Karen cobram-lhe uma taxa de utilização de 20 cêntimos por
milha. A este preço, Harry continua a utilizar o carro mais do que os pais
gostariam, mas estes sentem alguma relutância em contrariá-lo mais,
aumentando o preço. Assim, Tom e Karen perguntaram-lhe qual seria o aumento
mínimo de semanada que ele estaria disposto a aceitar em troca de um aumento
da taxa para 40 cêntimos a milha. Harry, que é conhecido por dizer sempre a
verdade e tem gostos convencionais, respondeu 10 dólares por semana. Se Tom
e Karen aumentarem a semanada de Harry para 10 dólares/semana e passarem a
cobrar-lhe 40 cêntimos por milha, será que ele guia menos do que antes? A
receita proveniente da taxa de utilização adicional será superior, inferior
ou igual a 10 dólares por semana?

Considere dois restaurantes italianos localizados em cidades semelhantes e
distanciadas uma da outra 200 milhas. Os restaurantes são idênticos em todos
os aspectos, exceto no que respeita à política de gorjetas. Num, existe uma
sobretaxa de encargos de serviço de 15 dólares e não se aceita qualquer
outro tipo de gorjetas. No outro, inclui-se na conta uma taxa de 15 por
cento. No primeiro restaurante, em média, uma conta, excluindo o encargo de
serviço, é de 100 dólares. Qual será a diferença entre a quantidade de
comida consumida nos dois restaurantes?  (Fácil, não...!)


A propósito, qual oferece maior desconto: comprar um sabonete e levar outro
pela metade do preço ou levar quatro e pagar três? (Inexplicavelmente, muita
gente escorrega no sabonete...)


Abraços!

_
Chegou o Windows Live Spaces com rede social. Confira
http://spaces.live.com/

=
Instruções para entrar na lista, sair da lista e usar a lista em
http://www.mat.puc-rio.br/~nicolau/olimp/obm-l.html
=



=
Instruções para entrar na lista, sair da lista e usar a lista em
http://www.mat.puc-rio.br/~nicolau/olimp/obm-l.html
=


Re: [obm-l] Problemas Olimpicos

2007-07-16 Por tôpico Rogerio Ponce
Ah, agora sim!

Bem, ou o numero inicial (o menor da sequencia) possui uma terminacao entre 
"81" e "99" inclusive, ou entre "00" e "80" inclusive .

No primeiro caso, havera' uma sub-sequencia indo de "...00" ate'  "...19" . 
Repare que com esses algarismos das dezenas e unidades, obtemos somas entre 0 e 
10, o que e' suficiente para, ao adicionarmos os outros algarismos, 
conseguirmos um multiplo de 11.

Similarmente, no segundo caso havera' uma subsequencia com 2 diferentes (e em 
sequencia) algarismos das dezenas, e todos os 10 algarismos das unidades, o que 
e' suficiente para obtermos uma soma que adicionada aos outros algarismos seja 
um multiplo de 11.

[]'s
Rogerio Ponce


Paulo Santa Rita <[EMAIL PROTECTED]> escreveu: Ola Pessoal,

Em mensagem anterior eu enviei um problema cujo enunciado faltava uma
palavra. Eis aqui o enunciado correto :

PROBLEMA) Prove que em qualquer sequencia de 39 inteiro positivos
CONSECUTIVOS ha ao menos um numero cuja soma dos algarismos e
divisivel por 11.

Esse e um dos problema das Olimpiadas Russas e voces podem ver mais
problemas deste mesmo nivel aqui :

http://ww.mat.puc-rio.br/~nicolau/psr

Um Abraço a todos
Paulo Santa Rita
2,0C1C,0F0707

=
Instruções para entrar na lista, sair da lista e usar a lista em
http://www.mat.puc-rio.br/~nicolau/olimp/obm-l.html
=


   
-
Novo Yahoo! Cadê? - Experimente uma nova busca. 

Re: [obm-l] PROBLEMAS INVULGARES!

2007-04-03 Por tôpico Chicao Valadares
>Agora, vem a "bomba" que pouca gente
> sabe desativar: Como 
> fracionar 7 pães entre 10 homens?  (Campeão!)

Divide cada pão em 10 pedaços e dá sete pedaços pra
cada homem.

"O Binômio de Newton é tão belo como a Vênus de Milo.
O que há é pouca gente para dar por isso... "
Fernando Pessoa - Poesias de Alvaro Campos

_
As informações existentes nessa mensagem e no(s) arquivo(s) anexado(s) 
são
para uso restrito, sendo seu sigilo protegido por lei. Caso não seja
destinatário, saiba que leitura, divulgação ou cópia são proibidas. 
Favor
apagar as informações e notificar o remetente. O uso impróprio será 
tratado
conforme as normas da empresa e a legislação em vigor. Agradecemos sua
colaboração.


The information mentioned in this message and in the archives attached 
are
of restricted use, and its privacy is protected by law. If you are not 
the
addressee, be aware that reading, disclosure or copy are forbidden. 
Please
delete this information and notify the sender. Inappropriate use will 
be
tracted according to company's rules and valid laws. Thank you for your
cooperation.

__
Fale com seus amigos  de graça com o novo Yahoo! Messenger 
http://br.messenger.yahoo.com/ 
=
Instruções para entrar na lista, sair da lista e usar a lista em
http://www.mat.puc-rio.br/~nicolau/olimp/obm-l.html
=


Re:[obm-l] Problemas em aberto

2007-02-14 Por tôpico claudio\.buffara

> 2. Num espaco metrico compacto, uma sequencia (x(n)) eh tal que lim(n->+inf) 
> dist(x(n+1),x(n)) = 0.
> Prove que o conjunto de valores de aderencia de (x(n)) eh conexo.
>
> Eu provei no caso de (x(n)) ser uma sequencia limitada na reta.
> Se x(n) -> a, entao A = conjunto dos valores de aderencia de (x(n)) = {a}, 
> que eh conexo.
> Se x(n) nao converge, sejam a = liminf(x(n)) e b = limsup(x(n)).
> (a e b existem pois (x(n)) eh limitada e, alem disso, a < b, pois (x(n)) 
> diverge)
> Finalmente, seja c tal que a < c < b.
> Tomemos eps > 0.
> Podemos supor spdg que eps eh pequeno o bastante para que os intervalos 
> (a-eps,a+eps), (c-eps,c+eps) e (b-eps,b+eps) sejam
> disjuntos dois a dois.
> Seja k_0 em N tal que k > k_0 ==> |x(k+1) - x(k)| < eps.
> Seja m > k_0 tal que x(m) pertence a (a-eps,a+eps).
> Seja n o menor inteiro maior do que m tal que x(n) pertence a (b-eps,b+eps).
> (m e n existem pois a e b sao limites de subsequencias de (x(n)) )
> Eh claro que x(m) < c-eps < c+eps < x(n).
> Seja X = {r em N | m <= r <= n e x(r) <= c-eps}.
> Naturalmente, X eh nao-vazio (m pertence a X) e limitado superiormente (por 
> n, que nao pertence a X, mas isso nao importa).
> Seja r = maior elemento de X.
> Entao, x(r) <= c-eps < x(r+1) e, portanto, x(r+1) < c+eps, pois se fosse 
> x(r+1) >= c+eps, entao:
> |x(r+1) - x(r)| >= 2*eps, o que eh impossivel pois r >= m > k.
> Logo, x(r+1) pertence a (c-eps,c+eps).
> Ou seja, para cada eps > 0, existe n em N tal que x(n) pertence a 
> (c-eps,c+eps) ==>
> c eh um valor de aderencia de (x(n)).
> Como c eh um elemento arbitrario de (a,b), concluimos que (a,b) estah contido 
> em A.
> Como a = min(A) e b = max(A), concluimos que A = [a,b] = conexo.
>


No caso geral, temos que A (conjunto de valores de aderência de (x(n)) é 
fechado (um bom exercício é provar isso). Como o espaço é compacto, A é 
compacto.

Suponhamos que A não seja conexo.
Então, podemos escrever A = B uniao C, onde B e C são compactos disjuntos e 
não-vazios. Temos dist(B,C) = d > 0 (outro bom exercício)

B pode ser coberto por um número finitos de bolas abertas com centro em algum 
ponto de B e raio = d/3 (Heine-Borel-Lebesgue). Idem para C. Chame a união 
dessas bolas de VB e VC respectivamente ("V" de vizinhança...).
Temos que dist(VB,VC) >= d/3 (mais um exercício)

Seja k_0 em N tal que para todo k > k_0, dist(x(k+1),x(k)) < d/3.
Seja m > k_0 tal que x(m) pertence a VB. Um tal m existe pois alguma 
subsequencia de (x(n)) converge para algum ponto de B.
Seja p o menor inteiro maior que m tal que x(p+1) pertence a VC. Um tal p 
também existe pois alguma subsequencia de (x(n)) converge para algum ponto de C.

Não é difícil ver que x(p) não pertence a VB nem a VC.
Não pertence a VC pela escolha de p e não pertence a VB pois, se esse fosse o 
caso, como p > m > k_0, teríamos:
d/3 > dist(x(p+1),x(p)) >= dist(VC,VB) >= d/3, o que é uma contradição.

Assim, para cada par (m,n) em NxN com m < n tal que x(m) pertence a VB e x(n) 
pertence a VC, vai existir p com m < p < n tal que x(p) não pertence a VB união 
VC. Como o conjunto dos pares (m,n) é infinito, o conjunto dos p também é. Ou 
seja, (x(p)) é uma subsequência de (x(n)) cujos pontos não pertencem a VB união 
VC. Como o espaço é compacto, (x(p)) é limitada e, portanto, possui uma 
subsequência convergente para um ponto c, o qual também não pertence a VB unão 
VC (pois o complementar de VB união VC é fechado). Logo, c é um valor de 
aderência de (x(n)) que não pertence a VB união VC e, com mais razão ainda, não 
pertence a B união C = A ==> contradição.

A única origem possível dessa contradição é a hipótese feita inicialmente de A 
não ser conexo. Logo, concluímos que A é conexo.


Belo problema esse! Eu levei um bom tempo pra formalizar uma solução apesar da 
idéia básica ser simples: pra sequência alternar infinitas vezes entre VB e VC, 
uma infinidade de termos teria que estar no complementar de B união C, já que, 
a partir de um certo ponto, a distância entre dois termos consecutivos é menor 
do que a distância entre VB e VC. Por causa da compacidade do espaço, esses 
termos teriam que se acumular no espaço entre B e C e formar uma "ponte" conexa 
entre os dois conjuntos.


[]s,
Claudio.


Re: [obm-l] Problemas em aberto

2007-02-14 Por tôpico Paulo Santa Rita

Oi claudio,

Leia com atencao. Eu disse que NO CASO DA SERIE GEOMETRICA . Alem
disso, voce ja sabe a forma definitiva do polinomio TRABALHAVEL ou
SOMAVEL ? E aqui que esta a graca do problema ...

Se nos ficarmos discutindo detalhes nao chegaremos a lugar algum. O
que e importante e o total sentido da questao e que ha uma resposta
definitiva em formula fechada. Se voce quer trabalhar nisso pense
assim :

Dado S = 1 + q + ... + q^[N(N+1)/2] + ... . Exprima esta soma em funca de "q"

SUGESTAO 1 : usando o exemplo da serie geometrica, multiplique S por
um expressao
conveniente de forma que o resultado seja mais facilmente trabalhavel

SUGESTAO 2 : IMAGINE que os termos de S foram retirado de uma PG infinita
T = 1 + q + q^2 + q^3 + ... O que sobra em T sao PG's facilmente
somaveis. Isso permite obter uma formula de recorrencia para Sn.
Trabalhe nesta formula

Este resultado e uma generalizacao do conceito de Progressao
Geometrica. Eu chamava de PROGRESSAO GEOMETRICA DE 2 ORDEM, quando era
crianca. Eis aqui o problema para a terceira ordem :

Seja reais positivos A1, A2, ..., An, ... tal que An+1 /  An =
q^[N(N-1)/2]. Onde 0 < q <1. Exprima em funcao 'q" a soma infinita A1
+ A2 + A3 + ...+ An + ...

De maneira geral, se 0 escreveu:

Oi, Paulo:

Se existir um tal polinomio p(q), entao eh facil ver que os coeficientes serao 
inteiros (eh soh montar a recorrencia).

No entanto, nao pode existir um tal polinomio pois, se tivermos:
p(q) * SOMA(k>=1) q^(k(k-1)/2) = SOMA(n>=0) q^n = 1/(1-q), se |q| < 1.
entao, com q = 1/2, teriamos:
SOMA(k>=1) (1/2^(k(k-1)/2)) = 2/p(1/2) ==>

Mas, pelo teorema das raizes racionais, p(1/2) <> 0 e, alem disso, eh 
claramente racional.
Logo, 2/p(1/2) eh racional.
No entanto, SOMA(k>=1) (1/2^(k(k-1)/2)) eh irracional (basta ver que, em base 
2, esta soma eh uma decimal infinita e nao
periodica) ==> contradicao ==> nao existe p(q).

[]s,
Claudio.

-- Cabeçalho original ---

De: [EMAIL PROTECTED]
Para: obm-l@mat.puc-rio.br
Cópia:
Data: Tue, 13 Feb 2007 17:58:20 +
Assunto: RE: [obm-l] Problemas em aberto

>
> Ola Ronaldo e demais colegas
> desta lista ... OBM-L,
>
> Parabens, a sua intuicao e muito boa. Eu acho que se voce parar para pensar 
com mais calma, sem pressupostos, vai
resolver...
> Talvez falte voce observar o seguinte :
>
> Na serie geometrica bem conhecida Sn = 1 + q +  q^2  +  ...  + q^(N-1),  0 < q 
< 1,  como calculamos o LIM Sn quando N
vai para
> o infinito ? Em geral, fixamos N e multiplicamos Sn por UM POLINOMIO p(q) EM 
"q", CONVENIENTE, tal que
>
> p(q)*Sn = ALGO SOMAVEL OU MAIS SIMPLES.
>
> no caso particular da serie geomentrica temos que p(q) = q - 1 pois (q - 1)*Sn 
=q^N  - 1. A seguir, dado que q^N  -> 0
quando N
> vai para o infinito seque que Sn = 1/(1-q). Note que aqui p(q)*sn= ALGO MAIS 
SIMPLES. Poderia ser tambem algo somavel ou
computavel ...
>
> Seja agora Sn = 1 + q + q^3 + ... + q^[(N(N-1))/2]. Quem e p(q) tal que
>
> p(q)*Sn = ALGO SOMAVEL OU MAIS SIMPLES ?
>
> Eis  uma questao na qual um software como o MAXIMA ou OCTAVE facilita muito 
as coisas, pois nos permite fazer experiencias
com as
> diversas possibilidades do polinomio p(q) que precisamos descobrir.
>
> E com os melhores votos
> de paz profunda, sou
> Paulo Santa Rita
> 3,0F38,130207
>
> ____
> > Date: Tue, 13 Feb 2007 12:50:30 -0300
> > From: [EMAIL PROTECTED]
> > To: obm-l@mat.puc-rio.br
> > Subject: Re: [obm-l] Problemas em aberto
> >
> > Se  o termo n(n-1)/2 fosse n(n+1)/2 ele seria a soma de uma P.A. com os n 
primeiros naturais.
> > Não parei ainda para pensar com calma, mas será que esse problema não está 
relacionado com partições de inteiros e
> > a função de Euler ?
> > http://en.wikipedia.org/wiki/Integer_partition
> > Note que o termo de x^n que é p(n) conta o número de vezes em que é 
possível escrever n = a_1 + 2a_2 + 3a_3 + ... onde
cada a_i
> > aparece i vezes.
> > Bem, alguém já deve ter pensado nisso, então o que eu falei não ajuda muito 
... :)
> > []s
> > Ronaldo
>
> _
> Busque em qualquer página da Web com alta proteção. Obtenha o Windows Live 
Toolbar GRATUITO ainda hoje!
> http://toolbar.live.com/
> =
> Instruções para entrar na lista, sair da lista e usar a lista em
> http://www.mat.puc-rio.br/~nicolau/olimp/obm-l.html
> =
>
>


=
Instruções para entrar na lista, sair da lista e usar a lista em
http://www.mat.puc-rio.br/~nicolau/olimp/obm-l.html
=



=
Instruções para entrar na lista, sair da lista e usar a lista em
http://www.mat.puc-rio.br/~nicolau/olimp/obm-l.html
=


RE: [obm-l] Problemas em aberto

2007-02-14 Por tôpico claudio\.buffara
Oi, Paulo:

Se existir um tal polinomio p(q), entao eh facil ver que os coeficientes serao 
inteiros (eh soh montar a recorrencia).

No entanto, nao pode existir um tal polinomio pois, se tivermos:
p(q) * SOMA(k>=1) q^(k(k-1)/2) = SOMA(n>=0) q^n = 1/(1-q), se |q| < 1.
entao, com q = 1/2, teriamos:
SOMA(k>=1) (1/2^(k(k-1)/2)) = 2/p(1/2) ==>

Mas, pelo teorema das raizes racionais, p(1/2) <> 0 e, alem disso, eh 
claramente racional.
Logo, 2/p(1/2) eh racional.
No entanto, SOMA(k>=1) (1/2^(k(k-1)/2)) eh irracional (basta ver que, em base 
2, esta soma eh uma decimal infinita e nao 
periodica) ==> contradicao ==> nao existe p(q).

[]s,
Claudio.

-- Cabeçalho original ---

De: [EMAIL PROTECTED]
Para: obm-l@mat.puc-rio.br
Cópia: 
Data: Tue, 13 Feb 2007 17:58:20 +
Assunto: RE: [obm-l] Problemas em aberto

> 
> Ola Ronaldo e demais colegas
> desta lista ... OBM-L,
> 
> Parabens, a sua intuicao e muito boa. Eu acho que se voce parar para pensar 
> com mais calma, sem pressupostos, vai 
resolver...  
> Talvez falte voce observar o seguinte :
> 
> Na serie geometrica bem conhecida Sn = 1 + q +  q^2  +  ...  + q^(N-1),  0 < 
> q < 1,  como calculamos o LIM Sn quando N 
vai para
> o infinito ? Em geral, fixamos N e multiplicamos Sn por UM POLINOMIO p(q) EM 
> "q", CONVENIENTE, tal que
> 
> p(q)*Sn = ALGO SOMAVEL OU MAIS SIMPLES.
> 
> no caso particular da serie geomentrica temos que p(q) = q - 1 pois (q - 
> 1)*Sn =q^N  - 1. A seguir, dado que q^N  -> 0 
quando N
> vai para o infinito seque que Sn = 1/(1-q). Note que aqui p(q)*sn= ALGO MAIS 
> SIMPLES. Poderia ser tambem algo somavel ou 
computavel ... 
> 
> Seja agora Sn = 1 + q + q^3 + ... + q^[(N(N-1))/2]. Quem e p(q) tal que
> 
> p(q)*Sn = ALGO SOMAVEL OU MAIS SIMPLES ?
> 
> Eis  uma questao na qual um software como o MAXIMA ou OCTAVE facilita muito 
> as coisas, pois nos permite fazer experiencias 
com as
> diversas possibilidades do polinomio p(q) que precisamos descobrir.
> 
> E com os melhores votos
> de paz profunda, sou
> Paulo Santa Rita
> 3,0F38,130207
> 
> ________
> > Date: Tue, 13 Feb 2007 12:50:30 -0300
> > From: [EMAIL PROTECTED]
> > To: obm-l@mat.puc-rio.br
> > Subject: Re: [obm-l] Problemas em aberto
> > 
> > Se  o termo n(n-1)/2 fosse n(n+1)/2 ele seria a soma de uma P.A. com os n 
> > primeiros naturais.
> > Não parei ainda para pensar com calma, mas será que esse problema não está 
> > relacionado com partições de inteiros e
> > a função de Euler ?
> > http://en.wikipedia.org/wiki/Integer_partition
> > Note que o termo de x^n que é p(n) conta o número de vezes em que é 
> > possível escrever n = a_1 + 2a_2 + 3a_3 + ... onde 
cada a_i
> > aparece i vezes.
> > Bem, alguém já deve ter pensado nisso, então o que eu falei não ajuda muito 
> > ... :)
> > []s
> > Ronaldo
> 
> _
> Busque em qualquer página da Web com alta proteção. Obtenha o Windows Live 
> Toolbar GRATUITO ainda hoje!
> http://toolbar.live.com/
> =
> Instruções para entrar na lista, sair da lista e usar a lista em
> http://www.mat.puc-rio.br/~nicolau/olimp/obm-l.html
> =
> 
> 


=
Instruções para entrar na lista, sair da lista e usar a lista em
http://www.mat.puc-rio.br/~nicolau/olimp/obm-l.html
=


Re: [obm-l] Problemas em aberto

2007-02-13 Por tôpico Carlos Eddy Esaguy Nehab

Oi, Claudio,

O problema de complexos que você mencionou é uma ferramenta 
extremamente útil que já usei para demonstrar inúmeros problemas de 
geometria, como por exemplo o famoso teorema atribuido ao 
Napoleão  (o Bonaparte, mesmo, acredite se quiser... :-)), que eu 
acho surpreendente:


"Sobre os lados de um triângulo arbitrário construa três triângulos 
equiláteros exteriores ao mesmo.  Mostre que os centros destes 3 
triângulos equiláteros determinam um novo triângulo equilátero".


O teorema do Napoleão também é relacionado a outro problema 
(atribuído a Pascal) igualmente interessante:  "Dado um triângulo 
qualquer, determine o ponto de seu plano cuja soma das distâncias aos 
vértices é mínima".


Os aficcionados em Geometria que se regozigem...  São bonitos, assim, 
como as soluções.


Quanto ao somatório (com expoentes sendo os números triangulares) tô 
pensando...


Abraços,
Nehab

At 13:50 13/2/2007, you wrote:


On 2/13/07, claudio.buffara 
<[EMAIL PROTECTED]> wrote:
Antes de postar um problema bonitinho sobre complexos, quero lembrar 
que ainda temos (pelo menos) dois problemas em aberto

na lista, um do PSRita e o outro do ACSteiner:

1. Calcule o valor de SOMA(n=1...+inf) q^(n(n-1)/2), onde |q| < 1.

Consultei meus alfarrabios e descobri que esta soma eh igual a um 
certo produto infinito, mas nao achei nenhuma formula
fechada e suspeito que nenhuma exista, a menos que envolva alguma 
funcao nao elementar - alias, como a serie acima

converge, ela pode ser usada pra definir uma funcao de (-1,1) -> R.


Se  o termo n(n-1)/2 fosse n(n+1)/2 ele seria a soma de uma 
P.A. com os n primeiros naturais.
 Não parei ainda para pensar com calma, mas será que esse problema 
não está relacionado com partições de inteiros e

a função de Euler?

http://en.wikipedia.org/wiki/Integer_partition

  Note que o termo de x^n que é p(n) conta o número de vezes em que 
é possível escrever n = a_1 + 2a_2 + 3a_3 + ... onde cada a_i

aparece i vezes.
   Bem, alguém já deve ter pensado nisso, então o que eu falei não 
ajuda muito ... :)


[]s
Ronaldo





RE: [obm-l] Problemas em aberto

2007-02-13 Por tôpico Paulo Santa Rita

Ola Ronaldo e demais colegas
desta lista ... OBM-L,

Parabens, a sua intuicao e muito boa. Eu acho que se voce parar para pensar com 
mais calma, sem pressupostos, vai resolver...  
Talvez falte voce observar o seguinte :

Na serie geometrica bem conhecida Sn = 1 + q +  q^2  +  ...  + q^(N-1),  0 < q 
< 1,  como calculamos o LIM Sn quando N vai para
o infinito ? Em geral, fixamos N e multiplicamos Sn por UM POLINOMIO p(q) EM 
"q", CONVENIENTE, tal que

p(q)*Sn = ALGO SOMAVEL OU MAIS SIMPLES.

no caso particular da serie geomentrica temos que p(q) = q - 1 pois (q - 1)*Sn 
=q^N  - 1. A seguir, dado que q^N  -> 0 quando N
vai para o infinito seque que Sn = 1/(1-q). Note que aqui p(q)*sn= ALGO MAIS 
SIMPLES. Poderia ser tambem algo somavel ou computavel ... 

Seja agora Sn = 1 + q + q^3 + ... + q^[(N(N-1))/2]. Quem e p(q) tal que

p(q)*Sn = ALGO SOMAVEL OU MAIS SIMPLES ?

Eis  uma questao na qual um software como o MAXIMA ou OCTAVE facilita muito as 
coisas, pois nos permite fazer experiencias com as
diversas possibilidades do polinomio p(q) que precisamos descobrir.

E com os melhores votos
de paz profunda, sou
Paulo Santa Rita
3,0F38,130207


> Date: Tue, 13 Feb 2007 12:50:30 -0300
> From: [EMAIL PROTECTED]
> To: obm-l@mat.puc-rio.br
> Subject: Re: [obm-l] Problemas em aberto
> 
> Se  o termo n(n-1)/2 fosse n(n+1)/2 ele seria a soma de uma P.A. com os n 
> primeiros naturais.
> Não parei ainda para pensar com calma, mas será que esse problema não está 
> relacionado com partições de inteiros e
> a função de Euler ?
> http://en.wikipedia.org/wiki/Integer_partition
> Note que o termo de x^n que é p(n) conta o número de vezes em que é possível 
> escrever n = a_1 + 2a_2 + 3a_3 + ... onde cada a_i
> aparece i vezes.
> Bem, alguém já deve ter pensado nisso, então o que eu falei não ajuda muito 
> ... :)
> []s
> Ronaldo

_
Busque em qualquer página da Web com alta proteção. Obtenha o Windows Live 
Toolbar GRATUITO ainda hoje!
http://toolbar.live.com/
=
Instruções para entrar na lista, sair da lista e usar a lista em
http://www.mat.puc-rio.br/~nicolau/olimp/obm-l.html
=


Re: [obm-l] Problemas em aberto

2007-02-13 Por tôpico Marcelo Salhab Brogliato

Olá,

tomemos os numeros complexos a, b, c, entao:

considerando que ||b-a|| = ||c-a|| = ||b-c||, temos:
(b-a)/(c-a) = cis(alfa), onde alfa é o ângulo entre as arestas AB e AC...
(a-c)/(b-c) = cis(beta), onde beta é o ângulo entre as arestas CA e CB...

se alfa = beta... temos: (b-a)/(c-a) = (a-c)/(b-c)
assim: bb - ab - bc + ac = ca - cc - aa + ac
logo: aa + bb + cc = ac + ab + bc

logo, se o triangulo é equilatero, vale a relacao!
para fazer a volta, basta fatorarmos, chegando a (b-a)/(c-a) = (a-c)/(b-c)
assim, os modulos tem q ser iguais e os angulos tb!

abracos,
Salhab


- Original Message - 
From: "claudio.buffara" <[EMAIL PROTECTED]>

To: "obm-l" 
Sent: Tuesday, February 13, 2007 10:37 AM
Subject: [obm-l] Problemas em aberto


Antes de postar um problema bonitinho sobre complexos, quero lembrar que 
ainda temos (pelo menos) dois problemas em aberto

na lista, um do PSRita e o outro do ACSteiner:

1. Calcule o valor de SOMA(n=1...+inf) q^(n(n-1)/2), onde |q| < 1.

Consultei meus alfarrabios e descobri que esta soma eh igual a um certo 
produto infinito, mas nao achei nenhuma formula
fechada e suspeito que nenhuma exista, a menos que envolva alguma funcao nao 
elementar - alias, como a serie acima

converge, ela pode ser usada pra definir uma funcao de (-1,1) -> R.


2. Num espaco metrico compacto, uma sequencia (x(n)) eh tal que lim(n->+inf) 
dist(x(n+1),x(n)) = 0.

Prove que o conjunto de valores de aderencia de (x(n)) eh conexo.

Eu provei no caso de (x(n)) ser uma sequencia limitada na reta.
Se x(n) -> a, entao A = conjunto dos valores de aderencia de (x(n)) = {a}, 
que eh conexo.

Se x(n) nao converge, sejam a = liminf(x(n)) e b = limsup(x(n)).
(a e b existem pois (x(n)) eh limitada e, alem disso, a < b, pois (x(n)) 
diverge)

Finalmente, seja c tal que a < c < b.
Tomemos eps > 0.
Podemos supor spdg que eps eh pequeno o bastante para que os intervalos 
(a-eps,a+eps), (c-eps,c+eps) e (b-eps,b+eps) sejam

disjuntos dois a dois.
Seja k_0 em N tal que k > k_0 ==> |x(k+1) - x(k)| < eps.
Seja m > k_0 tal que x(m) pertence a (a-eps,a+eps).
Seja n o menor inteiro maior do que m tal que x(n) pertence a (b-eps,b+eps).
(m e n existem pois a e b sao limites de subsequencias de (x(n)) )
Eh claro que x(m) < c-eps < c+eps < x(n).
Seja X = {r em N | m <= r <= n  e  x(r) <= c-eps}.
Naturalmente, X eh nao-vazio (m pertence a X) e limitado superiormente (por 
n, que nao pertence a X, mas isso nao importa).

Seja r = maior elemento de X.
Entao, x(r) <= c-eps < x(r+1) e, portanto, x(r+1) < c+eps, pois se fosse 
x(r+1) >= c+eps, entao:

|x(r+1) - x(r)| >= 2*eps, o que eh impossivel pois r >= m > k.
Logo, x(r+1) pertence a (c-eps,c+eps).
Ou seja, para cada eps > 0, existe n em N tal que x(n) pertence a 
(c-eps,c+eps) ==>

c eh um valor de aderencia de (x(n)).
Como c eh um elemento arbitrario de (a,b), concluimos que (a,b) estah 
contido em A.

Como a = min(A) e b = max(A), concluimos que A = [a,b] = conexo.

***

O probleminha sobre complexos eh o seguinte:
a, b, c sao numeros complexos. Prove que:
a, b, c sao os vertices de um triangulo equilatero no plano complexo
se e somente se
a^2 + b^2 + c^2 = ab + ac + bc.


[]s,
Claudio.



=
Instruções para entrar na lista, sair da lista e usar a lista em
http://www.mat.puc-rio.br/~nicolau/olimp/obm-l.html
= 


=
Instruções para entrar na lista, sair da lista e usar a lista em
http://www.mat.puc-rio.br/~nicolau/olimp/obm-l.html
=


Re: [obm-l] Problemas em aberto

2007-02-13 Por tôpico Ronaldo Alonso

On 2/13/07, claudio.buffara <[EMAIL PROTECTED]> wrote:


Antes de postar um problema bonitinho sobre complexos, quero lembrar que
ainda temos (pelo menos) dois problemas em aberto
na lista, um do PSRita e o outro do ACSteiner:

1. Calcule o valor de SOMA(n=1...+inf) q^(n(n-1)/2), onde |q| < 1.

Consultei meus alfarrabios e descobri que esta soma eh igual a um certo
produto infinito, mas nao achei nenhuma formula
fechada e suspeito que nenhuma exista, a menos que envolva alguma funcao
nao elementar - alias, como a serie acima
converge, ela pode ser usada pra definir uma funcao de (-1,1) -> R.



   Se  o termo n(n-1)/2 fosse n(n+1)/2 ele seria a soma de uma P.A. com os
n primeiros naturais.
Não parei ainda para pensar com calma, mas será que esse problema não está
relacionado com partições de inteiros e
a função de Euler?

http://en.wikipedia.org/wiki/Integer_partition

 Note que o termo de x^n que é p(n) conta o número de vezes em que é
possível escrever n = a_1 + 2a_2 + 3a_3 + ... onde cada a_i
aparece i vezes.
  Bem, alguém já deve ter pensado nisso, então o que eu falei não ajuda
muito ... :)

[]s
Ronaldo


Re: [obm-l] PROBLEMAS

2007-01-25 Por tôpico Carlos Gomes
Arkon, eh o seguite:

 

1) Sejam x,y,z, e w as quantidades de rabos de morcegos, unhas de largatixas, 
olhos de salamandra e litros de sangue de novilho, respectivamente. Assim, de 
acordo com o enunciado teremos que:

 

5x+5y+5z+20w = 100  ==>  x+y+z+4w = 20  ==>  w = 5 - (x+y+z)/4. Como w deve ser 
inteiro positivo segue que (x+y+z) deve ser múltiplo de 4. Logo temos as 
possibilidades a seguir:

 

(x+y+z)=0 ==> x=0, y=0, z=0 (POIS X,Y E Z NÃO PODEM SER NEGATIVOS), o que não 
serve, pois como estamos fabricando uma porção mágina todos os ingredientes são 
necessários.



fazendo (x+y+z)=4 (que é o próximo múltiplo de 4) ==> w=4. Neste caso, 
(x+y+z)=4 tem três possibilidades x=1, y=1 e z=2  ou x=1, y=2 e z=1  ou ainda 
x=2, y=1 e z=1.

 

obs. Melhor que listar as soluções inteias positivas da equação (x+y+z)=4, 
seria raciocinar da seguinte forma:

 

já que x, y e z são no mínimo 1, temos que x=a+1, y=b+1 e z=c+1 , onde a,b e c 
são não negativos. Agora substituindo as relações x=a+1, y=b+1 e z=c+1 , na 
equação x+y+z=4  obtemos a+b+c=1 e agora então determinamos a quantidade de 
soluções inteiras e não negativas da equação a+b+c=1, que são  C(3,2)=3.

 

continuando...com os próximos múltiplos de 4 e fazendo  x=a+1, y=b+1 e z=c+1

 

(x+y+z)=8  ==> a+b+c=5 que apresenta C(7,2)=21 soluções.

(x+y+z)=12  ==> a+b+c=9 que apresenta C(11,2)=55

(x+y+z)=16 ==> a+b+c=13 apresenta C(15,2)=105 soluções.

 

assim existem 3+21+55+105=184 soluções. (Não é divisível por 13).



acho que é isso!





2) 





  - Original Message - 
  From: arkon 
  To: obm-l 
  Sent: Thursday, January 25, 2007 2:52 PM
  Subject: [obm-l] PROBLEMAS


  Olá pessoal, muito obrigado pelas resoluções.
  Tem mais.

  Desde já agradeço.

  ABRAÇOS.

  (OBJETIVO) Um feiticeiro vai comprar os ingredientes que sua esposa bruxa 
precisa para preparar uma poção mágica. Ele tem cem reais para fazer as compras 
numa loja de artigos estranhos. A loja vende rabo de morcego por cinco reais o 
pedaço, unhas de lagartixa por cinco reais o pedaço, olhos de salamandra por 
cinco reais a unidade e sangue de novilho por vinte reais o litro. Calcular o 
número de maneiras distintas que a compra poderá ser feita com os cem reais do 
feiticeiro. Divida o resultado por 13. 

  Obs.: Os ingredientes não podem ser subdivididos.

   

  (OBJETIVO) Um político contrata quatro segurança para poder participar de um 
showmício de seu partido. Os seguranças localizam-se nos vértices de um 
quadrado. Sabe-se que três deles estão a 1, 4 e 5 m de seu patrão e sempre 
mantém esta configuração. De posse dessas informações, resolva:

   

  a)  Numa emergência em que um dos seguranças fosse atingido 
por uma bala, os outros deveriam constituir um formato de um triângulo 
eqüilátero de lado 2 cm, onde o político localizar-se-ia em seu centro. 
Calcular a distância, em cm, do político aos seguranças nesta ocasião. Despreze 
a parte fracionária do resultado, caso exista.



  b) Calcular em m2, a área do quadrado.



--


  No virus found in this incoming message.
  Checked by AVG Free Edition.
  Version: 7.1.410 / Virus Database: 268.17.8/649 - Release Date: 23/1/2007


Re: [obm-l] Re:[obm-l] Problemas com Triângulo s?=

2006-12-15 Por tôpico Eduardo Wilner
No problema 1)
   
 O triângulo ABC é isóceles com os ãngulos A e B iguais. 
   
   Assim A+B+C= 2A+(A/2) =180°   ==>   5A= 360°==>  A=72°  


"claudio.buffara" <[EMAIL PROTECTED]> escreveu:
Solução do Problema 2:
   
  Seja P = B1C2 inter B2C1.
   
  AB1B e AC1C são triângulos retângulos de 30, 60 e 90 ==>
  B1C2 = BC2 = AB/2  e  C1B2 = CB2 = AC/2 ==>
  BB1C2 e CC1B2 são equiláteros ==>
  BB1C1 + C1B1C2 = BB1C2 = 60   (i);
  CC1B1 + B1C1B2 = CC1B2 = 60   (ii);
  CBB1 + BCC1 = 180 - A - C2BB1 - B2CC1 = 180 - 30 - 60 - 60 = 30  (iii).
   
  BC1B1C é inscritível (num semi-círculo), pois BB1C = CC1B = 90 ==>
  BCC1 = BB1C1  e  CBB1 = CC1B1.
   
  Usando (i) e (ii), obtemos:
  BCC1 + C1B1C2 = 60   e   CBB1 + B1C1B2 = 60 ==>
  (BCC1 + CBB1) + (C1B1C2 + B1C1B2) = 120
   
  Finalmente, usando (iii):
  30 + (C1B1C2 + B1C1B2) = 120 ==>
  B1PB2 = C1CC2 = C1B1C2 + B1C1B2 = 90
   
  []s,
  Claudio.
   
  De:  [EMAIL PROTECTED]
  Para:  [EMAIL PROTECTED]
  Cópia:  
  Data:  Wed, 6 Dec 2006 04:01:11 -0200
  Assunto:  [obm-l] Problemas
Perdão, mas ainda não descobri como colocar problemas disponibilizados para 
todos os membros da lista, entaão estou enviando para a sua pessoa fazer-me 
essa gentiliza, obrigado.

Quebrei muito a cabeça mas não cheguei a lugar nenhum, por favor ajudem-me: 

1) Num triângulo ABC temos que o ângulo B é o dobro do ângulo C e a bissetriz 
AD divide BC em dois segmentos de modo que DC = AB. Determine o ângulo A: 
Resp.: 72º

2) Num triângulo acutângulo ABC, o ângulo A mede 30º, B1 e C1 são os pés das 
alturas marcadas pelos vértices B e C. Os pontos B2 e C2 são os pontos médios 
dos lados AC e AB respectivamente. Determine o ângulo  entre B1C2 e B2C1: 
Resp.: 90º

Obrigado pela atenção! 



-
 Você quer respostas para suas perguntas? Ou você sabe muito e quer 
compartilhar seu conhecimento? Experimente o Yahoo! Respostas!

Re: [obm-l] Problemas

2006-12-11 Por tôpico Conrado Costa

marcelo perdoe o atraso e o nao atendimentto do seu pedido, so vi sua
mensagem agora

abraco
como vai ser o verao?


[obm-l] Re:[obm-l] Problemas com Triângulo s

2006-12-07 Por tôpico claudio\.buffara
Solução do Problema 2:

Seja P = B1C2 inter B2C1.

AB1B e AC1C são triângulos retângulos de 30, 60 e 90 ==>
B1C2 = BC2 = AB/2  e  C1B2 = CB2 = AC/2 ==>
BB1C2 e CC1B2 são equiláteros ==>
BB1C1 + C1B1C2 = BB1C2 = 60   (i);
CC1B1 + B1C1B2 = CC1B2 = 60   (ii);
CBB1 + BCC1 = 180 - A - C2BB1 - B2CC1 = 180 - 30 - 60 - 60 = 30  (iii).

BC1B1C é inscritível (num semi-círculo), pois BB1C = CC1B = 90 ==>
BCC1 = BB1C1  e  CBB1 = CC1B1.

Usando (i) e (ii), obtemos:
BCC1 + C1B1C2 = 60   e   CBB1 + B1C1B2 = 60 ==>
(BCC1 + CBB1) + (C1B1C2 + B1C1B2) = 120

Finalmente, usando (iii):
30 + (C1B1C2 + B1C1B2) = 120 ==>
B1PB2 = C1CC2 = C1B1C2 + B1C1B2 = 90

[]s,
Claudio.

De:[EMAIL PROTECTED]

Para:[EMAIL PROTECTED]

Cópia:

Data:Wed, 6 Dec 2006 04:01:11 -0200

Assunto:[obm-l] Problemas

Perdão, mas ainda não descobri como colocar problemas disponibilizados para 
todos os membros da lista, entaão estou enviando para a sua pessoa fazer-me 
essa gentiliza, obrigado.

Quebrei muito a cabeça mas não cheguei a lugar nenhum, por favor ajudem-me:

1) Num triângulo ABC temos que o ângulo B é o dobro do ângulo C e a bissetriz 
AD divide BC em dois segmentos de modo que DC = AB. Determine o ângulo A:
Resp.: 72º

2) Num triângulo acutângulo ABC, o ângulo A mede 30º, B1 e C1 são os pés das 
alturas marcadas pelos vértices B e C. Os pontos B2 e C2 são os pontos médios 
dos lados AC e AB respectivamente. Determine o ângulo  entre B1C2 e B2C1:
Resp.: 90º

Obrigado pela atenção!


Re: [obm-l] Problemas

2006-11-10 Por tôpico Ronaldo Alonso
On 11/10/06, GERALDO FRANCISCO DE SOUZA REBOUÇAS <[EMAIL PROTECTED]> wrote:
1. quantos triangulos diferentes existem se levarmos em consideração apenas os angulos?    Suponha que  o primeiro ângulo seja c_1 no intervalo   0 < c_1 < pi    O segundo ângulo c_2 restinge o intervalo para 0 < c_2 < pi-c_1 
    O terceiro ângulo c_3 já é determinado, uma vez que c_1+c_2 = pi - c_3   Seja n o número de ângulos no primeiro intervalo e -n+p o número de ângulos possíveis no restantedo intervalo.  O número de triângulos é:
 n(p-n) = n*p - n^2 A solução deve seguir nesta linha. 
OBS.: Acho que esse problema tem varios niveis de dificuldade pois podemos resolve-lo com os angulos inteiros ou decimais.     2. dadas duas retas paralelas r e s de equações  ax + by + c = 0 e ax + by + d = 0 qual a relaçao existente entre os termos independentes c e d ?
  Basta notar que já que (a,b) é um vetor normal à reta então ele determina o ângulo que a reta forma com o eixo dos xe c e d tem que ser diferentes, caso contrário as retas coincidem.
[]sRonaldo  Aguardo respostas   
 
		 
Yahoo! Acesso Grátis - Internet rápida e grátis. Instale o discador agora! 

-- Ronaldo Luiz Alonso--Computer Engeener LSI-TEC/USP - Brazil.


Re: [obm-l] Re: [obm-l] Problemas de C�lculo

2006-10-26 Por tôpico LEANDRO L RECOVA


O livro do Elon de Espacos Metricos seria um bom comeco na minha opiniao. 
Eu, particularmente, acho muito interessante. Para um curso mais avancado, o 
livro do Munkres seria outra opcao.


Leandro.


From: Bruno Carvalho <[EMAIL PROTECTED]>
Reply-To: obm-l@mat.puc-rio.br
To: obm-l@mat.puc-rio.br
Subject: Re: [obm-l] Re: [obm-l] Problemas de Cálculo
Date: Thu, 26 Oct 2006 13:26:06 + (GMT)

Prezados, agradeço a boa vontade de todos vocês.
  O enunciado geral daquelas questões é  verificar se elas são falsas ou 
verdadeiras  justificando em cada um dos casos.


  Esclarecendo as dúvidas geradas pela minha mensagem.

  Ao Arthur.

  No ítem  b :  I é realmente um intervalo de R.
  E o objetivo desse ítem é verificar se o limite existe, dadas aquelas 
condições.


  Ao salhab.

  No ítem d : a função f dada é: f(x,y,z)=x^2+y^2

  ´Mais uma vez agradeço .
  Peço , também, uma orientação para um livro de Topologia  " para 
iniciantes", ( estou encontrando algumas dificuldades nessa disciplina).


  Um abraço.

  Bruno


Marcelo Salhab Brogliato <[EMAIL PROTECTED]> escreveu:
  Olá,

  d) superficies de nivel sao aquelas nas quais a funcao é constante..
  isso é: f(x, y) = c . x^2 + y^2 = c ... que são circunferencias 
centradas na origem de raio igual a raiz(c).

  logo, sao cilindros.

  e) a mesma coisa... f(x, y) = c  arctg(xy) = c ... xy = tg(c) + k*pi 
.. que, para cada valor de k, sao hiperboles.


  nas outras, da uma confirmada, mas acho que é f: R^2->R

  abracos,
  Salhab

- Original Message -
  From: Bruno Carvalho
  To: obm-l@mat.puc-rio.br
  Sent: Wednesday, October 25, 2006 10:48 AM
  Subject: [obm-l] Problemas de Cálculo


  Pessoal, bom dia !

  Peço ajuda para resolver os seguintes problemas

  a) Se F é um conjunto fechado em R^2 e z1 e z2 são elementos de R^2-F 
então FU {z1,z2} é fechado em R^2.


  b)Considere as funções f:R^2->R , g:I ->R^2 e h:I->R^2 sendo g e h 
contínuas e injetoras.Se to pertence a I, g(to) =h(to)=(xo,yo e Lim 
f(g(t))=Limf(h(t)) quando  t->to então Lim(x,y) quando (x,y)->(xo,yo) 
existe.


  c)Seja f:R^2->R, definida por f(x,y)=x*sen(1/y) se x for diferente de 
zero e

   f(x,0)=0.Temos que Limf(x,y)=0 quando (x,y)->(0,0) mas Limf(x,y) quando
  y->0 não existe.

  d)As superfícies de nível da função f(x,y,z)=x^2+y^2 são cilindros e o 
eixo z.


  e) As curvas de nível da função f(x,y)=arctg(x,y) são hipérboles e os 
eixos

  x e y

  f) Se f:R->R^2 é contínua em (0,0) então Df/Dx(0,0) existe.

  g) Se f:R-R^2 é tal que Df/Dx(0,0) existe então f é contínua em (0,0).

  h) Se u=( raiz de 3/2 ,1/2) e f:R->R^2 é definida por f(x,y)= 
x^3/(x^2+y^2), se

  (x,y) diferente de (0,0) e f(0,0)=(0,0) então Df/Du.(0,0)=3*raiz de 3.

  Desde já agradeço qualquer orientação.

  Mais uma vez, obrigado.

  Bruno
  __
Fale com seus amigos de graça com o novo Yahoo! Messenger
http://br.messenger.yahoo.com/
-

No virus found in this incoming message.
Checked by AVG Free Edition.
Version: 7.1.408 / Virus Database: 268.13.11/496 - Release Date: 24/10/2006



-
 Yahoo! Search
 Música para ver e ouvir: You're Beautiful, do James Blunt



=
Instruções para entrar na lista, sair da lista e usar a lista em
http://www.mat.puc-rio.br/~nicolau/olimp/obm-l.html
=


Re: [obm-l] Re: [obm-l] Problemas de Cálculo

2006-10-26 Por tôpico Bruno Carvalho
Prezados, agradeço a boa vontade de todos vocês.  O enunciado geral daquelas questões é  verificar se elas são falsas ou verdadeiras  justificando em cada um dos casos.     Esclarecendo as dúvidas geradas pela minha mensagem.     Ao Arthur.     No ítem  b :  I é realmente um intervalo de R.  E o objetivo desse ítem é verificar se o limite existe, dadas aquelas condições.     Ao salhab.     No ítem d : a função f dada é: f(x,y,z)=x^2+y^2     ´Mais uma vez agradeço .   Peço , também, uma orientação para um livro de Topologia  " para iniciantes", ( estou encontrando algumas dificuldades nessa disciplina).     Um abraço.     Bruno    
 Marcelo Salhab Brogliato <[EMAIL PROTECTED]> escreveu:  Olá,     d) superficies de nivel sao aquelas nas quais a funcao é constante..  isso é: f(x, y) = c . x^2 + y^2 = c ... que são circunferencias centradas na origem de raio igual a raiz(c).  logo, sao cilindros.     e) a mesma coisa... f(x, y) = c  arctg(xy) = c ... xy = tg(c) + k*pi .. que, para cada valor de k, sao hiperboles.     nas outras, da uma confirmada, mas acho que é f: R^2->R     abracos,  Salhab   - Original Message -   From: Bruno Carvalho   To: obm-l@mat.puc-rio.br   Sent: Wednesday, October 25, 2006 10:48 AM  Subject: [obm-l] Problemas de Cálculo 
   Pessoal, bom dia !     Peço ajuda para resolver os seguintes problemas     a) Se F é um conjunto fechado em R^2 e z1 e z2 são elementos de R^2-F então FU {z1,z2} é fechado em R^2.     b)Considere as funções f:R^2->R , g:I ->R^2 e h:I->R^2 sendo g e h contínuas e injetoras.Se to pertence a I, g(to) =h(to)=(xo,yo e Lim f(g(t))=Limf(h(t)) quando  t->to então Lim(x,y) quando (x,y)->(xo,yo) existe.     c)Seja f:R^2->R, definida por f(x,y)=x*sen(1/y) se x for diferente de zero e   f(x,0)=0.Temos que Limf(x,y)=0 quando (x,y)->(0,0) mas Limf(x,y) quando   y->0 não existe.     d)As superfícies de nível da função f(x,y,z)=x^2+y^2 são cilindros e o eixo z.     e) As curvas de nível da função f(x,y)=arctg(x,y) são hipérboles e os
 eixos    x e y     f) Se f:R->R^2 é contínua em (0,0) então Df/Dx(0,0) existe.     g) Se f:R-R^2 é tal que Df/Dx(0,0) existe então f é contínua em (0,0).     h) Se u=( raiz de 3/2 ,1/2) e f:R->R^2 é definida por f(x,y)= x^3/(x^2+y^2), se  (x,y) diferente de (0,0) e f(0,0)=(0,0) então Df/Du.(0,0)=3*raiz de 3.     Desde já agradeço qualquer orientação.     Mais uma vez, obrigado.     Bruno   __Fale com seus amigos de graça com o novo Yahoo! Messenger http://br.messenger.yahoo.com/ No virus found in this incoming message.Checked by AVG Free Edition.Version: 7.1.408 / Virus Database: 268.13.11/496 - Release Date: 24/10/2006 
		 
Yahoo! Search 
Música para ver e ouvir: You're Beautiful, do James Blunt

[obm-l] Re: [obm-l] Problemas de Cálculo

2006-10-25 Por tôpico Marcelo Salhab Brogliato



Olá,
 
d) superficies de nivel sao aquelas nas quais a 
funcao é constante..
isso é: f(x, y) = c . x^2 + y^2 = c ... que são 
circunferencias centradas na origem de raio igual a raiz(c).
logo, sao cilindros.
 
e) a mesma coisa... f(x, y) = c  arctg(xy) = c 
... xy = tg(c) + k*pi .. que, para cada valor de k, sao hiperboles.
 
nas outras, da uma confirmada, mas acho que 
é f: R^2->R
 
abracos,
Salhab
 

  - Original Message - 
  From: 
  Bruno 
  Carvalho 
  To: obm-l@mat.puc-rio.br 
  Sent: Wednesday, October 25, 2006 10:48 
  AM
  Subject: [obm-l] Problemas de 
  Cálculo
  
  Pessoal, bom dia !
   
  Peço ajuda para resolver os seguintes problemas
   
  a) Se F é um conjunto fechado em R^2 e z1 e z2 são elementos de R^2-F 
  então FU {z1,z2} é fechado em R^2.
   
  b)Considere as funções f:R^2->R , g:I ->R^2 e h:I->R^2 
  sendo g e h contínuas e injetoras.Se to pertence a I, g(to) =h(to)=(xo,yo e 
  Lim f(g(t))=Limf(h(t)) quando  t->to então Lim(x,y) quando 
  (x,y)->(xo,yo) existe.
   
  c)Seja f:R^2->R, definida por f(x,y)=x*sen(1/y) se x for diferente de 
  zero e
   f(x,0)=0.Temos que Limf(x,y)=0 quando (x,y)->(0,0) mas Limf(x,y) 
  quando 
  y->0 não existe.
   
  d)As superfícies de nível da função f(x,y,z)=x^2+y^2 são cilindros e 
  o eixo z.
   
  e) As curvas de nível da função f(x,y)=arctg(x,y) são hipérboles e os 
  eixos  
  x e y
   
  f) Se f:R->R^2 é contínua em (0,0) então Df/Dx(0,0) existe.
   
  g) Se f:R-R^2 é tal que Df/Dx(0,0) existe então f é contínua em 
  (0,0).
   
  h) Se u=( raiz de 3/2 ,1/2) e f:R->R^2 é definida por f(x,y)= 
  x^3/(x^2+y^2), se
  (x,y) diferente de (0,0) e f(0,0)=(0,0) então Df/Du.(0,0)=3*raiz de 
  3.
   
  Desde já agradeço qualquer orientação.
   
  Mais uma vez, obrigado.
   
  Bruno 
  __Fale com seus amigos 
  de graça com o novo Yahoo! Messenger http://br.messenger.yahoo.com/ 
  
  

  No virus found in this incoming message.Checked by AVG Free 
  Edition.Version: 7.1.408 / Virus Database: 268.13.11/496 - Release Date: 
  24/10/2006


Re: [obm-l] Problemas de Olimpiadas

2006-09-25 Por tôpico saulo nilson
+
On 9/20/06, Paulo Santa Rita <[EMAIL PROTECTED]> wrote:
Ola Pessoal,( escreverei sem usar acentos )Mantendo a tradicao desta nossa lista, que,  conforme diz a pagina da OBM no
enderecohttp://www.obm.org.br/frameset-lista.htmfoi concebida originalmente para a discussao de problemas olimpicos e naopara a solucao dos trivialissimos  problemas de vestibulares e/ou de
concursos, seguem abaixo 5 problemas das Olimpiadas Russas. Estes problemassao direcionados sobretudo aos nossos estudantes olimpicos do fim do nivelfundamental ( antiga 7/8 series ).PROBLEMA 1)  Dois jogadores escolhem, alternadamente, o sinal de um dos
números 1, 2, 3, ... 20. Desde que o sinal de um número foi escolhido, elenão poderá ser modificado.  Após todos os números terem recebido sinal, éefetuado a soma algébrica dos números e, a seguir, tomado o valor absoluto
desta soma. O primeiro jogador procura minimizar o valor absoluto da soma,enquanto que o segundo jogador procura maximiza-lo.  Como pode ser oresultado final, supondo-se que cada jogador joga com perfeição ?
PROBLEMA 2) Os dígitos de um número natural são reordenados e o númeroresultante é acrescido ao número original. Prove que a resposta não pode serum número formado apenas com o algarismo nove. Prove também que se a
resposta for 10^10, então o número original é divisível por 10.PROBLEMA 3) Prove que existe um número divisível por 5^1000 que não temdígito zero.PROBLEMA 4 ) Três vértices KLM de um losango KLMN são pontos respectivamente
dos lados AB, BC e  CD de um quadrado de lado unitário. Encontre a área doconjunto de todos os possíveis valores do vértice N.PROBLEMA 5 ) Um número natural K tem a propriedade de que se K divide N,então o número obtido N pela reversão de seus dígitos é também divisível por
K. Prove que K é um divisor de 99 ( Reversão dos dígitos de N significa queo primeiro dígiton passa a ser o último, o segundo passa a ser o penúltimo eassim sucessivamente )Mais problemas de Olimpiadas Russas em :
http://www.mat.puc-rio.br/~nicolau/psrUm Abracao a Todos !Paulo Santa Rita4,1701,200906_
Chegou o Windows Live Spaces com rede social. Confirahttp://spaces.live.com/=Instruções para entrar na lista, sair da lista e usar a lista em
http://www.mat.puc-rio.br/~nicolau/olimp/obm-l.html=



Re: [obm-l] PROBLEMAS DE ESCALA!

2006-08-27 Por tôpico Rogerio Ponce
Oi Jorge,juro que nao conheco sua Tia!Quanto `a afirmacao de que "minha projecao poliedrica foi o mais didatica possivel", eu diria que no minimo voce foi caridoso...Talvez com boa vontade seja possivel acompanhar o que eu pretendi explicar, mas se alguem se atrapalhar com "a projecao", e' so' perguntar que eu tentarei explicar melhor - prometo! Grande abraco,Rogerio PonceJorge Luis Rodrigues e Silva Luis <[EMAIL PROTECTED]> escreveu: Valeu Rogério e demais colegas! A sua projeção poliédrica foi a mais didática e higiênica possível. Coincidências à parte, Rex é o nome do cachorro da minha Tia e melhor amigo do homem, por não conhecer dinheiro.. 
		 
Yahoo! Acesso Grátis - Internet rápida e grátis. Instale o discador agora! 


Re: [obm-l] PROBLEMAS INSTIGANTES!

2006-08-19 Por tôpico Rogerio Ponce
Ola' Jorge,seu comentario e' muito bem vindo pois alguns tem certa dificuldade em compreender a expressao "dimensoes lineares" .  Meu amigo Rex, por exemplo, ontem mesmo me perguntou : "Mas as dimensoes de um poligono nao sao sempre lineares? "  Pensei em explicar-lhe como outras grandezas com dimensoes diferentes de 'comprimento'  podem ser utilizadas na avaliacao do tamanho de um objeto. Mas Rex nao precisava de tanto.  Disse-lhe apenas que um poligono tambem pode ser avaliado por outras medidas (ou dimensoes) que nao as lineares. A area, por exemplo,  e' uma grandeza comumente utilizada.     Assim, no dia a dia, encontramos tanto "lotes de 900m2 `a venda"  quanto "lotes de 30m x 30m" .No primeiro anuncio, a dimensao do terreno e' dada pela area, enquanto no segundo anuncio, as dimensoes lineares sao fornecidas.  Rex, que nunca foi `a escola mas aprende
 facil, entendeu perfeitamente essa singela explicacao, e foi-se embora, satisfeito.     Grande abraco,Rogerio PonceJorge Luis Rodrigues e Silva Luis <[EMAIL PROTECTED]> escreveu:  OK! Rogério e demais colegas! Grato pelo contra exemplo e parabéns pelo magistral emprego do termo "Dimensões Lineares"... 
		 
Yahoo! Search 
Música para ver e ouvir: You're Beautiful, do James Blunt

Re: [obm-l] Problemas

2006-08-06 Por tôpico Marcelo Salhab Brogliato



Olá,
 
1) Para termos isso, basta que o (p-1) > 0 e que 
delta < 0
assim:
p > 1
delta = (2p-2)^2 - 4*(p-1) < 0 . 4(p-1)^2 - 
4(p-1) < 0  (p-1) - 1 < 0  p < 2
logo: 1 < p < 2 
 
2) tem que resolver as 2 desigualdades e fazer a 
intersecao, assim:
 
-2x^2 + 2x <= -4 . 2x^2 - 2x - 4 >= 0 
 x^2 - x - 2 >= 0
delta = 1 + 8 = 9 ... raizes: (1+-3)/2 => 2 
e -1 
assim: x^2 - x - 2 >= 0   
<==>   x <= -1 ou x>= 2
 
-2x^2 + 2x > 3x-1  2x^2 + x - 1 < 
0
delta = 1 + 8 = 9 ... raizes: (-1 +- 3)/4 => -1 
e 1/2
assim: 2x^2 + x - 1 < 0   
<==>  -1 < x < 1/2
 
a interseccao é vazia.. logo, o conjunto solucao é 
vazio.
 
abraços,
Salhab
 

  - Original Message - 
  From: 
  Natan 
  Padoin 
  To: obm-l@mat.puc-rio.br 
  Sent: Saturday, August 05, 2006 10:48 
  PM
  Subject: [obm-l] Problemas
  
  Olá pessoal,
   
  Alguém pode me ajudar a resolver os seguintes problemas:
   
  1) Determine "p" para que se tenha (p-1)x^2+(2p-2)x+p+1>0
   
  2) Dar o conjunto solução de -4 >= (maior ou igual) -2x^2+2x> 
  3x-1
   
  Desde já agradeço a atenção. Abraço!
  
  
  Novidade no Yahoo! Mail: receba alertas de novas mensagens no seu celular. Registre 
  seu aparelho agora!
  
  

  No virus found in this incoming message.Checked by AVG Free 
  Edition.Version: 7.1.394 / Virus Database: 268.10.7/410 - Release Date: 
  5/8/2006


  1   2   3   4   >